Masterclass Book Part 2

March 10, 2017 | Author: Eng Kian Ng | Category: N/A
Share Embed Donate


Short Description

MRCP Paer 2...

Description

CONTENTS 1. CARDIOLOGY 2. RESPIRATORY 3. NEPHROLOGY 4. NEUROLOGY 5. GIT 6. RHEUMATOLOGY 7. HAEMATOLOGY 8. ONCOLOGY 9. ENDOCRINOLOGY 10. INFECTIOUS DIASEASES 11. EMERGENCY MEDICINE 12. DERMATOLOGY 13. PSYCHIATRY 14. OPTHALMOLOGY 15. PHARMA/TOXOCOLOGY 16. STATISTICS 17. PLATES

1 53 95 120 156 202 230 246 264 287 301 310 324 333 343 361 367

1. A 46-year-old woman with permanent atrial fibrillation is experiencing rapid palpitations due to an elevated heart rate. Which from the following list of drugs are the three most appropriate for controlling ventricular rate response? A: Adenosine B: Amiodarone C: Aspirin D: Atenolol E: Digoxin F: Doxasocin G: Flecainide H: Isoprenaline I: Lidocaine J: Magnesium K: Mexilitine L: Moxonidine M: Propafenone N: Verapamil O: Warfarin. Comment : Beta-blockers, cardiac glycosides and calcium channel antagonists all have a negative chronotropic effect on the atrio ventricular (AV) node and are very useful in the acute and long-term control of ventricular rate in AF. Digoxin and verapamil however should be avoided in the rare patients who also have Wolf-Parkinson-White syndrome. While Amiodarone, Flecainide and Propafenone are commonly used to chemically cardiovert AF to sinus rhythm, they have only a small effect on AV nodal conduction and should not be chosen for rate control alone. Adenosine causes rapid but only very brief (a few seconds) blockade of AV nodal conduction. Lidocaine, Mexilitine and Magnesium are used only for the control of ventricular arrhythmias. Warfarin and Aspirin are commonly used in patients with AF for preventing thromboembolic complications. Isoprenaline is a beta agonist and would accelerate ventricular response in AF.Doxazosin and Moxonidine are antihypertensive agents. D : E : N: 2.

A 19-year-old man presents to accident and emergency with a fast pounding in his throat and presyncope. He is found to be tachycardic and his ECG is as shown. What is the most likely arrhythmia? A: Atrial fibrillation (AF) B: Ventricular tachycardia (VT) C: Atrial tachycardia D: Atrio-ventricular nodal re-entry tachycardia (AVNRT) E: Atrial flutter. Comment : The ECG shows a regular narrow complex tachycardia at a rate of 170. This excludes VT (broad complex)and AF(irregular).Although atrial flutter and atrial tachycardia are possible diagnoses there are no visible p waves,which would be expected. The most likely diagnosis by far is AVNRT,which is caused by a re-entry circuit formed within the AV node causing atria and ventricles to be activated simultaneously.This can cause reflux of jugular venous blood and a sensation of pounding in the throat. D:

1

MOHAMMED IS-HAG

3. A 65-year-old lady is admitted with left sided pneumonia and pleural effusion. Pleural fluid is aspirated and sent for tests. Which of the following is an indication for inserting a chest drain? A: Pleural fluid pH 2mmol/l E: Pleural fluid lactate dehydrogenase > 200IU/l. Comment : Infected pleural effusions should be drained. Infected pleural effusions should be drained if the pH 60%), triple vessel disease and impaired left ventricular function and triple vessel disease with proximal left anterior descending artery disease, when compared to medical therapy. Angioplasty is exceedingly high risk and therefore is only contemplated when patients are deemed as non-operative candidates. C : 5. A 68-year-old female is referred to outpatients with a three-month history of dyspnoea and significant peripheral oedema. Clinical findings are solely of the oedema. Echocardiography demonstrates tickened myocardium with impaired relaxation and a bright speckled appearance. What is the likely diagnosis? A: Hypertrophic cardiomyopathy B: Cor pulmonale C: Sarcoidosis D: Amyloidosis E: Secondary neoplastic myocardial deposition. Comment : The commonest cause of death in amyloidosis is secondary to cardiac involvement. It may present with the insidious onset of vague symptoms, such as lethargy. Later peripheral oedema becomes a prominent feature.Amyloid depositis in the heart produce generalized thickening of the myocardium (as opposed to asymmetrical septal hypertrophy commonly seen in hypertrophic

2

MOHAMMED IS-HAG

cardiomyopathy) and diastolic dysfunction. This produces a "stiffened" appearance to relaxation in the diastolic phase.A search for non-cardiac amyloid deposits is usually the most efficient way to coinfirm the diagnosis histologically. Cardiac biopsy may be useful. The treatment of cardiac amyloid is supportive and generally it carries a very poor prognosis. D : 6.

A 75-year-old woman presents with palpitations. Her ECG shows: A: Atrial fibrillation B: Atrial fibrillation with acute inferior myocardial infarction C: Atrial flutter D: Atrial flutter with acute inferior myocardial infarction E: Atrioventricular re-entrant tachycardia. Comment : The ECG shows atrial flutter with 4/1 block. There are no features to suggest myocardial infarction. C : 7. Which of the following statements regarding the natriuretic peptides ANP (atrial natriuretic peptide) and BNP (brain natriuretic peptide) is not correct? A: They are both vasodilators. B: BNP is produced by the ventricular myocardium. C: They inhibit sodium reabsorption in the collecting duct of the kidney. D: They stimulate aldosterone production in the adrenal glomerulosa. E: ANP is released in response to atrial stretch. Comment : ANP and BNP are members of the natriuretic peptide family. As its name suggests, ANP is primarily released from the atria in response to enhanced stretch. Although first discovered in porcine brain, BNP is found in large quantities in ventricular myocardium. Release of BNP is thought to occur in response to increased ventricular wall stress. ANP and BNP have similar biological properties, which include vasodilatation, natriuresis and diuresis. ANP and BNP have direct effects on renal function. These include increasing glomerular filtration rate as a consequence of efferent glomerular arteriolar vasoconstriction and afferent vasodilatation. In addition, they directly inhibit sodium reabsorption in the collecting tubules. Both ANP and BNP also inhibit renin and aldosterone release. Plasma ANP and BNP levels are elevated in patients with chronic heart failure and indeed levels may be used to aid in the diagnosis of heart failure. Furthermore, plasma ANP and BNP are helpful in providing prognostic assessment of patients with heart failure and post myocardial infarction. In view of the potential beneficial haemodynamic effects of ANP and BNP, there is current interest in the therapeutic manipulation of the natriuretic peptide system in patients with heart failure. This could be achieved by inhibiting their breakdown (neutral endopeptidase inhibitors) or infusing exogenous synthetic peptide. D :

3

MOHAMMED IS-HAG

8. A 42-year-old man reviewed in outpatients has a 6-month history of increasing shortness of breath on exertion and feelings of lightheadedness when digging in his garden. His general practitioner organised an open access echocardiogram which showed a septal thickness of 26mm and a left ventricular outflow gradient of 85mmHg. Which of the following is NOT a risk factor for sudden death in patients with hypertrophic cardiomyopathy? A: Unexplained syncope B: Sudden death from HOCM in 2 or more first degree relatives 30mm E: Hypertension. Comment : Major risk factors for hypertrophic cardiomyopathy are: · Cardiac arrest (ventricular fibrillation) · Spontaneous sustained ventricular tachycardia · Family history of sudden death · Minor risk factors · Unexplained syncope · Left ventricualr wall thickness>30mm · Abnormal blood pressure on exercise(failure to rise from baseline by 25mmHg) · Non sustained Ventricular tachycardia · Left ventricular outflow obstruction · Microvascular obstruction · High risk genetic defect. E : 9. A 42-year-old man visits his GP, complaining of a recent onset of shortness of breath and dizziness upon exertion. His GP arranges for an echocardiogram, which showed hypertrophic obstructive cardiomyopathy (HOCM) with thickening of the septal wall and a left ventricular outflow gradient of 86mmHg. The man is referred to a cardiology outpatient clinic, where he volunteers that his father died suddenly of a heart attack aged 38 years of age and his younger brother died aged 17 years of age during a rugby scrum more than 20 years ago. He is not sure of the cause of death for his younger brother but does not think a post-mortem was ever carried out. He has two teenaged sons, both of whom are well, but his 14-year-old son fainted after scoring a goal for the school team last week. Which of the following statements are correct? A: The majority of patients with HOCM are symptomatic throughout life. B: The overall mortality in patients with HOCM is 50% per year. C: All first degree family members should undergo screening every 6 months. D: All patients should receive advice on avoiding dehydration and strenous exercise. E: Patients with HOCM do not need prophylaxis against infective endocarditis. Comment : · The majority of patients with HOCM are ASYMPTOMATIC. · The overall mortality in patients with HOCM is 85. However ambulatory monitoring demonstrated a mean BP of 130/75. Which of the following are inappropriate indications for ambulatory blood pressure monitoring? A: Deciding diagnosis in borderline hypertension B: Making a diagnosis of left ventricular failure C: Identification of nocturnal hypertension D: Diagnosing Phaeochromocytoma E: Hypertensive patients resistant to treatment F: Hypertension of pregnancy G: Diagnosis of hypotension H: Monitoring hypertensive tretament I: Elderly patients J: Exclusion of white coat hypertension. A diagnosis of "white coat hypertension" was made in this lady and thus she did not require Comment : antihypertensive treatment. This is probably the strongest indication for ambulatory BP monitoring but the indications are broad. A diagnosis of left ventricular failure will not be made on BP alone. Equally a phaeochromocytoma may have transient rises in BP but as most ambulatory monitors will only measure once or twice an hour it is unlikely to be helpful in this situation B : D :

7

MOHAMMED IS-HAG

20. A 40-year-old woman was referred for investigation of shortness of breath on effort over the preceding 1 year. Data from her cardiac catheterisation iare as follows: Pressure (mmHg) Oxygen saturation (%) SVC 15 64 IVC 13 65 RA 13 82 RV 51/7 85 PA 53/31 83 LV 134/3 99 Aorta 135/82 99 Which of the following are true concerning her diagnosis? A: This condition rarely presents in adulthood. B: Paradoxical splitting of the second sound is common. C: A rumbling mid diastolic murmur may occur. D: A systolic murmur is rare. E: The ECG often shows left bundle branch block. F: The chest radiograph is usually normal. G: Echocardiography shows left ventricular dilatation. H: Medical treatment is recommended in this case. I: Arrhythmias are rare with this condition. J: In children, this condition may be associated with skeletal abnormalities. Comment : This patient has a secundum atrial septal defect as shown by a step-up in oxygen saturations at atrial level. She also has pulmonary hypertension. Characteristic findings on auscultation include a wide fixed-split second sound, ejection systolic murmur and a mid diastolic murmur. The murmurs are caused by increased flow across the pulmonary and tricuspid valves respectively. The ECG often shows a right bundle branch block pattern and the chest radiograph shows right ventricular enlargement and prominent pulmonary arteries. Surgical or percutaneous closure is recommended in patients with symptoms, right ventricular dilatation, or a pulmonary: systemic flow ratio > 1.5. The HoltOram syndrome is the association of an atrial septal defect with bony abnormalities of the extremities. C : J: 21. A 57-year-old diabetic man with stable angina wishes to use Sildenafil (Viagra) to treat impotency. Concomitant use of which two of the following cardiac medications is contraindicated? A: Aspirin B: Clopidogrel C: Atenolol D: Amlodipine E: Digoxin F: Isosorbide mononitrate G: Lisinopril H: Nicorandil I: Simvastatin J: Warfarin. Comment : Sildenafil enhances Nitric Oxide (NO) mediated smooth muscle relaxation by blocking the degradation of cGMP. Although it is relatively selective to the corpus cavernosum it does still have some systemic effect. Both Isosorbide Mononitrate and Nicorandil release NO and their action is potentiated by Sildenafil, which can lead to profound hypotension. The other commonly used cardiac medications listed have no specific interaction with Sildenafil. F:H: 22. A 56-year-old man develops cardiogenic shock 8 hours after admission with an anterior myocardial infarction. He was thrombolysed with tPA on admission with rapid resolution of the ST elevation. He has a loud pansystolic murmur which is new. Which of the following are inappropriate in the management of this patient? A: Repeat thrombolysis B: Coronary angiography C: Intra aortic balloon pump D: Intravenous beta-blockers E: Inotropic support F: Urgent echocardiography G: Swann–Ganz pulmonary artery catheter H: Intravenous diamorphine I: Transfer to cardiothoracic surgical centre J: Consider positive pressure ventilation.

8

MOHAMMED IS-HAG

Comment : Increasing revascularisation both with coronary artery bypass grafting (CABG) and percutaneous coronary intervention (PCI) has seen reductions in the number of deaths following acute myocardial infarction. However, shock remains the leading cause of death in hospital. It is clear that this patient has most likely developed a post MI ventricular septal defect or acute mitral regurgitation. In the absence of new ECG changes, thrombolysis is clearly contraindicated. The use of intravenous beta-blockers in the shocked patient is also relatively contraindicated. The mainstay of treatment is therefore supportive and surgical intervention, if appropriate. A : D : 23. A 65-year-old man presents with increasing breathlessness and swelling of his legs. These symptoms have occurred over a period of 2 months. He has otherwise been previously well, apart from a 2-year history of diarrhoea which his gastroenterologists have been investigating. On echo, he is found to have severe tricuspid regurgitation and a diagnosis of carcinoid syndrome is made. Which of the following statements about carcinoid syndrome is NOT true? A: Common sites of carcinoid tumours are the appendix and terminal ileum. B: After the tricuspid valve, the mitral valve is most commonly affected. C: Patients with cardiac carcinoid ususally die from valvular disease rather than carcinomatosis. D: Patients with cardiac involvement usually have more advanced disease. E: Patients with cardiac involvement usually have higher levels of 5-HIAA. Comment : Carcinoid heart disease is rare and usually affects the right-sided valves (i.e. tricuspid and pulmonary valves). Cardiac involvement is associated with more advanced disease and carries with it a poorer prognosis. It is associated with higher circulating levels of 5-HIAA. Most patients present with right-sided heart failure. Treatment of cardiac carcinoid involves management of the right-sided valve failure, pharmacotherapy to reduce secretion of tumour products and surgical intervention for the valvular pathology. B : 24. A 21-year-old university student complains of difficulty sleeping. She is in the middle of sitting her final exams and would like some medication for a few days to help her sleep. However, she is concerned about potential 'hang-over' effects and would prefer a drug which doesn't cause daytime drowsiness. Which agent would you prescribe? A: Diazepam B: Midazolam C: Promethazine D: Loprazolam E: Clomethiazole. Comment : Diazepam has a long half-life, principally because of its active metabolites. Midazolam is short-acting but is only used intravenously. Promethazine is an antihistamine with a 12-hour half-life and may cause daytime sedation. Clomethiazole is less safe in overdose, has dependence potential and is only licensed for sedation in the elderly. Loprazolam is short-acting (half-life 6–12 hours) and would be a reasonable choice. D : 25. A 68-year-old man with a past history of myocardial infarction, from which he made a good recovery, now presents with chest pain that might be due to cardiac ischaemia, but the history is not typical. He would not be able to perform a Bruce protocol treadmill test because of severe osteoarthritis of his knees and it is decided to perform radionucleotide myocardial perfusion imaging. Which two of the following statements about such imaging are correct? A: It should be avoided in asthmatic patients. B: It should be avoided in patients with aortic stenosis. C: It is useful in determining the functional severity of coronary lesions but does not predict prognosis. D: It can be used to risk stratify patients undergoing surgical procedures. E: It is not helpful in patients unable to physically exercise to maximal capacity. F: It is readily possible to differentiate between reversible ischaemia and infarcted areas of myocardium. G: It is of no value following coronary angiography. H: False positives are seen more commonly in middle aged women as compared to conventional exercise testing I: It should not be used in patients with very poor left ventriuclar function. J: A normal image indicates risk of of coronary events of 2-3% per year. Comment : Radionucleotide myocardial perfusion imaging is very useful in assessing the degree of coronary obstruction in patients with suspected coronary artery disease. It provides diagnostic and prognostic data. Patients can be stressed using conventional treadmill or pharmacological stress with agents such as adenosine or dobutamine. Adenosine should be avoided in asthmatics. A normal image indicates risk of coronary events of 3.5mmol/L and his HDL is 70 mmHg). Severe aortic stenosis can still be present with gradients < 70 mmHg, when left ventricular function is impaired. B 82. A 48-year-old female is referred to the clinic with palpitations. There are no other associated symptoms. She does however have a family history of sudden cardiac death. She has a normal ECG and echocardiogram. Her 24 hour ECG demonstrates frequent ventricular ectopic activity. Which of the following is the likely diagnosis?

A: B: C: D: E:

Brugada syndrome Long QT syndrome Hypertrophic obstructive cardiomyopathy Arrhythmogenic right ventricular dysplasia None of the above.

Comment : Palpitations are a very common presenting feature and in the context of a normal ECG and echo, reassurance is generally all that is needed. In this lady the most concerning feature is a history of sudden cardiac death in the family. More information regarding this is therefore required to assess her risk. All of the others generally will have abnormal ECGs but in the absence of a specific family history of them it is likely that she will need no further investigation. E :

28

MOHAMMED IS-HAG

83.

PLATE 8 A 75-year-old man had a pacemaker implanted 18 months earlier. He presents with pain over his pacemaker. This is what you find (see image). What is the appropriate management? A: Intravenous antibiotics and closure of the wound B: Reassurance C: Replace pacemaker generator and treat with antibiotics D: Sterile dressings and wait for healing via secondary intention E: None of the above. Comment : The generator has eroded through the chest wall skin. This denotes infection and therefore requires extraction of the whole pacemaker system, i.e. the generator and leads. It is inevitable that the leads will be infected as well. Replacement of the generator alone will be inadequate. Pacing system infection is rare (10% and >200 ml as a positive response. E: Give oral prednisolone, 30 mg daily for 2 weeks, regarding a clear statement of subjective improvement by the patient as a positive response. Comment : Steroid challenge is indicated in chronic obstructive pulmonary disease (COPD) of more than moderate severity. Standard practice would be to give prednisolone 30 mg daily for 2 weeks, regarding an increase in FEV1 of >10% and >200 ml as a positive response. Given the non-specific effects and many side effects of steroids, it is crucial to demonstrate functional improvement: many patients with COPD have sustained severe complications of steroid treatment, e.g. vertebral fracture, without any evidence that the steroids were beneficial for their chest. D :

31

MOHAMMED IS-HAG

93. A 60-year-old man is given streptokinase for an anterior myocardial infarction (MI) six hours after the onset of pain. Ninety minutes post-thrombolysis he continues to have ischaemic sounding chest pain with persistent anterior ST elevation. Which of the following is the established treatment in this situation: A: Intravenous heparin. B: Further thrombolysis with streptokinase C: Further thrombolysis with tPA D: Emergency angiography and target PTCA or bypass surgery E: None of the above. Comment : The best way to manage patients with failed thrombolysis has yet to be determined. Continued ischaemic pain and failure of >50%resolution of STelevation are the most accurate predictors of failed reperfusionThe UK REACT study aims to resolve the most appropriate treatment strategy in this clinical setting. This multicentre, prospective, randomised trial compares intravenous heparin, repeat thrombolysis with rPA, and rescue percutaneous transluminal coronary angioplasty (PTCA) in patients with evidence of failed thrombolysis still within 12 hours of presentation. E : 94. A 34-year-old male known to have AIDS presents with dyspnoea and cardiomegaly. His echo demonstrates a dilated poorly functioning left ventricle. Which of the following are not common cardiac manifestations of HIV/AIDS? A: Aortic dissection B: Pericardial effusion C: Endocarditis D: Myocarditis E: Pulmonary hypertension. Comment : Cardiac complications of HIV infection are common with myocardial, pericardial and endocardial involvement. This is a result of not only direct involvement as a result of HIV infection, but also the cardiotoxicity of the drugs used in the management of HIVinfected patients.Aortic dissection is not a

common recognized complication. A: 95. A 43-year-old male with myotonic dystrophy is admitted with ventricular tachycardia. This requires DC cardioversion as he is hypotensive. Ultimately he has an implantable cardioverter defibrillator to protect him. Which of the following is NOT a recognized cardiac complication of myotonic dystrophy?

A: Mitral valve prolapse B: Pulmonary stenosis C: Atrial flutter D: Ischaemic heart disease E: Diastolic heart failure. Comment : Endomyocardial biopsies frequently demonstrate non-specific changes such as fibrofatty infiltration, hypertrophy and myocarditis. All of these may lead to all of the above apart, from pulmonary stenosis. B : 96. A 68-year-old woman presents with 6 weeks of increasing breathlessness. She is in atrial fibrillation at rate of 120/min and has signs of mild/moderate cardiac failure. She gives no history of previous arrhythmia or to suggest ischaemic heart disease, but has had mild hypertension for at least 5 years. There are no cardiac murmurs. There are no ischaemic changes on the ECG. The chest radioagraph shows a normal sized heart and mild pulmonary oedema. She is not thyrotoxic. Select the best of these management options: A: Perform DC cardioversion within next 24 hours. B: Give sotalol with aim of achieving ‘chemical cardioversion’. C: Give diuretic and digoxin; perform DC cardioversion within next 24 hours. D: Give diuretic and digoxin; initiate anticoagulation; perform cardioversion in 6 weeks. E: Give diuretic; start full-dose intravenous heparin; perform DC cardioversion within next 24 hours. Comment : In a patient with a probable first episode of AF an attempt should be made to restore sinus rhythm. However, if AF has been present for >48 hrs – as seems very likely in this case – it is necessary to anticoagulate before cardioversion, unless it can be documented by trans-oesophageal echocardiography (TOE) that the left atrium is free of thrombus. If this were the case, then cardioversion within the next 24 hours would be the preferred management option, with warfarinisation for a month afterwards (which is a high risk period for thromboembolism). D : 97. A 50-year old man presents 3 months post mitral valve replacement (metallic) (MVR) with increasing shortness of breath, fever and weight loss. Clinically he is in pulmonary oedema. Transoesophageal echo (TOE) confirms severe paravalvular mitral regurgitation. The first blood culture is positive for Staph epidermidis. Which is the optimal therapeutic approach? A: Intravenous antibiotics for 4 weeks and then repeat TOE B: Oral antibiotics and angiotensin-converting enzyme (ACE) -inhibitor C: Re-do mitral valve replacement - bioprosthetic D: Re-do mitral valve replacement - metallic E: Withold antibiotics and repeat 3 sets of blood cultures.

32

MOHAMMED IS-HAG

Comment : Clinically this man has an infected MVR, with severe paravalvular leak. Following surgery, the commonest infecting organisms (up to around 9 months) are coagulase negative staphylococci. Antibiotics alone will not cure the infection; the valve must be replaced again. In this case this should be with a metallic valve since a bioprosthetic valve would be likely to need replacing after 10-15 years due to degeneration thereby subjecting him to a high-risk third operation. Bioprosthetic and metallic valves have similar risk for subsequent endocarditis. D : 98. A 28-year-old woman who abuses heroin intravenously presents with a high fever. She has aortic incompetence. Which of the following organisms is most likely to be the cause of her infective endocarditis? A: Staphylococcus aureus B: Streptococcus viridans C: An Enterococcus D: Staphylococcus albus E: A gram-negative bacillus Comment : In intravenous drug abusers Staph aureus accounts for 50% of cases of infective endocarditis, with other causes being enterococci (10%), Strep viridans (5%), gram negative bacteria (5%), other bacteria (5%), fungi (5%) and polymicrobial infection (5%). Coagulase-negative staphylococci account for less than 5%. By contrast, in patients who do not abuse drugs intravenously (and do not have prosthetic valves) the frequency of infection is as follows: Strep viridans 35%, enterococci 25%, Staph aureus 23%, culture negative 5-10%, other identified organisms 5-10%. A : 99. Which of the following is a strong indication for thrombolysis for a patient with a typical history for myocardial infarction presenting within 12 hours of onset of symptoms? A: S-T segment elevation of 1 mm in one lead only B: left bundle branch block C: elevated myoglobin without ECG changes D: second degree heart block E: S-T segment depression >2 mm in two adjacent chest leads Comment : The greatest benefit following thrombolytic therapy is seen in patients whose electrocardiogram (ECG) shows S-T segment elevation or bundle branch block. S-T segment elevation of >1 mm in two adjacent limb leads or >2 mm in two adjacent chest leads is significant. Although myoglobin concentration may be elevated within 2-3 hours of myocardial infarction there are no data on the benefits of thrombolysis in patients with elevated levels of this or other cardiac enzymes in the absence of ECG changes. Heart block may occur after myocardial infarction, but should not influence the decision on thrombolysis. If transvenous pacing is required following thrombolytic therapy, an internal jugular, femoral or brachial route of access is preferred to the subclavian route. B : 100.

PLATE 10 Which of the following does this ECG show? A: pacing for complete heart block B: temporary pacing in a patient with an anterior MI C: antitachycardia pacing D: VVI pacing E: DDD pacing. Comment : A temporary pacing system in the context of a myocardial infarction is usually for complete heart block. You would, therefore, see pacing spikes before each QRS complex rather than each P wave.

33

MOHAMMED IS-HAG

Temporary dual chamber systems are becoming increasingly used, however, in patients with reduced left ventricular function. VVI pacing is single chamber ventricular pacing with no sensing in the atrium. Antitachycardia pacing is seen in Implantable Cardioverter Defibrillators (ICDs) for the treatment of ventricular tachycardia. This is clearly not the case here. If the patient's underlying rhythm was complete heart block a pacing spike would be seen before each QRS complex. This patient has a dual chamber pacemaker in DDD mode. No P waves are sensed so there is a pacing spike before each P wave. However, each QRS is sensed and hence no paced QRS complexes are seen. A patient with a single chamber (AAI) pacemaker for sick sinus syndrome may have a similar ECG. E : 101. A 28-year-old lady is referred by her general practitioner with a history of recurrent syncopal episodes dating back to her early teens. She is not on any regular medication. Her GP has faxed through a 12-lead Electrocardiogram taken in his surgery shortly after her last blackout which shows polymorphic ventricular tachycardia. Which of the following statements are correct ? A: It is unlikely that she has a congenital long QT interval. B: Hypokalaemia is not a risk factor for polymorphic ventricular tacycardia. C: Female sex is not a risk factor for polymorphic ventricular tachycardia. D: Sotalol may cause torsade de pointes. E: The risk of torsade de pointe is a linear function of the QT interval. Comment : Although torsade de pointe can occur in many settings it is usually seen in patients with one of the congenital long QT syndromes or in association with drug therapy. Given that she is not on medication a congenital long QT syndrome needs to be considered. Some further points to be considered are: · Hypokalaemia increases the risk of torsade de pointe. · Female sex is a powerful predictor of the risk of torsade de pointes in patients with congenital and acquired long QT intervals. · Sotalol can prolong the QT interval and may cause torsade de pointe. · The risk of torsade de pointe is NOT a linear function of the QT interval. D : 102. A 48-year-old man is admitted with worsening angina. Which two of the following drugs are most likely to have precipitated his condition? A: Sildenafil B: Nicorandil C: Methotrexate D: Piroxicam E: Prazosin F: Sumatriptan G: Nifedipine H: Carbimazole I: Carbamazepine. Comment : Sildenafil (or viagra), a phosphodiesterase type 5 inhibitor, is associated with angina pectoris. Catastrophic hypotension, myocardial infarction and death have occured in patients taking viagra in conjunction with nitrates so this drug combination is absolutely contraindicated. Sumatriptan is a 5HT1 agonist used in acute migraine which causes vasospasm. It is contraindicated in patients with ischaemic heart disease as it can cause coronary vasospasm. Drugs which increase heart rate such as adrenergic drugs (salbutamol, isoprenaline) or rapid thyroid replacement, and drugs which cause a drop in blood pressure (e.g. misoprostil) may also worsen existing angina. A : F: 103. A 59-year-old man has an inferior myocardial infarct and is treated with streptokinase. Twenty-four hours later he suddenly becomes hypotensive. Which of the following diagnoses does NOT need to be considered? A: Mitral regurgitation B: Pulmonary embolus C: Ventricular septal defect D: Right ventricular infarction E: Hypotensive effect of streptokinase. Comment : Streptokinase can cause hypotension, but not after a delay of 24 hours. Diagnoses that require serious consideration are: · Arrhythmia · Re-infarction (of left or right ventricle) · Acute mitral regurgitation (due to dysfunction or rupture of papillary muscle) · Ventricular septal defect · Cardiac rupture with tamponade. Pulmonary embolism cannot be excluded, but is much less likely than any of the other three possible diagnoses listed. E :

34

MOHAMMED IS-HAG

104. A 60-year-old man presents to A&E with a 1.5 hour history of chest pain, a heart rate of 68 / min, blood pressure 140/85, and no signs of cardiac failure. An ECG shows ST elevation in leads V1-V3. Which of the following statements is correct? A: Treatment with a thrombolytic is preferable to primary coronary angioplasty. B: Streptokinase is the thrombolytic agent of choice. C: If the ST elevation resolves spontaneously aortic dissection should be considered. D: There is a role for intravenous beta blocker therapy. E: Intravenous heparin is mandatory whatever treatment strategy is used. Comment : In the early 1990s, controversy still existed about the use of angioplasty for the treatment of AMI. A meta-analysis in 1997 of 10 randomized trials comparing thrombolytic therapy with primary angioplasty (n=2606 patients) demonstrated that primary angioplasty was superior to thrombolytic therapy in terms of in-hospital mortality, non-fatal reinfarction, long-term survival and a lower incidence of stroke. The largest trial (PAMI-1) showed greatest benefit in high risk patients (cardiogenic shock, elderly patients, anterior wall myocardial infarctions). The under use of primary angioplasty in the UK is largely due to lack of resources, although most regard it as the optimal treatment strategy. GUSTO-1 demonstrated that Streptokinase was inferior to tPA when used as an accelerated 90-minute bolus infusion along with iv heparin. An analysis of pooled data from 28 trials showed an average 28% mortality reduction with use of beta blockers in acute myocardial infarction. The ISIS-1 trial first used intravenous atenolol (5mg) in AMI with a favourable mortality benefit, but this treatment is widely underused in current practice. D : 105. A 75-year-old gentleman with known peripheral vascular disease and intermittent claudication is unsuitable for surgical intervention. It has been decided to commence him on cilostazol. Which of the following is true concerning cilostazol? A: It is licensed for use in patients with rest pain. B: Its main action is to activate phosphodiesterases. C: It is safe in patients with previous haemorrhagic stroke. D: It is metabolised by cytochrome P450 3A4. E: It can be used as a treatment for other vascular diseases e.g. IHD, TIAs. Comment : Cilostazol is a potent inhibitor of platelet phosphodiesterases leading to inhibition of platelet aggregation. It also has vasodilating actions. It is currently licensed for the management of patients with intermittent claudication without rest pain and with no signs of tissue necrosis. Studies have shown that its principal hepatic metabolism is by the cytochrome P450 3A4 isoenzyme. Its use is contraindicated in patients with known bleeding tendencies (e.g. active peptic ulcer disease, previous haemorrhagic stroke in the last 6 months). Trials show an improvement in time to initial pain on walking and maximal walking distance when compared to placebo. D : 106.

PLATE11 A 57-year-old male had a pacemaker implanted 7 years ago following a number of syncopal episodes. He is otherwise well. This is his ECG recorded at a pacemaker check. What pacing mode might his device be programmed to? A: VVI B: DDDR C: OVO D: VOO E: AAIR F: VDO G: VVD H: IVI I: CDD J: VVIR.

35

MOHAMMED IS-HAG

Comment : His ECG clearly shows a pacing spike before each p wave. Therefore, there has to be pacing and sensing in the atrium, so the only possible pacing modes are AAIR or DDR. The former mode could be used for someone with sinus node abnormalities but intact atrioventricular conduction. There may be a pacing lead in the ventricle that is sensing the QRS complexes. On the basis of the ECG it is not possible to determine this as there are no paced QRS complexes. The first letter in the configuration denotes the paced chamber (0=none, A=atrium, V=ventricle, D=dual). The second letter represents the sensed chamber (0=none, A=atrium, V=ventricle, D=dual). The third letter represents the response to sensing (0=none, T=triggered, I=inhibited, D=dual). The final letter denotes programming modes, e.g. R=rate response. B : E : 107. A 17-year-old boy is brought to the doctor because his mother has noticed that his exercise capacity is decreasing and he ‘looks a funny colour’. She says that ‘he had a heart murmur when he was a baby’ and he is cyanosed and clubbed. The most likely diagnosis is: A: Tetralogy of Fallot B: Transposition of the Great Arteries C: Atrial septal defect D: Coarctation of the aorta E: Eisenmenger’s syndrome. Comment : Presentation with cyanosis of cardiac cause in adolescence is typical of Eisenmenger’s syndrome, the pathophysiology of which is as follows: a large congenital left-to-right shunt causes increased pulmonary blood flow, resulting in a rise in pulmonary vascular resistance and pulmonary hypertension. When the pulmonary vascular resistance exceeds the systemic vascular resistance the shunt is reversed, causing cyanosis. Virtually any large left-to-right shunt can lead to Eisenmenger’s syndrome, including atrial septal defect, ventricular septal defect, patent ductus arteriosus and Ebstein’s anomaly. Tetralogy of Fallot and Transposition of the Great Arteries present at birth or in infancy. E : 108. A 56-year-old man is admitted to hospital with crushing central chest pain. He is known to have had a myocardial infarction (MI) 3 months ago. Despite treatment with simvastatin in addition to other usual cardiac medication, his ECG shows that he has had a second MI. Which of the following statements is LEAST accurate? A: The overall effect of a drug depends on the net effect on polymorphisms within its metabolic pathways and genetic variation in target cells. B: There is an interaction between apo E genotype and lipid response to statin therapy. C: When a genetic variation affects more than 1% of the population it is termed a polymorphism. D: Lack of concordance with treatment is a possible explanation. E: Data from RCTs show that all MI patients should receive a statin. Comment : A, B and C are correct statements about pharmacogenetics, and D is also true. E looks sensible, but remember that RCTs show what happens in a group of patients in comparison with a group of controls. In any trial showing the survival advantage on a drug, some individuals will have a greater than average response, some less than average and some will be harmed. At present we cannot easily distinguish between these groups, so all patients are given a statin. E : 109. A 79-year old female with known aortic stenosis (aortic valve gradient 60 mmHg 2 years previously), presents with uncontrolled atrial fibrillation (140 bpm). She is dyspnoiec on exertion and on examination has an elevated jugular venous pressure and basal crepitations. Which two of the following are indicated as part of the initial treatment regimen?

A: Angiotensin-converting-enzyme inhibitors B: Amiodarone C: Beta-blockers D: Digoxin E: Flecainide F: Intravenous diuretics G: Intravenous nitrates H: Valvuloplasty I: Verapamil J: Warfarin. Comment : Negative inotropic drugs and arterial vasodilators should be avoided if possible in patients with significant aortic stenosis. The definitive treatment for symptomatic aortic stenosis is valve replacement. In adults, valvuloplasty has produced very disappointing results. If patients develop atrial fibrillation, attempts should be made to restore sinus rhythm and thereby maintain the atrial component to ventricular filling. Amiodarone has a much lesser negative inotropic effect than class I agents. Diuretics are required in patients with evidence of fluid overload, but caution must be exercised to avoid hypovolaemia and thereby reduce left ventricular diastolic filling pressures (and therefore cardiac output). Anticoagulation would be required as prophylaxis in view of the atrial fibrillation, but one should aim to obtain immediate therapeutic anticoagulation with heparin (or LMWH). B : F:

36

MOHAMMED IS-HAG

110.

A 57-year-old man presents with dyspnoea and orthopnea. He has an early diastolic murmur. See image for an illustration of a dilated aorta. Which of the following does not cause an aortopathy? A: Marfan's syndrome B: Ehlers Danlos syndrome C: Syphilis infection D: Atherosclerosis E: Duchenne's muscular dystrophy. Comment : The image is an aortogram demonstrating a dilated ascending thoracic aorta. This most commonly arises from the process of cystic medial degeneration from conditions such as Ehlers Danlo’s syndrome or Marfan’s syndrome. These aneurysms often involve the aortic root. Dilatation of the root causes aortic regurgitation. The treatment of choice is usually aortic root and valve replacement. Atherosclerosis is a common cause whilst syphilitic infection is becoming considerably less common. Duchenne’s muscular dystrophy is associated with a cardiomyopathy rather than a vasculitic process. E : 111.

A 35-year-old man presents with a history of collapse and transient loss of consciousness for a few seconds after running upstairs. He has a 10-year history of chest pains on exertion and presyncope. On examination he has an ejection systolic murmur at the left sternal edge with a normal aortic closure sound and no radiation to the carotids. His ECG demonstrated left ventricular hypertrophy with inferolateral ST segment/T wave changes. His echocardiogram is shown (see image). What is the most likely cause of his collapse? A: Bicuspid aortic valve

37

MOHAMMED IS-HAG

B: Atrioseptal defect C: Acute myocardial infarction D: Hypertrophic obstructive cardiomyopathy E: Coarctation of the aorta. Comment : 20-30% of patients with hypertrophic cardiomyopathy have dynamic left ventricular outflow tract obstruction caused by systolic contact between the anterior mitral valve leaflet and the interventricular septum. Clinically the presence of left ventricular hypertrophy and an ejection systolic murmur at the left sternal edge with a normal aortic closure and no radiation to the neck should suggest the diagnosis, particularly if associated with mitral regurgitation. The murmur is characteristically increased by manoeuvres that reduce afterload or preload. The M-mode echocardiogram demonstrates a parasternal long-axis view of the left ventricular with asymmetric septal hypertrophy and contact between the mitral leaflet and septum in systole. D : 112.

PLATE12 The first response to the cardiac rhythm shown should be: A: call the cardiac arrest team. B: defibrillate with 200J. C: confirm cardiac arrest. D: adrenaline (epinephrine) 1mg IV. E: start cardiopulmonary resuscitation. Comment : Look at the patient and apply the “lime jelly” test. If they are sitting up in bed eating their lime jelly, the chances are that the rhythm results from artefact (shivering, Parkinson’s) or that it comes from another patient. If they are face down in their lime jelly, it is likely that they have arrested! C : 113. A 77-year-old man presents with sudden onset of weakness of his right arm on a background of a 3-week history of thoracolumbar backpain, weight loss, fever and night sweats. Blood tests: full blood count (FBC) 9.8 g/dL, white cell count (WCC) 12.0, platelets 450, erythrocyte sedimentation rate (ESR) 110 mm/hr, creatinine 180 micromol/L and microscopic haematuria. What is the most likely diagnosis? A: Spinal osteomyelitis B: Myeloma C: Infective endocarditis D: Mycobacterium tuberculosis infection E: Tertiary syphilis Comment : Infective endocarditis is the only diagnosis that would explain all the symptoms and results. Vertebral osteomyelitis/discitis and stroke are both recognised complications of infective endocarditis and either can be the presenting feature of the disease. C : 114. A 48-year-old man of African origin presents with blood pressure 200/110mmHg. Urinalysis is negative and fundoscopy shows AV nipping. Which of the following treatment options is most appropriate? A : Oral atenolol 50mg od if hypertension confirmed over 1–2 weeks B : Oral enalapril 10mg bd if hypertension confirmed over 1–2 weeks C : Urgent admission for control of accelerated hypertension D : Oral nifedipine capsules 20mg tds if hypertension confirmed over 1–2 weeks E : Oral nifedipine LA 30mg od if hypertension confirmed over 1–2 weeks Comment :

38

MOHAMMED IS-HAG

The diagnosis of accelerated hypertension requires the finding of fundal haemorrhages and exudates, with or without papilloedema, as manifestations of fibrinoid necrosis. This patient has very high blood pressure but no evidence of accelerated hypertension.BTS guidelines suggest that the finding of blood pressure 200–219/110–119mmHg should be confirmed with repeat measurements after 1–2 weeks, then treated if still elevated.Black patients have low renin hypertension, therefore ACE inhibitors and beta-blockers as single agents do not lower blood pressure in this group.Calcium channel blockers and diuretics are effective agents.Nifedipine should be used as a long-acting preparation,not a short acting one.E :

115.

PLATE 13

This 63-year-old man had a short history of heart failure and was commenced on lisinopril and frusemide by his GP. He had a fatal cardiac arrest three weeks later. The post mortem showed a restrictive cardiomyopathy. The first image shows cardiac histology stained with 'haematoxylin and eosin', the second stained with a

specialized stain. Which of the following is the most likely diagnosis? A: Hypertension B: Hypertrophic obstructive cardiomyopathy C: Ischaemic heart disease D: Amyloidosis E: Haemochromatosis. Comment : The first image shows an amorphous eosinophilic substance within the specimen. The second image shows the classical apple green birefringence seen when the Congo red stain is viewed with polarised light. This is diagnostic of amyloid. Note the staining in the vessels as well as in the cardiac tissue. The man had no evidence of myeloma or lymphoma, but this was found to be an AL type of amyloid.The cardiac arrest was probably due to a dysrhythmia associated with amyloid rather than hypokalaemia. Coronary vessels were healthy for his age. D : 116. A 70-year-old man presents with shortness of breath that has developed over a few days. He has no other symptoms. One week ago he returned on a transatlantic flight from a holiday in Florida. Which two investigations are most likely to reveal the diagnosis? A: ECG B: Ventilation-Perfusion isotope scanning C: Echocardiogram D: Serum troponin I E: Chest radiograph F: Spiral computed tomography scanning G: Chest radiograph taken in expiration H: Spirometry I: Arterial blood gases J: Ultrasound scanning of the leg veins. Comment : The most likely diagnosis is pulmonary embolism, the two most specific tests for this being ventilation-perfusion isotope scanning and spiral computed tomography scanning.Ventilation scans are obtained using krypon-81m, technegas or xenon-133 and perfusion scans with intravenous 99m-Tc-labelled macroaggregates of albumin. Scans are reported as being normal or of low, medium or high probability, but it is very important to remember that reports must ALWAYS be interpreted in the light of the clinical context, and that a low probability scan does NOT mean that ‘pulmonary embolism is excluded’.Spiral computed tomography can detect intravascular clot from the pulmonary trunk down to the segmental arteries. It is the investigation of choice for patients with pre-existing lung disease, which renders the interpretation of ventilation-perfusion scans difficult or impossible. B : F:

39

MOHAMMED IS-HAG

117. A 39-year-old lady who who has a past history of treated hypertension is in her 3rd trimester of pregnancy and requires on-going anti-hypertensive treatment. Which antihypertensive would you definitely not prescribe? A : Hydralazine B : Labetalol C : Lisinopril D : Methyl-dopa E : Nifedipine. Comment : Evidence underpinning the choice of anti-hypertensive therapy in pregnancy is inadequate to make firm recommendations. There are no reports of serious effects with methyl-dopa following long and extensive use. Calcium antagonists, labetalol and hydralazine are commonly used, particularly for resistant hypertension in the third trimester. However, angiotensin-converting enzyme (ACE)-inhibitors should be avoided because they may cause oligohydramnios, renal failure and intra-uterine death. C : 118.

A 28-year-old woman was referred for the investigation of hypertension. Routine examination demonstrated that she had absent pulses in the left arm. She was a non-smoker. An arch aortogram was performed (see image). Which of the following statements is correct? A: She has coarctation of the aorta which is the likely cause of her hypertension. B: The selective picture shows a severe left subclavian stenosis most probably due to atherosclerosis C: The study is normal and she most likely has essential hypertension D: The selective picture shows a severe left subclavian stenosis most probably due to Takayasu's arteritis E: She has hypertension due to renovascular disease. Comment : Hypertension in the young always needs investigation. The physical findings in this woman were highly suggestive of a large vessel disease process.The selective angio picture does not show coarctation but does reveal critical stenosis of the left subclavian artery,which is most likely to be due to Takayasu's arteritis in this case.Atherosclerosis would be unusual in a young non-smoker.Hypertension is probably driven by renal ischaemia due to either direct renal vessel involvement or indirectly by aortic narrowing (not seen on these pictures). Raised erythrocyte sedimentation rate (ESR) is typical of the active inflammatory phase of this illness; in Japan there have been reports of linkage with tuberculosis steroids and other immunosuppressants can be effective. D : 119. A 20-year-old female student presents with central chest pain after four days of a ‘flu-like illness. The most likely diagnosis is: A: Acute viral pericarditis B: Gastro-oesophageal reflux C: Acute myocardial infarction D: Systemic lupus erythematosus E: Pulmonary embolism. Comment : The combination of a young patient and a ‘flu-like illness make acute viral pericarditis the most likely diagnosis in this case. The chest pain of pericarditis can be indistinguishable from that of myocardial infarction, excepting that sitting forward often eases it. The key physical sign to elicit would be a pericardial rub, and the key initial investigation would be the ECG, looking for widespread ST segment elevation, concave upwards. A : 120. Which of the following is not associated with cocaine abuse? A: Hypotension B: Contraction band necrosis C: Dilated cardiomyopathy D: Acceleration of atherosclerosis E: Aortic dissection.

40

MOHAMMED IS-HAG

Comment : Cocaine is associated with both acute and long-term cardiovascular disorders. Most commonly it is associated with myocardial ischaemia secondary to coronary spasm. Coronary spasm usually responds to nitrates, calcium antagonists or alpha blockers. Beta-blockers should be used with caution. A : 121. A 76-year-old lady is referred to the outpatient clinic with a 3-week history of new onset angina. In clinic she complains of chest pain at rest and is found on examination to have bilateral basal crepitations, a third heart sound and a pansystolic murmur. A 12 lead electrocardiogram shows ST depression of > 2 mm in her anterior leads. Which of the following are correct? A: There is a higher risk of death with unstable angina than an acute myocardial infarction (MI) B: Short episodes of chest pain are markers of high risk unstable angina C: The finding of congestive cardiac failure has no prognostic signifigance D: Only ST depression of 2mm or more is of prognostic signifigance E: A new or worsening MR murmur is a high risk factor for death from unstable angina. Comment : In patients with unstable angina the risk of death is lower than with an acute MI. Prolonged episodes of severe chest pain are important risk factors of high-risk unstable angina Any evidence on physical examination of acute congestive heart failure or a new or worsening MR murmur place the patient at increased risk of death. Dynamic shifts in ST segment depression or elevation of >1mm or T wave inversions that resolve when symptoms are relieved are of prognostic significance. E : 122. A 70 year old woman presents with 8 hours of chest pain. Her pulse rate is 40/minute and blood pressure 105/85. The ECG shows complete heart block, ST segment elevation and Q waves in leads II, III and AVF. Which of the following statements is correct? A: Atropine should be given immediately. B: An isoprenaline infusion should be set up immediately. C: Thrombolysis should be given immediately. D: Thrombolysis should be avoided because she has completed her myocardial infarction. E: Thrombolysis should be avoided because she may require a temporary pacing wire. Comment : The top priority is to achieve myocardial reperfusion. The presence of chest pain and ST segment elevation, despite Q waves, on ECG indicate that thrombolysis is needed immediately. C : 123. A 37-year-old man presents with recurrent episode of atrial fibrillation. He is having approximately three episodes each week, lasting for up to 5 hours. He has previously been treated with flecainide, sotalol and amiodarone. Which of the following is appropriate in his management? A: Consider digoxin B: Consider AV node ablation and pacemaker implantation C: Consider him for an implantable cardioverter defibrillator (ICD) D: Consider radiofrequency ablation to isolate his pulmonary veins E: None of the above. Comment : He is spending a significant proportion of his time in atrial fibrillation (high atrial fibrillation burden). However, it is paroxysmal and therefore digoxin is inappropriate and may increase the number of episodes.AV node ablation should not be considered in a young patient with paroxysmal symptoms. Generally this is reserved for patients in chronic atrial fibrillation resistant to drug treatment. An atrial defibrillator may be appropriate but certainly not an ICD used to treat malignant ventricular arrhythmias.Pulmonary vein isolation for atrial fibrillation is an evolving technique and may be appropriate for this patient. D : 124. In a patient with resistant hypertension, which of the following would increase the degree of clinical suspicion for significant renovascular disease as aetiology? A: Arterial bruits B: Co-morbidity with diabetes C: Concentric left ventricular hypertrophy D: Grade III hypertensive change on fundoscopy E: Palpable kidney. Comment : The highest degree of clinical suspicion of renovascular disease will occur in patients with arterial bruits, discrepancy in renal size of >1.5 cm (on ultrasound) and co-morbid vascular disease. If a patient has none of these then renovascular disease is highly unlikely, Evidence of end-organ damage is not helpful in differentiating aetiology. A : 125. A 60-year-old woman develops hypotension and a new systolic murmur 36 hours after being successfully thrombolysed for an anterior myocardial infarction. Which of the following statements is correct? Acute mitral incompetence due to rupture of the posterior papillary muscle is the most likely diagnosis. A: Acute mitral incompetence due to rupture of the anterior papillary muscle is the most likely diagnosis.. B: C: A basal ventricular septal defect (VSD) is the most likely diagnosis. D: An apical ventricular septal defect is the most likely diagnosis. E: The systolic murmur is likely to be due to mitral valve prolapse.

41

MOHAMMED IS-HAG

Comment : Anterior myocardial infarction is typically associated with apical VSDs whilst inferior myocardial infarctions are more commonly associated with basal VSD or posterior papillary muscle rupture. After confirmation of diagnosis by echocardiography or right heart catheter, which reveals a step up in oxygenation at ventricular level, urgent referral to a surgical centre is required, the outlook without surgical repair being extremely poor.Anterior myocardial infarction associated with apical VSD carries a better surgical outlook than inferior myocardial infarction associated with basal VSD. D : 126. A 72-year-old man on the coronary care unit has had a temporary transvenous pacing wire inserted. He suddenly becomes symptomatic with pre-syncope and a palpable pulse of 28 bpm. The ECG monitor shows pacing spikes that are not related to the QRS complexes. Which of the following actions is appropriate? A: Check that the pacemaker is switched on B: Check the pacemaker leads are connected C: Arrange for an urgent permanent pacemaker D: Start chest compressions E: Increase the pacing voltage to a maximum. Comment : If a patient with a temporary pacemaker becomes symptomatic it is important to check that the pacemaker is switched on and that the leads are still connected. The presence of pacemaker spikes on the ECG confirms that these are okay. It is likely that the tip of the pacing wire has migrated and that the threshold has increased. This may be overcome by increasing the voltage. If this is successful, the pacemaker wire will need to be repositioned. If inceasing the voltage is unsuccessful, external pacing should be substituted. E : 127. A 35-year-old lady with a long history of migraine presented with a mild right hemiparesis. She made a full recovery after 4 days but subsequent investigation revealed a patent foramen ovale (PFO). Paradoxical thromboembolism through a PFO is an important cause of stroke. Which of the following statements regarding PFO and closure of PFO is incorrect? A: Percutaneous closure of PFO is safe with a low incidence of recurrent neurological events. B: Divers with decompression sickness may warrant PFO closure. C: Spontaneous right to left shunting through a PFO indicates high risk. D: Closure of PFO in patients with recurrent migraine with aura should be considered. E: A three to five-fold higher prevalence of PFO is noted in patients with cryptogenic stroke. Comment : PFO occurs in approximately 20-30% of the population. Percutaneous closure of a PFO using a variety of closure devices is now an accepted practice in certain populations and in certain centres. All of the above are correct apart from D. A number of studies have suggested a correlation between migraine and PFO. However, there is currently insufficient evidence to support closure in these patients. D: 128. You find a middle aged man on a path in a park. He has no pulse and is not breathing. Which is the appropriate next step: A: Give two rescue breaths and initiate CPR at a ratio of 5 compressions to 2 breaths B: Give two rescue breaths and initiate CPR at a ratio of 15 compressions to 2 breaths C: Give a precordial thump D: Give two rescue breaths and go for help. E: Go to call 999 (emergency services) immediately. Comment : The guidelines on basic life support identify the importance of early access to defibrillation in cardiac arrest. They therefore suggest that no CPR is commenced until a call for emergency services has been made and the potential for early defibrillation is made possible. Once CPR begins the ratio of compressions to ventilations is 15 to 2. A precordial thump is not indicated in the unwitnessed collapse. E : 129. A 43-year-old woman with pulmonary hypertension attends clinic and asks to be prescribed bosentan. Which of the following is true? A: It is licensed for use in all patients with pulmonary hypertension. B: It inhibits the effects of endothelin-2 (ET-2). C: It binds to both endothelin A (ET-A) and endothelin B (ET-B) receptors. D: It is excreted primarily unchanged in the urine. E: It is not effective in patients with scleroderma. Comment : Bosentan is an antagonist of endothelin-1 binding to ET-A and ET-B receptors. It is licensed for the treatment of 'pulmonary arterial hypertension (PAH) to improve exercise capacity and symptoms in patients with grade III functional status'. It has been shown to be effective in patients with primary PAH and in those with PAH secondary to scleroderma. It is excreted in bile following metabolism by the cytochrome P450 enzymes and this is a potential source of interaction with drugs metabolised by the same isoenzyme (e.g. glibenclamide, ritonavir, ketoconazole, ciclosporin and itraconazole). C:

42

MOHAMMED IS-HAG

130.

PLATE 14 A 62-year-old man presents with chest pain. What two features does his ECG show (see image)? A: Posterior myocardial infarction B: Atrial fibrillation C: Anterior myocardial infarction D: Right axis deviation E: Unstable angina F: Ventricular ectopic beats G: Inferior myocardial infarction H: Atrial ectopic beats I: Left axis deviation J: Left bundle branch block. Comment : This ECG shows the classical appearances of acute myocardial infarction with ST segment elevation. This is seen in leads V1-V5, indicating that the infarct is anterior. The rhythm strip shows a number of ectopics, of which there is a compensatory pause, indicating that they are ventricular in origin.C:F: 131.

This patient presented acutely unwell with the following ECG (see image). What is the correct diagnosis? A: Hyperkalaemia B: Ventricular tachycardia C: Acute pericarditis D: Acute anterolateral myocardial infarction E: Digoxin toxicity.

43

MOHAMMED IS-HAG

Comment : The presence of tall, peaked T waves, flattened P waves, prolonged PR interval and wide QRS complexes are pathognomonic of hyperkalaemia. Give IV calcium immediately (10mls, 10% calcium gluconate) and call the renal team. A: 132.

A 30-year-old male with a heart murmur and breathlessness on effort is referred for transthoracic echocardiogram (see image). Which of the following is true of his condition? A: Sudden death is very common. B: Exercise testing is contraindicated. C: Atenolol may help his symptoms. D: Frusemide will help his shortness of breath. E: The ECG is always abnormal. F: It is very rare in elderly patients over 75 years of age. G: The condition can be confidently diagnosed in patients over 10 years pf age. H: Ischaemia may occur with a normal coronary arteriogram. I: Endocarditis prophylaxis is not required. J: Prophylactic amioderone is mandatory to prevent sudden death. Comment : The above patient has hypertrophic cardiomyopathy with marked asymmetrical septal hypertrophy and a resting outflow tract gradient of 55mmHg.The ECG may be normal in 25% cases and morphological expression of the disease may not be completed until the end of puberty.Twenty -five per cent of all cases occur in those over 75.Sudden death occurs in 1% cases and those considered high risk should be referred for an implantable defibrillator.Exercise testing is extremely useful at identifying those patients with dynamic outflow tract gradients who would benefit from more aggressive therapy.Surgical myectomy or percutaneous alcohol septal ablation is reserved for those with high (> 50mmHg) resting gradients with refractory symptoms despite optimal medical treatment. C:H: 133. A 72-year-old Caucasian woman in your outpatient clinic has uncomplicated essential hypertension. Her blood pressure is 162/102mmHg despite optimization of non-pharmacological therapy. Which one of the following treatments would you choose as the first-line agent for her? A: Atenolol 50 od B: Bendrofluazide 2.5mg od C: Bendrofluazide 5mg od D: Enalapril 5mg od E: Ramipril 2.5mg od. Comment : Hypertension is particularly common in those aged above 60 not least because of the steady rise in systolic blood pressure with age. These patients are at a high absolute risk of cardiovascular complications. Furthermore, anti-hypertensive treatment may also reduce incidence of heart failure and possibly dementia. Non-pharmacological therapy for hypertension should always be optimised prior to commencement of medication, whenever possible. Low-dose diuretics are accepted as the first-line treatment for hypertension in the elderly and appear to confer greater benefit than Beta-adrenergic receptor antagonists in this subgroup. 1. Treatment of isolated systolic hypertension in the elderly with the longacting calcium channel blocker, nitrendipine, has been shown to reduce stroke and cardiovascular outcome. Therefore, calcium channel blockers may be suitable when diurectics are not tolerated, ineffective or contra-indicated.2 B:

44

MOHAMMED IS-HAG

134.

A 35-year-old man is admitted with chest pain. His ECG and cardiac enzymes are normal. Because a murmur is heard he has a transesophageal echocardiography (TOE) (see image). Which of the following is true concerning the disorder shown? AoA is ascending aorta, AoD is descending aorta.

A: Medical treatment is the best long term option. B: The most likely cause of the murmur is mitral regurgitation. C: Methyldopa is contraindicated in the initial management. D: The diagnosis can usually be made as easily with transthoracic echo. E: CT scan of the chest is usually superior to TOE in making the diagnosis. F: Pregnancy is protective from condition. G: Atherosclerosis is the most likely underlying cause of the condition in this case. H: Untreated overall prognosis is good. I: Paraplegia is a recognized complication. J: Persistent abdominal pain may be a worrying feature. Comment : The TOE images demonstrate a Type A aortic dissection with an intimal flap seen in the ascending and descending aorta. The most likely cause for his murmur is aortic regurgitation or coexistent bicuspid aortic valve. Untreated surgically, mortality is high so early referral to a cardiothoracic unit is crucial. Type B dissections involving the descending aorta only are usually treated medically in the first instance as the complications of surgery are high.TOE,MRI and CTchest have similar sensitivities and specificities in making the diagnosis.Methyldopa may be used in the initial management to control blood pressure. When the descending aorta is involved persistent abdominal pain may indicate involvement of the mesenteric arteries. Pregnancy is a rare but recognized cause. In young patients atherosclerotic disease is rarely the cause. I:J: 135.

PLATE 15 A 35-year-old man is referred for a transthoracic echocardiogram (see image) after a murmur is heard by his GP. Which of the following physical signs would be expected.

45

MOHAMMED IS-HAG

A: An immediate diastolic murmur B: A machinery murmur C: An ejection click D: A loud first heart sound E: A late systolic murmur F: A fourth heart sound G: Pulsus alternans H: A wide pulse pressure I: Paradoxical splitting of the second heart sound J: Kussmaul's sign Comment : This patient has mitral valve prolapse (MVP) of the posterior leaflet with mitral regurgitation. Typical findings include a mid systolic click followed by a late apical systolic murmur. When the mitral regurgitation becomes severe the murmur can become pansystolic with a third heart sound and a displaced hyperdynamic apex. The first heart sound is usually soft. Atrial fibrillation may also occur.C:E: 136.

This is a M-Mode trace from a transthoracic echocardiogram (see image) taken on a 60-year-old man presenting with shortness of breath for 6 months. A subsequent angiogram showed normal coronary arteries. He is in sinus rhythm. Which of the following treatments will improve his prognosis long term? A: Atenolol B: Carvedilol C: Pindolol D: Digoxin E: Dobutamine F: Milrinone G: Frusemide H: Bendrofluazide I: Spironolactone J: Amlodipine. Comment : The M-mode trace demonstrates a dilated cardiomyopathy with significantly reduced ejection fraction. There is also a small pericardial effusion seen below the posterior wall of the left ventricle. Carvedilol, bisoprolol, bucindolol and metoprolol are the only beta-blockers thus far which improve long-term prognosis. Inotropes long term are associated with a worse prognosis.Digoxin has little impact on mortality but may improve quality of life and reduce hospital readmission rates in end stage disease.Amlodipine has no impact on mortality.B:I: 137. A 78-year-old man with known ischaemic heart disease and congestive cardiac failure is admitted as an emergency with severe pulmonary oedema. He is sitting with his legs over the side of the casualty trolley and gasping. He has smoked 20 cigarettes per day for many years. What is the first treatment he should receive? A: Frusemide 40 mg intravenously B: High flow oxygen via reservoir bag C: 35% oxygen D: Diamorphine 2.5 mg intravenously, with anti-emetic E: Isosorbide dinitrate by intravenous infusion at dose titrated against blood pressure.

46

MOHAMMED IS-HAG

Comment : Unless it is definitely known that a patient has a tendency to retain carbon dioxide, all patients with severe breathlessness should be given high flow oxygen via a reservoir bag once it has been established that their airway is clear.After the patient has been started on high flow oxygen, give frusemide 40-80 mg IV and diamorphine 2.5-5 mg IV. If matters do not improve consider isosorbide dinitrate 2-10 mg/hr IV. If matters worsen, then call the ICU sooner rather than later (assuming that the man’s condition prior to this acute presentation means that this is appropriate).Concurrently try to establish a cause for the acute deterioration: has he had a myocardial infarction? If so, would he benefit from thrombolysis? B: 138. A 22-year-old man presents to A&E claiming to have taken a large quantity of paracetamol 24 hours previously. He washed the tablets down with vodka. Which of the following statements are correct? A: Measuring paracetamol levels at 24 hours is of no use. B: N-acetyl cysteine should be witheld until plasma paracetamol levels are known. C: The prognostic accuracy of the treatment nomogram is less certain at 24 hours post-ingestion. D: Methionine can be given if the patient is intolerant of N-acetyl cysteine (NAC). E: Clinical symptoms may occur > 24 hours post-ingestion. F: If the urea and electrolytes are normal, the patient can be sent home. G: Even at 24 hours, activated charcoal should be given as gastric transit time is prolonged by paracetamol. H: The measurement of alanine transferase is of prognostic value. I: If he drank more than two units of alcohol with the overdose, he is at greater risk. J: If he is obese, he is at greater risk from the effects of the overdose. Comment : Levels should always be measured. If paracetamol is still present it may suggest that the overdose was taken over a period of time making interpretation of the nomogram very difficult. If a clinically significant quantity of paracetamol has been taken, start NAC whilst waiting for the results - it can always be stopped (in most hospitals it takes a significant amount of time for blood tests to be processed and the results retrieved). Most reactions to NAC are mild and can be overcome by giving it more slowly. For patients who are truly intolerant, methionine can be given up to 12 hours after ingestion. Clotting, liver function and U&E should all be checked before consideration is made as to whether the patient could go home. The best prognostic indicator is the INR. Patients who are anorexic for any reason, who drink significantly (>21 units/wk for men, >14 units/wk for women) or who are on enzyme-inducing drugs are at greater risk from paracetamol overdose. C:E: 139. A 72-year-old female patient has been admitted acutely to the ward via the Emergency Department. She was found collapsed at home, semiconscious. Her Glasgow coma score (GCS) is currently 10 and she is receiving oxygen via a venturi mask. You notice that she has stridor with marked inspiratory effort. Which of the following statements regarding basic airway management are correct? A: B: C: D: E: F: G: H: I: J:

The length of an oropharyngeal airway should correspond to the length of the patient's little finger. Oropharyngeal airways are contraindicated in patients with a base of skull fracture. Patients with preserved laryngeal reflexes will not tolerate a nasopharyngeal tube. Insertion of an oropharyngeal airway can trigger laryngospasm. A nasopharyngeal airway cannot be used if the nose is fractured. Both nasopharyngeal and oropharyngeal airways can provide a definitive airway for comatose patients. Nasopharyngeal and oropharyngeal airways should be used when a jaw thrust is contraindicated. The diameter of a nasopharyngeal tube should be similar to the diameter of the patient's little finger. A head-tilt chin lift produces less neck movement than a jaw thrust manoeuvre. Vomitus should be removed from the airway using a blind finger-sweep.

Comment : Basic airway management involves initially opening the airway. This should be done with a head-tilt chin lift unless there is suggestion of a neck injury when a jaw-thrust manoeuvre is preferred. Material in the oropharynx should be removed under direct vision.Oropharyngeal (OPA) and nasopharyngeal (NPA) airways are useful adjuncts but do not provide a definitive airway for unconscious patients. A quick way to size a NPA is to choose one with an external diameter similar to the patient’s little finger. NPAs are contraindicated in base of skull fractures. If there is a nasal fracture, the most patent nostril should be chosen. NPAs are generally better tolerated than OPAs and can be used in patients where the laryngeal reflexes are preserved.OPAs should be sized with the length corresponding to the distance from the angle of the patient's mouth to the angle of the jaw. Inserting OPAs can trigger vomiting and laryngospasm.D:H:

47

MOHAMMED IS-HAG

140. A 67-year-old man is on coronary care following an inferior myocardial infarct. He has had a temporary transvenous pacemaker inserted for Mobitz type II heart block. He suddenly becomes light-headed with chest pain. The cardiac monitor shows complete heart block with a rate of 20 bpm. What should your first two actions be? A: Call the cardiac arrest team B: Ask him to cough repeatedly C: Give a precordial thump D: Check the lead connections on the pacemaker E: Institute external pacing F: Obtain a 12 lead electrocardiogram G: Increase the pacemaker voltage to maximum H: Start an isoprenaline infusion I: Give streptokinase 1.5 MU intravenously J: Give morphine 5-10 mg intravenously. Comment : The patient's symptoms result from his bradycardia. The pacemaker is failing to capture. The first thing to do is to check the leads. The pacemaker voltage should then be increased to maximum which may enable it to capture. The threshold for pacing increases with time and should be checked at least daily. If capture occurs, arrangements should be made to reposition the pacemaker lead. If it does not, external pacing should be instituted.Cough and percussion pacing are emergency measures that occasionally work.D:G: 141. Isoprenaline is rarely found even on coronary care units nowadays. A 48-year-old man presents with 40 minutes of cardiac chest pain. Which two of the following ECG criteria are indications for thrombolysis? A: 2 mm or more elevation of the ST segment in any one standard ECG lead B: Atrial fibrillation C: 2 mm or more ST segment depression in two or more standard ECG leads D: 2 mm or more elevation of ST segments in two or more contiguous praecordial leads E: New right bundle branch block F: Ventricular tachycardia G: 1 mm or more elevation of ST segments in two or more standard ECG leads H: 2 mm or more elevation of ST segments in any two ECG leads I: 1 mm or more elevation of ST segments in any two ECG leads J: 1 mm or more elevation of ST segments in two or more contiguous praecordial leads. Comment : The other recognised ECG criterion for thrombolysis is new left bundle branch block. D:G: 142. A 46-year-old female has been referred by her GP with a two-year history of lethargy, an episode of atrial fibrillation and a murmur. On examination she is in sinus rhythm, normotensive with normal first heart sound. A transthoracic echo has demonstrated a significant left to right shunt. Which of the following is the most likely diagnosis? A: Patent ducutus arteriosus B: Patent foramen ovale C: Ostium secundum atrial septal defect D: Ostium primum atrial septal defect E: Ventricular septal defect. Comment : Secundum atrial septal defects are often not identified in childhood and can present with non specific symptoms such as lethargy. The first heart sound is often split but equally may be normal. The definitive investigation is transthoracic echocardiography. This will demonstrate first of all the anatomy and secondly using doppler indicate any significant shunting.Ostium secundum defects are more midseptal and by far more common than primum defects which are more inferiorly displaced towards the AV valve. Primum defects are often associated with anomalies of the mitral and/or tricuspid valve, and as such patients more commonly present in childhood.It is clear from the clinical picture that this is not a patent ductus arteriosus. Patent foramen ovale (PFO) is a common condition and generally benign. There may be some left to right flow on doppler imaging but this will not be significant. The harsh pansystolic murmur of a ventricular septal defect is usually obvious unless the defect is very large. In this case the patient would be likely to have more pronounced symptoms.c: 143. A patient with documented systolic dysfunction has permanent atrial fibrillation. His resting heart rate is 100 and systolic blood pressure 120 mmHg; there is no evidence of fluid retention. Creatinine is normal and he is already receiving appropriate doses of frusemide and angiotensin-converting-enzyme inhibitor. Which of the following would be the next therapy? A: Amiodarone B: Carvidolol C: DC cardioversion D: Digoxin E: Metoprolol.

48

MOHAMMED IS-HAG

Comment : Beta-blockers have prognostic and symptomatic benefit in heart failure. In the UK only carvedilol and bisoprolol are licensed for this use.Digoxin can improve symptoms in severe heart failure. Whilst it may well slow resting heart rate in atrial fibrillation it has less benefit on exercise related increases in rate. B: 144. You see a patient with poorly-controlled atrial fibrillation and decide to start him on amiodarone. There is a potentially serious drug interaction whereby amiodarone increases the plasma concentrations of the following two drugs: A: Aspirin B: Verapamil C: Ramipril D: Nifedipine E: Ranitidine F: Bendrofluazide G: Digoxin H: Pravastatin I: Glibenclamide J: Spironolactone. Comment : Amiodarone inhibits CYP 3A4 and will increase the plasma concentrations of drugs such as ciclosporin, phenytoin, verapamil and warfarin. This may cause bradycardia, AV block and myocardial depression if verapamil and amiodarone are used together in treating atrial fibrillation, thus great care is needed in adjusting the dose. There is a similar interaction between digoxin and amiodarone, and it is recommended that the digoxin dose be halved in those taking both agents.B:G: 145. This patient has permanent atrial fibrillation and as such DC cardioversion will not, by definition, restore sinus rhythm. The following results were found at cardiac catheterization in a 45-year-old man who has been complaining of increasing breathlessness. What is the diagnosis? RA

Pressure(mmHg)

Saturations (%)

15

67

RV

40/7

67

PA

43/16

68

PCWP (mean)

26



110/8

96

Ao

107/68

96

A: Aortic stenosis B: Ventricular septal defect (VSD) C: Aortic Regurgitation D: Mitral stenosis E: Pulmonary stenosis. Comment : There is no significant pressure gradient across the aortic or pulmonary valves thereby excluding aortic or pulmonary stenosis. There is no change in saturation from RA to RV, thereby excluding the diagnosis of a VSD with left to right shunting. We can, however, see the increase in mean capillary wedge pressure (providing a reflection of left atrial pressure) which is greater than left ventricular diastolic pressure (8 mmHg) in keeping with mitral stenosis. There is also an associated mild/moderate elevation in pulmonary pressures.D: 146. The degree of obstruction often progresses after birth and therefore children may present at a later date (although usually within the first two years). Which of the following statements about the unwanted effects of amiodarone is NOT true? A: Most patients develop corneal microdeposits within 1 month of commencing treatment. B: Thyroid dysfunction is more common in women, older patients and those who already have thyroid problems. C: High doses of amiodarone are associated with tremor, ataxia and peripheral neuropathy. D: Patients should be monitored for early signs of pulmonary toxicity, as this is reversible if the amiodarone is discontinued early enough. E: Amiodarone is often implicated in torsade de pointes. Comment : Amiodarone is useful in maintaining sinus rhythm in patients with paroxysmal atrial fibrillation, or who have remained in AF after cardioversion. It may also be used to prevent ventricular tachycardia. However, long-term use is associated with several unwanted effects. Some of these can be reduced by using a lower dose i.e. 100-200mg daily,for example the neurological effects such as tremor, ataxia and peripheral neuropathy are less common at lower doses. Others, such as early pulmonary toxicity and skin discoloration, are reversible if the drug is stopped, but remember that it has a very long half life. Amiodarone itself is not a frequent cause of torsade de pointes, unless it is coprescribed with other drugs which also prolong the QT interval, e.g. sotalol, quinidine and disopyramide. E:

49

MOHAMMED IS-HAG

147. A 28-year-old woman is admitted with left sided pleuritic chest pain that began suddenly six hours ago. She is very anxious, with a respiratory rate of 18/min and a pulse rate of 90/min. The following would reassure you that she has not had a pulmonary embolus: A: Entirely normal physical examination (aside from RR 18/min and PR 90/min) B: None of the other answers is correct C: Oxygen saturation of 99% on air D: Slight tenderness elicited by pressing on the left side of the chest E: Her admission that she is anxious because a friend of hers had a pulmonary embolus recently. Comment : The common causes of pleuritic chest pain are pulmonary embolism, pneumonia and musculoskeletal pain. The safe and correct management is to assume pulmonary embolism unless the history or examination findings enable another positive diagnosis to be made, and this woman has had a pulmonary embolus until proven otherwise.B: 148. A history of local trauma or definite localised tenderness may enable a confident diagnosis of musculoskeletal pain to be made, but slight tenderness is not uncommon in pulmonary embolism, when examination can be normal, as can oxygen saturation. Which of the following statements is true concerning pregnancy and cardiovascular disease? A: Severe mitral stenosis is usually well tolerated. B: In valve disease antibiotic prophylaxis is recommended for a normal vaginal delivery. C: Caesarean section is recommended with hypertrophic cardiomyopathy. D: Epidural anaesthesia is recommended with hypertrophic cardiomyopathy. E: Peripartum cardiomyopathy is usually seen after delivery. Comment : Severe mitral stenosis is often poorly tolerated in pregnancy and in suitable symptomatic patients balloon valvuloplasty can be performed with minimal radiation. In those with valvular disease antibiotic prophylaxis is recommended for caesarean section or instrumented deliveries only. E: 149. Patients with hypertrophic cardiomyopathy can usually have a safe vaginal delivery and caesarean section should only be offered on obstetric grounds. Epidural anaesthesia should be avoided in these women due to poorly tolerated vasodilation. In a study of patients with myocardial infarction, the death rate of those given aspirin is 8%, compared with 10% in those not given aspirin. This means that: A: the relative risk of death after myocardial infarction is 1.25 in those given aspirin B: the relative risk reduction produced by aspirin is 2% C: the number needed to treat with aspirin to prevent one death is 8 D: the number needed to treat with aspirin to prevent one death is 10 E: the absolute risk reduction produced by aspirin is 2%. Comment : Absolute risk reduction (or increase) = (Risk in group 1) minus (Risk in group 2), which is 2% in this example.Relative risk is the difference of outcome in one group compared to another = (Risk in group 1) divided by (Risk in group 2). In this case aspirin reduced relative risk by 20%.E: 150. The Number Needed to Treat = 1 divided by (Absolute Risk Reduction), which is 1/0.02 or 50 in this example. When considering patients with Tetralogy of Fallot, which of the following is the most important determinant of the severity of clinical presentation? A: Size of the ventricular septal defect B: Degree of right ventricular outflow tract obstruction C: Size of the right atrium D: Severity of right ventricular hypertrophy E: The presence of an over-riding aorta. Comment : In Tetralogy of Fallot (right ventricular outflow tract obstruction, ventricular septal defect, right ventriuclar hypertrophy, over-riding aorta) the severity of clincial presentation is dependent upon the degree of obstruction to the right ventricular outflow tract. This is often dynamic (muscular bulge below the pulmonary valve) and will ultimately determine the degree of right to left shunting (and hence cyanosis etc.). B: 151. Which of the following is not true of diastolic heart failure? A: It affects a younger age group B: Reversal of E:A ratio is demonstrated on transthoracic echocardiography. C: Presence of fourth heart sound D: Ejection fraction of >40% E: It is associated with obesity. Comment : Diastolic impairment is estimated to occur in 20-50% of patients with heart failure. Symptoms of breathlessness are secondary to impaired filling of the LV, resulting in high filling pressures. This in turn results in pulmonary congestion, dyspnoea and oedema. Management of diastolic heart failure is aimed at reducing pulmonary congestion. There is much less evidence for the benefit of therapeutic interventions from clinical trials than for patients with left ventricular systolic dysfunction.A:

50

MOHAMMED IS-HAG

152. Which of the following is commonly associated with cyanosis in the infant? A: Atrial septal defect B: Co-arctation C: Patent ductus arteriosus D: Transposition of the great arteries E: Ventricular septal defect. Comment : Integral to cyanotic congenital heart disease is right to left shunting of blood in the heart. Transposition of the great arteries is the commonest cardiac cause of cyanosis at birth.It is incompatible with sustained life unless there is an associated atrial or ventricular septal defect or patent ductus arteriosus. Atrial septal defects, patent ductus arteriosus and ventricular septal defects may all result in cyanosis in the form of the Eisenmenger syndrome; left to right shunting results in pulmonary hypertension with eventual reversal of shunt. D: 153. Which of the following conditions is not associated with a rise in serum cardiac troponins: A: myocardial infarction B: pulmonary embolus C: renal failure D: liver failure E: intracerebral bleed. Comment : Raised serum cardiac troponins are sensitive markers of cardiac necrosis secondary to any primary cause. Therefore, raised levels can be seen in acute pulmonary embolus (due to right ventricular infarction), myocarditis, and any intracerebral event (due to focal myocardial necrosis). Up to 40% of all patients with renal failure have raised serum troponins. The reasons and significance of this remain unclear. In acute coronary syndromes the presence of a raised troponin suggests a high risk group who would benefit from inpatient treatment. D: 154. A 50-year-old woman is seen in cardiology clinic with progressive breathlessness and ankle swelling. Auscultation of her heart reveals a fixed split second heart sound. Which of the following is true?

A: The most likely diagnosis is an ostium primum atrial septal defect. B: The most likely diagnosis is a sinus venosus defect. C: The most likely diagnosis is mitral stenosis. D: She is likely to have pulmonary vascular disease. E: Surgery is indicated in the majority of cases. Comment : The most likely diagnosis is an ostium secundum atrial septal defect (ASD). Eighty-five per cent of ASDs are of the secundum type, 11% of the primum type and 4% sinus venosus defects involving the inferior vena cava (IVC) or superior vena cava (SVC). Clinical signs are rarely present. When people present in middle age with symptoms this usually relates to the development of progressive pulmonary

vascular disease, pulmonary arterial hypertrophy and a change in the direction of flow across the shunt (left to right changes to right to left).Many ASDs are now being closed using percutaneous closure devices. D:

155. A 38-year-old woman who was previously fit and well presents with breathlessness that has been getting worse over 3 or 4 months. You suspect that she might have primary pulmonary hypertension. Which of the following physical signs would NOT be consistent with this diagnosis? A: Elevated jugular venous pressure B: Clubbing C: Palpable heave at left sternal edge D: Added heart sound over right ventricle E: Loud P2. Physical signs include elevated JVP (sometimes with a giant v wave of tricuspid regurgitation and pulsatile hepatomegaly), left parasternal heave, pansystolic murmur (tricuspid regurgitation) and right ventricular S4, peripheral oedema. Clubbing is not a feature and suggests chronic lung disease or cyanotic congenital heart disease in this context.An important differential diagnosis of primary pulmonary hypertension is chronic pulmonary thromboembolism. B: 156. Which of the following statements is correct regarding the neurohormonal response to stimulation of cardiovascular mechanoreceptors? A: non-osmotic release of vasopressin occurs as a result of increased atrial stretch B: a reduction in blood pressure leads to stimulation of high-pressure mechanoreceptors in the aortic arch and subsequent inhibition of renal sympathetic activity C: a reduction in peripheral vascular resistance is sensed by the high-pressure mechanoreceptors and results in stimulation of the renin-angiotensin-aldosterone system D: atrial natriuretic peptide (ANP) is released in response to stimulation of receptors in the carotid sinus and results in natriuresis and diuresis E: endothelin-1 is a potent vasodilator, released in response to elevated blood pressure Comment : Two distinct categories of cardiovascular mechanoreceptors exist: high-pressure and lowpressure. High-pressure receptors are located in the left ventricle, carotid sinus, aortic arch and renal juxtaglomerular apparatus. These receptors respond to decreases in arterial pressure, peripheral vascular resistance or renal perfusion by stimulating reflexes that result in the activation of the

51

MOHAMMED IS-HAG

sympathetic and the renin-angiotensin-aldosterone systems and the non-osmotic release of vasopressin. The major net effects are vasoconstriction and the retention of sodium and water. Low-pressure receptors are primarily found within the atria. An increase in atrial stretch results in enhanced secretion of ANP with subsequent vasodilatation and both natriuresis and diuresis. Endothelin-1 is a potent vasoconstrictor. The interactions between these systems are relatively complex in both health and diseased states. For example ANP, angiotensin II and the renal sympathetic nerves are able to modulate the renal juxtaglomerular release of renin.C: 157. You see a 48-year-old Afro-Caribbean man in the outpatient clinic with uncomplicated essential hypertension. His blood pressure today is 154/102mmHg despite optimization of non-pharmacological therapy. Which one of the following drugs would you use as the first-line agent in this patient?

A: Atenolol 50mg od B: Nifedipine 10mg tds C: Amlodipine 5mg od D: Ramipril 2.5mg od E: Enalapril 5mg bd. Comment : Non-pharmacological therapy should always be optimized prior to commencement of medication, whenever possible. Hypertension is particularly common in Afro-Caribbeans and associated with particularly higher risk of complications. Therefore effective long-term treatment, with a low threshold for multiple therapy where necessary, is particularly important. Studies indicate that drugs such as ACE (angiotensin-converting enzyme) inhibitors and Beta-receptor antagonists are less effective in Afro-Caribbeans. The reason appears to be related to the finding that the renin-angiotensinaldosterone (RAA) system is commonly suppressed in the majority of Afro-Caribbeans. As such, drugs that suppress the RAA system are less likely to be effective. Calcium-channel blockers (CCBs) and diuretics appear to be more effective in this subgroup. However, diuretics may not be suitable in this case as they are commonly associated with impotence. Short-acting CCBs do not provide prolonged BP control, can cause reflex tachycardia and may be associated with higher mortality. Therefore, long-acting CCB should be the first-line drug of choice.Ideally, a once-daily agent with that provides a smooth 24-hour BP control (e.g. Nifedipine LA 30mg od or Amlodipine 5 mg od) to improve compliance would be preferable. C:

158. In acute aortic dissection: A: surgery should only be considered for type A dissection when the aortic valve is involved B: opiates are best avoided due to the risk of bradycardia C: type B dissections commonly begin just prior to the origin of the left subclavian artery D: intravenous beta-blockers should be used for acute control of blood pressure E: calcium channel blockers should be avoided. Comment : The classification of aortic dissection depends upon the site of origin of the dissection flap. Type-A is more common and involves the ascending aorta, whereas type-B begins distal to the ascending aorta, usually just after the origin of the left subclavian artery. Type-B dissections are generally treated medically with strict blood pressure control. Early surgical intervention is indicated in the case of Type A dissection; tight blood pressure control is also required. Adequate analgesia, thereby decreasing sympathetic stimulation, is imperative in all cases and usually requires opiates.To achieve tight blood pressure control an intravenous agent with short half-life is generally used. The therapy of choice is generally considered to be a beta-blocker such as proanolol or labetolol (alpha and betareceptor blocker). Beta-blockers have additional beneficial effects by reducing arterial wall stress (reducing rate of change of arterial wall pressure). D: 159. A previously fit young woman presents with pleuritic chest pain 48 hours after a transatlantic aeroplane flight. Her chest radiograph is clear. The correct course of action is: A: Order a lung ventilation / perfusion scan. Give low molecular weight heparin if this is positive B: Reassure. Give non-steroidal anti-inflammatory agent for pain control C: Give oral amoxicillin and clarithromycin D: Give low molecular weight heparin. Order a lung ventilation / perfusion scan E: Order a chest radiograph in expiration to exclude pneumothorax. Comment : This woman has had a pulmonary embolus until proven otherwise. It is correct to start treatment immediately when the index of suspicion is very high and the risks of treatment (anticoagulation) are low, as in this case. D: 160.

Which of the following is true concerning Group 1 entitlement to drive with cardiovascular disease:

A: Patients with angina cannot drive B: After myocardial infarction patients can drive after one week C: Patients with syncope cannot drive until the underlying cause has been found and treated D: After implantation of a pacemaker patients cannot drive for one month E: After angioplasty patients cannot drive for one month Comment : Group 1 entitlement refers to normal driving. In this situation patients can drive one week after angioplasty or permanent pacing. After myocardial infarction patients must not drive for one month. Those with stable angina may drive if their symptoms dont occur at rest or at the wheel. The laws concerning Group 2 entitlement are quite different (see module reference).C:

52

MOHAMMED IS-HAG

1. A 26-year-old man is admitted to hospital with a history of dyspnoea of 4-6 weeks duration. He initially had flu-like symptoms and was treated by his doctor with a 10-day course of ciprofloxacin. However, he then started coughing up blood, leading to urgent referral. On examination he was dyspnoeic at rest, with bilateral crackles on auscultation of the lungs. Investigation revealed anaemia and impaired renal function (creatinine 220 micromol/l). Pulmonary function tests were normal apart from an abnormally high diffusion factor. Urine dipstick testing showed the presence of red blood cells. What is the most likely diagnosis? A: Lymphangiomyomatosis B: Goodpasture's syndrome C: Chronic eosinophilic pneumonia D: Bronchiolitis obliterans E: Invasive Aspergillosis. This patient has both lung and kidney involvement typical of a 'pulmonary renal syndrome'.Goodpasture described the association of pulmonary haemorrhage with renal failure(Goodpasture's syndrome)in1919, and the 'classic' cause of this, where the condition is due to the presence of circulating anti-glomerular basement membrane antibodies (anti-GBM antibodies) is termed Goodpasture's disease (although this wasn't the cause of the cases he described).Other causes of pulmonary haemorrhage and renal failure include Wegener's granulomatosis, microscopic polyangiitis and systemic lupus erythematous (SLE). B : 2. A 75-year-old man with a 30 pack year cigarette smoking history complains of continuous right shoulder pain, a persistent cough and weight loss. His chest radiograph shows a right apical shadow. Which of the following signs are consistent with a right Horner’s syndrome? A: Ipsilateral papillary constriction B: Contralateral papillary constriction C: Ipsilateral pupillary dilatation D: Contralateral pupillary dilatation E: Ipsilateral ptosis F: Contralateral ptosis G: Ipsilateral enophthalmos H: Contralateral enophthalmos I: Ipsilateral homonymous hemianopia J: Contralateral anhydrosis K: Ipsilateral deviation of the tongue on protrusion L: Inability for right lateral gaze M: Failure of the ipsilateral pupil to react to light N: Failure of the contralateral pupil to react to light O: Absent knee and ankle reflexes. Comment : Horner’s syndrome consists of papillary constriction, enophthalmos, ptosis and loss of sweating on the same side of the face due to interruption of the sympathetic supply to the pupil. This may occur in the brainstem, cervical part of the spinal cord, or in the thoracic outlet. Holmes–Adie syndrome is a benign condition. The affected pupil is larger than the other and appears not to react to light or accommodation, but in fact the reactions are slow and prolonged. The pupil is myotonic and in association with absent knee and ankle jerks forms Holmes-Adie syndrome. A : E : G: 3. A 45-year-old man is taking long-term theophylline for asthma. One evening, he is admitted to the Accident and Emergency department with convulsions. You suspect theophylline toxicity. Which one of the following statements is true? A: His convulsions should not be treated until a theophylline level is available. B: Theophylline toxicity may have been precipitated by the concomitant prescription of phenytoin. C: Theophylline toxicity may have been precipitated by the concomitant prescription of erythromycin. D: Theophylline toxicity only occurs in the elderly. E: Theophylline is an example of a drug with a wide therapeutic range (therapeutic index). Convulsions should be treated immediately in the usual way, without waiting for confirmation of the theophylline level.Theophylline is metabolised by the CYP450 enzymes in the liver. Erythromycin inhibits CYP450 enzymes and increases the half-life of theophylline and hence plasma theophylline concentrations, which may lead to toxicity. By contrast, phenytoin induces CYP450 enzymes, which will decrease the half-life of theophylline and may lead to inadequate therapeutic levels.Theophylline toxicity is more likely in the elderly due to age-related reduction in the rate of its metabolism, but it can occur at any age.Theophylline is an example of a drug with aNARROW therapeutic range. It is recommended that plasma theophylline levels be maintained between 10 and 20 mg/l. C :

53

MOHAMMED IS-HAG

4. Which one of the following statements regarding the lung and pleura is NOT true? A: The lung apex extends about 3cm above the medial part of the clavicle. B: The horizontal fissure separates the right middle lobe from the right lower lobe. C: The horizontal fissure may be visible on a plain radiograph of the chest. D: The lower margin of the pleura is about two ribs below the lower margin of the lung. E: The lower parts of the lung and pleura overlap the right surface of the liver. Comment : The apical pleura and lung project about 3–4 cms above the inner aspect of the clavicle, where they are related to the subclavian vessels and the brachial plexus: hence a chest radiograph to exclude pneumothorax is warranted following subclavian vein cannulation. The oblique fissure lies between the upper and lower lobes. The horizontal fissure, often seen on a plain radiograph, demarcates the middle lobe from the upper lobe on the right side. The lower border of the lung extends up to the tenth rib at the back and the pleura up to the twelfth, hence the lung and pleura overlaps the liver, kidney and spleen. B : 5. A 62-year-old woman is admitted with an infective exacerbation of chronic obstructive pulmonary disease (COPD). Admission blood gases taken while breathing 28% oxygen are as follows: pH 7.31, pO2 8.4 kPa and pCO2 7.4 kPa. Which of the following best describes the blood gas picture? A: Compensated Type I respiratory failure B: Compensated Type II respiratory failure C: Decompensated Type I respiratory failure D: Decompensated Type II respiratory failure E: Partially compensated respiratory alkalosis. Comment : These blood gases show the patient is hypoxic on oxygen and the elevated pCO2 combined with the low pH confirm a respiratory acidosis. Therefore, the blood gas picture is of decompensated Type II (respiratory) failure and the prognosis is poor unless treated appropriately, including consideration of non-invasive positive pressure ventilation (NIPPV). D : 6. An 86-year-old lady, generally fit and self-caring, is admitted with an acute exacerbation of chronic obstructive pulmonary disease (COPD). One hour after admission she remains distressed with a respiratory rate of 30/minute and is peripherally cyanosed. Repeated arterial blood gases show a severe respiratory acidosis with a pH of 30) and Blood Pressure (systolic < 90, diastolic 10% and >200 ml as a positive response. E: Give oral prednisolone, 30 mg daily for 2 weeks, regarding a clear statement of subjective improvement by the patient as a positive response. Comment : Steroid challenge is indicated in chronic obstructive pulmonary disease (COPD) of more than moderate severity. Standard practice would be to give prednisolone 30 mg daily for 2 weeks, regarding an increase in FEV1 of >10% and >200 ml as a positive response. Given the non-specific effects and many side effects of steroids, it is crucial to demonstrate functional improvement: many patients with COPD have sustained severe complications of steroid treatment, e.g. vertebral fracture, without any evidence that the steroids were beneficial for their chest. D : 13. A 35-year-old woman is referred with a history of red, painful legs of 3 weeks duration that have not responded to a course of amoxycillin and flucloxacillin given for presumed cellulitis. She is afebrile, does not have any other symptoms, and has never smoked. Examination reveals tender purple / red nodules on her shins. Her full blood count, kidney and liver function tests are normal. A chest radiograph shows prominent hilae. What is the appropriate management? A: Arrange bronchoscopy and bronchoalveolar lavage to exclude malignancy B: Start azathioprine and prednisolone; follow up in clinic C: Start prednisolone; follow up in clinic D: Start simple analgesics; follow up in clinic E: Arrange CT scan of the lungs and lung biopsy Comment : The combination of bilateral hilar lymphadenopathy and erythema nodosum is diagnostic of sarcoidosis. This is usually self-limiting. She should however be followed up in clinic with full lung function tests including transfer factor and lung volumes. Serum angiotensin-converting enzyme (ACE) level and lung functions can be used to monitor disease. Worsening disease should be treated with prednisolone. D :

56

MOHAMMED IS-HAG

14. A 63-year-old man attends the hospital with a history of proximal muscle weakness. He also gives a history of cough of 8 weeks duration and complains of pain of the small joints of the hands. He has small haemorrhages in the nail folds, but is not clubbed. In the chest he has bibasal crackles, and a chest radiograph reveals diffuse reticular infiltrates. Lung function tests confirm a restrictive pattern. What is the underlying cause of his interstitial lung disease? A: Cryptogenic fibrosing alveolitis B: Ankylosing spondylitis C: Polymyositis and dermatomyositis D: Rheumatoid arthritis E: Mixed connective tissue disorder. Comment : Polymyositis and dermatomyositis are inflammatory conditions involving the muscle and skin. Patients often complain of proximal muscle weakness and of pain in the small joints of the fingers. They may have ragged cuticles and haemorrhages at the finger nail folds. Interstitial lung disease can occur. Underlying malignancy (lungs, ovaries, breasts and stomach) is present in 5-8% of cases. C : 15. A 38-year-old asthmatic woman presents with an acute attack. Her arterial blood gases breathing air are as follows: pH 7.36, pO2 9.8 kPa, pCO2 5.2 kPa. These mean: A: the attack is not severe B: she should be given supplemental oxygen, but is unlikely to need a high FiO2 to achieve normoxia C: cardiorespiratory arrest could be imminent D: her respiratory effort may be failing because she is getting tired E: she could have had a pneumothorax. Comment : A normal or elevated pCO2 in an asthmatic indicates failing respiratory effort, and although this woman's oxygen saturation is not severely depressed she is in danger of decompensation and - aside from high flow oxygen, nebulised salbutamol and ipatropium, and steroids - it would be prudent to inform the ICU of her existence. The gases are not bad enough, however, to suggest that cardiorespiratory arrest is imminent.Pneumothorax must be excluded in any asthmatic, but the presence or absence of pneumothorax can never be inferred from arterial blood gas analysis. D : 16. A 34-year-old previously fit homosexual man is admitted to the hospital with a one-week history of dyspnoea. His chest radiograph shows bilateral alveolar infiltrates and apart from a PaO2 of 6Kpa on breathing room air, his routine blood tests are normal. The admitting doctor makes a diagnosis of Pneumocystis carinii pneumonia (PCP). Which of the following is NOT true? A: Unilateral pleural effusions are commonly seen in this condition. B: The patient should be treated with intravenous co-trimoxazole. C: The patient should be treated with prednisolone. The patients lung function tests will show a reduced diffusing capacity for carbon monoxide (DLCO). D: E: PCP is associated with the syndrome of inappropriate ADH (SIADH) secretion. Comment : PCP is commonly seen in the immunosupressed. Patients usually complain of cough and dyspnoea. The cough is usually non-productive. Patients are usually hypoxic and chest radiograph characteristically shows bilateral alveolar infiltrates. Pleural effusions are very rare. The patient should receive intravenous co-trimoxazole in a dose of 120mg/kg for at least 3 weeks. Early administration of glucocorticoids to patients with moderate to severe hypoxia, decreases the risk of respiratory failure and death by over 50%. Pulmonary function tests demonstrate a decreased vital capacity, increased residual capacity, normal flow rates and a reduced DLCO. SIADH is known to occur with PCP. A : 17. A 27-year-old lady with asthma was admitted with an acute attack. Her symptoms started around 8.00am, but by 10.00am she was so unwell that she needed admission to the hospital. Shortly after arriving to the accident and emergency she was admitted to the intensive care unit. Similar incidents had occurred twice in the past. Which two of the following statements are correct for near fatal asthma? A: Previous severe asthma attacks are a risk factor. B : The short time lag between the start of symptoms and hospital admission is a risk factor. C: Female patients are especially at risk. D: The risk increases in obese patients. E: Family history of asthma is common. F: Allergy to peanut is commoner in patients with near fatal asthma. G: Near fatal asthma occurs more frequently during winter season. H: Long acting beta2 agonists are beneficial in preventing asthma attacks. I: Childhood eczema is a risk factor. J: Parental smoking is a risk factor. Comment : Near fatal asthma or brittle asthma is responsible for around 1000 deaths every year in the UK. It mainly occurs in young patients. Previous attacks and the short time between the start of symptoms and hospital admission are the two main risk factors. The treatment of choice is hospitalisation with early start of nebulised bronchodilators, oxygen and intravenous corticosteroids.

57

MOHAMMED IS-HAG

The recovery rate is normally quick. Long acting beta2 agonists and leukotrine receptor antagonists are not successful in preventing near fatal asthma. There is no known relationship between near fatal asthma and personal history of allergy. The incidence of the disease does not differ according to gender and there are no seasonal variations for the rate of hospitalisation. A : B : 18. A 60-year-old man who smoked 20 cigarettes a day for 40 years enquired about recent advances in screening for lung cancer that he had read on the internet. Features of low-dose thoracic CT scans for lung cancer (LC) screening studies include: A: low sensitivity B: high specificity C: low cost D: no contrast media required E: the size of malignancies detected are no smaller than with chest radiograph F: less radiation than chest radiograph G: low false positive rate H: high false negative rate I: positive predictive value (PPV) over 50% J: low-dose CT detects more cases of lung cancer than chest radiograph. Comment : ADVANTAGES · Extremely sensitive and reliable: able to detect LC at early stages · No contrast media · Radiation dose of low-dose CT is 1/7th the radiation dose of a standard CT DISADVANTAGES · Expensive: $350 · Radiation exposure is about 10 times more than one for chest radiograph · False-positive results are very common. PPV is less than 10%. · Spiral CT scanning screening Two non-randomized studies from Japan: - almost 7000 people - smokers and non-smokers over 40 years of age -chest radiograph, sputum cytology and low-dose CT Early Lung Cancer Action Project, non-randomized trial: - 1000 people - high smokers over 60 years of age - chest radiograph and low-dose CT. RESULTS: Low-dose CT detects more cases of lung cancer than chest radiograph (27/1000 vs 9.1 to 7.6/1000) Low-dose CT compared with chest radiograph detected: - Non-calcified nodules: three times as commonly (23% vs 7%) - Malignancies: four times as commonly (2.7% vs 0.7%) - Stage I malignancies: six times as commonly (2.3% vs 0.4%) The size of malignancies are smaller with low-dose CT. However, the prevalence-screening rate for advanced disease on low-dose CT did not decrease. True clinical significance of small tumours founded by screening is unknown. CT founded at least one indeterminate nodule in 23% of patients. D : J: 19. A 28-year-old man is has recently been discharged from hospital after treatment for pneumococcal pneumonia. He has had repeated courses of antibiotics for sinus, ear and lower respiratory tract infections, and had sinus surgery the previous year. He is a life long non-smoker and is not on medication. His blood count prior to discharge was normal. In the absence of further clues in the history or examination, which single blood test is the most important? A: HIV antibody test B: Pneumococcal antibodies C: Immunoglobulin levels D: Liver function tests E: IgG subclass levels. Comment : Hypogammaglobulinaemia is associated with recurrent bacterial infections, most commonly of the respiratory tract. Delay of several years prior to diagnosis is usual, with associated morbidity. Patients with low immunoglobulin levels and recurrent infections should be treated with immunoglobulin replacement. More minor antibody defects, such as IgG subclass or specific antibody (to pneumococcus) defects can often be treated with appropriate vaccinations and/ or prophylactic antibiotics. C :

58

MOHAMMED IS-HAG

20. A 74-year-old man was treated in hospital for exacerbation of Chronic Obstructive Pulmonary Disease (COPD). His condition has improved significantly and he is keen to go home. His repeated arterial blood gas (ABG) reading (on air) showed a pH of 7.35, pCO2 of 4.5kPa, pO2 of 7.1kPa, HCO3 of 26 mmo/L. What action would be most appropriate? A: Discharge the patient, prescribing an oxygen cylinder. B: Discharge the patient once his ABG returns to normal. C: Request an oxygen concentrator and discharge the patient once an oxygen concentrator is fitted. D: Discharge the patient and arrange a follow up in 6 weeks' time with a repeated ABG. E: Discharge the patient and advise a follow up by his GP. Comment : Long-term oxygen treatment is recommended if pO2 is less than 7.3kPa on two occasions in remission of COPD. This patient's p02 is likely to improve when he recovers from his current exacerbation and his ABG should be re-checked in about 6 weeks time. As he seems able to tolerate his mild hypoxia, he would not require any oxygen supplementation at home at this stage. D : 21. A 65-year-old lady is admitted with left sided pneumonia and pleural effusion. Pleural fluid is aspirated and sent for tests. Which of the following is an indication for inserting a chest drain? A: Pleural fluid pH 2mmol/l E: Pleural fluid lactate dehydrogenase > 200IU/l. Comment : Infected pleural effusions should be drained. Infected pleural effusions should be drained if the pH7mmol/l B: Confusional state C: PaO2 30/min E: Diastolic BP 50 years Co-existing disease · Core clinical adverse prognostic features (CURB criteria): Confusional state · Urea >7mmol/l · Respiratory rate >30/min · Systolic BP 30/min, and hypotension (SBP 1.5 cm (on ultrasound) and co-morbid vascular disease. If a patient has none of these then renovascular disease is highly unlikely, Evidence of end-organ damage is not helpful in differentiating aetiology. A : 6. A 23-year-old African woman presents with seizures, hypertension, a rash, a raised ESR, a normal CRP and a creatinine of 373 micromoles/l. What is the most likely diagnosis? A: Myeloma B: Hashimoto’s disease C: Systemic lupus erythematosus D: Staphylococcal septicaemia E: Sickle cell disease. Comment : Systemic lupus erythematosus is a common condition and the prevalence is higher in women than men and is higher in black people than white. Typical features include skin rashes, neurological or psychiatric abnormalities and renal disease. It is often found that although the ESR is elevated, the CRP is normal. C :

96

MOHAMMED IS-HAG

7. A pregnant 35-year-old presents with right upper quadrant pain at 28 weeks gestation. On examination, her blood pressure is 180/105. She has proteinuria, a creatinine of 120micromoles/litre and abnormal liver function tests. Her blood film shows a microangiopathic haemolytic anaemia and her platelet count is 30. A few weeks previously her blood pressure was 110/80 and her platelet count was 120. Which two of the following statements are true? A: As a pregnant woman her blood pressure would be expected to be higher than before the pregnancy. B: The low platelets suggest that she has systemic lupus erythematosus. C: She certainly has essential hypertension. D: The development of proteinuria suggests pre-eclampsia. E: Blood pressure is usually higher in the first half than the second half of pregnancy. F: The problem is unlikely to recurr in a subsequent pregnancy. G: Alpha methyl dopa cannot safely be used to lower her blood pressure. H: Grand mal seizures may occur if the condition persists. I: She requires urgent dialysis. J: Acute liver failure is inevitable. Comment : Blood pressure falls during the first half of pregnancy and then rises again during the second half of pregnancy to levels approaching those before the pregnancy. Generally, blood pressure is a little higher in the second half than the first half of pregnancy. These changes probably reflect changes in peripheral resistance. Pregnancy induced hypertension can be severe and can be associated with endothelial and organ dysfunction. The clinical consequences of such dysfunction can be varied and certainly include proteinuria, renal impairment, abnormal liver function, microangiopathic haemolytic anaemia, platelet dysfunction and seizures. The priority is to reduce the blood pressure to prevent these changes. Aspirin is of prophylactic benefit in subsequent pregnancies, which carry a significant risk of recurrence. Alpha methyl dopa is a standard well tested drug used during pregnancy. Labetolol is also used. Other drugs, such as calcium channel blockers, have been used, but have a less well-established track record of safety during pregnancy D : H: 8. A 68-year-old lady presents with a 4-day history of anuria. She has a past history of intermittant claudication and is a long-term smoker. Which of the following statements are correct? A: If kidneys are of normal size renal artery occlusion can be excluded. B: If there is evidence of renal asymmetry and first investigation should be a biospy of the smaller kidney. C: An IVU would be the investigation of choice. D: One should consider urgent arteriography and revascularisation. E: Weakly positive pANCA serology would establish a diagnosis of microscopic polyangitis. Comment : Absolute anuria would be a rare presentation of a rapidly progressive glomerulonephritis. A weakly positive pANCA is a relatively common finding and in many instances is a false positive result. One would need histology to confirm the diagnosis in these circumstances. Renal artery occlusion can present in this way and the presence of two normal size kidneys does to exclude the diagnosis. An IVU is not helpful in these instances, as non-opacification of the kidneys would occur in the presence of poor renal function irrespective of the cause. When there is renal asymmetry the larger of the two kidneys should be biopsied as the diagnostic yield is likely to be greater. Arteriography and revascularisation should be considered as this may well salvage the situation is carried out early enough. D : 9. An anxious 52-year-old woman has adult polycystic kidney disease. She shows you a long list of things that she says are associated with this condition. Which of the following is NOT a recognized association? A: Subarachnoid haemorrhage B: Cerebellar cysts C: Liver cysts D: Colonic diverticuli E: Mitral valve prolapse. Comment : Adult polycystic kidney disease (APKD) is the most common inherited renal disease leading to end-stage renal failure, accounting for 5-10% of those receiving renal replacement therapy. There are two known loci: PKD1 on chromosome 16 accounts for 85% of cases and PKD2 on chromosome 4 for 10%. Presentation is with abdominal pain, haematuria, hypertension, urinary tract infection, incidental discovery of an abdominal mass, or as a result of family screening (or serendipitous imaging of the kidneys, e.g. by ultrasound examination ordered for another purpose). B :

97

MOHAMMED IS-HAG

10.

PLATE 19 The enclosed image shows a portion of a percutaneous renal biopsy taken from a 48-year-old woman with nephrotic syndrome. This includes medulla and transitional cell epithelium. Which one of the following statements is correct? A: The appearances would be consistent with cholesterol embolisation. B: The appearances are secondary to proteinuria. C: A portion of a normal glomerulus is seen in the biopsy. D: The procedure was probably complicated by haemorrhage. E: There is evidence of interstitial fibrosis. Comment : Percutaneous renal biopsy aims to obtain samples of the renal cortex. Samples of renal medulla are not only less likely to yield diagnostic information, but also carry a substantially increased risk of haemmorhage, particularly if the renal pelvis is breached by the biopsy needle - as has clearly occurred in this case since the sample includes transitional epithelium. D : 11. A 25-year-old man has been feeling unwell for a few days and has noticed his urine is darker than usual. His GP finds that he is hypertensive (BP 160/100), his urine is positive for blood and protein, and his creatinine is 940 µmol/l. The patient remembers having a sore throat about a week before becoming ill. You consider post-streptococcal glomerulonephritis to be a likely diagnosis. Select the two answers that you agree with. A: A renal biopsy showing a paucimmune focal segmental glomerulonephritis with crescent formation would support your diagnosis. B: If your diagnosis is correct the patient has an approximately 85% likelihood of needing longterm renal replacement therapy. C: The patient could have chronic renal failure with small kidneys on renal ultrasound. D: Red cell casts in the urine would make post-infectious nephritis unlikely. E: It would be reasonable to offer the patient an outpatient appointment within 1 week and ask the GP to check the creatinine in 24 hours. F: Serological tests are generally unhelpful in diagnosing post-streptococcal nephritis. G: If your diagnosis is correct, plasma exchange should be offered as soon as possible. H: The patient may well need dialysis within 24 hours. I: The presence of antibodies to double stranded DNA would support your diagnosis. J: Typically in post-streptococcal glomerulonephritis circulating complement levels are normal. Comment : The patient certainly has renal failure. Whether this is acute or chronic, he is very likely to need dialysis within 24 hours. Indeed, emergency dialysis may be indicated depending on knowledge of the potassium, pH, the clinical status of the patient and whether there is a pericardial rub or a flap. Sore throats are common, and it may be that the patient has chronic renal failure, and the sore throat is a coincidence. Post-streptococcal nephritis generally resolves spontaneously and the likelihood of requiring long term renal replacement therapy if this is the underlying diagnosis is probably about 1%.

98

MOHAMMED IS-HAG

Post-streptococcal nephritis has a diffuse proliferative appearance on light microscopy, and is associated with the presence of antibodies to streptococcal antigens and hypocomplementaemia, both of which are important in making the diagnosis. C :H: 12. A 50-year-old man attends the Accident and Emergency department and reports that on passing urine he also passed a small lump. He retrieved this and has brought it along in a jam jar of water. He is otherwise well apart from severe osteoarthritis of the left hip. His serum creatinine is 160 micromoles/l and his albumin is 40 g/l. A frozen section is performed by a helpful pathologist who reports that the lump is a renal papilla. Which of the following could account for this in his case? A: Membranous nephropathy B: Non-steroidal anti-inflammatory drug use for his osteoarthritis C: Nephrotic syndrome D: HIV nephropathy E: End stage renal disease. Comment : Analgesics, including non-steroidal anti-inflammatory drugs can cause papillary necrosis. This is likely to be caused by vasoconstriction of blood vessels within the kidney. Prostaglandins cause tonic vasodilatation of renal blood vessels. Non-steroidal anti-inflammatory drugs block prostaglandin synthesis and so block this vasodilation, resulting in relative vasoconstriction. B : . 13. A 36-year-old renal transplant receipient of 18 months is admitted with a history of a rising serum creatinine, pyrexia, weight loss, anaemia, abnormal liver enzymes and cervical lymphadenopathy. His immediate post transplant period was complicated by an episode of cellular rejection and one of vascular rejection treated with methylprednisolone and anti thymoglobulin (ATG) respectively. Which of the following statements are true regarding his condition? A: Cytomegalovirus (CMV) disease is the most likely underlying problem. B: His anaemia is best treated with erythropoeitin. C: A transplant biopsy is unlikely to be helpful in establishing a diagnosis. D: If post transplant lymphoma is diagnosed, graded reduction in immunosuppression is likely to be of benefit. E: The features described are most consistant with chronic rejection. Comment : Post transplant lymphoma, or more correctly post transplant lymphoproliferative disease (PTLD), is associated with the degree of previous immunosuppression. In the case of this patient, he was exposed to baseline immunosuppressive drugs and also potent anti-rejection therapy in the form of ATG. It is often driven by Epstein–Barr virus and can be treated by graded reduction in immunosuppression. In a significant number of cases PTLD is present in the transplanted organ. Treatment of the underlying PTLD will often improve blood counts. It would be unusual for CMV disease to present at this time, usually occurring much earlier in the post transplant period. Chronic rejection rarely causes systemic symptoms such as those describe here. D: 14. A 36-year-old man presents with microscopic haematuria and hypertension. Ultrasound scan shows several cysts in the kidneys, and two solid lesions, the larger of which is 5 cm diameter. The man is not in contact with his family, but he knows that his father died undergoing surgery for a brain tumour aged 30 and also that one of his father's two sisters had some kind of kidney problem, and was deaf in one ear. Which of the following statements is true? A: Kidney surgery is unlikely to be indicated. B: Should the patient ever require renal replacement therapy, transplantation would not be an option. C: The most likely diagnosis is Alport's disease. D: The patient may have von Hippel Lindau disease. E: It is reasonable not to investigate the hypertension further. Comment : von Hippel Lindau disease is transmitted as an autosomal dominant condition. Affected individuals may have any of the following: · Renal cysts · Clear cell renal cell carcinoma (CCRCC) · Retinal angiomas · Central nervous system haemangioblastoma · Phaeochromocytoma. Another manifestation is endolymphatic sac tumours, causing deafness. Below 3 cm, solid lesions can be monitored, but above this size they need removal. Hypertension suggests the patient may have a phaeochromocytoma, and this should be actively sought as it would be important to remove it. Although surgeons try to preserve renal tissue, ultimately operations to remove CCRCC may lead to end stage renal failure. As long as there is no evidence of metastasis, transplantation can be considered. D :

99

MOHAMMED IS-HAG

15.

PLATE 20 A 52-year-old accountant, with a 6-year history of Raynaud’s phenomenon presents to her G.P. with a 4-month history of worsening dysphagia. Incubation of her serum on Hep 2 cells reveals the following immunofluorescent pattern (see image). A: Primary Raynauds disease B: systemic lupus erythematosus (SLE) C: Limited systemic sclerosis D: Rheumatoid arthritis E: Sjogren's disease. Comment : Raynaud’s phenomenon is common, occurring in 4-15% of the general population and in the majority of cases is not associated with connective tissue disease. However, the detection of an anticentromere nuclear staining pattern has a high specificity for limited systemic sclerosis, especially in the context of dysphagia which is a feature of the disease. C : 16. You see a 12-year-old with sickle cell (SC) disease who describes a recent episode of macroscopic haematuria. The parents are worried that the child has serious kidney disease. Concerning sickle cell disease, select the two statements which you agree with: A: Renal involvement is more likely in SC disease than in Hb S homozygotes. B: Hypertension is common in patients with sickle cell disease. C: Renal transplantation is contraindicated in patients with sickle cell disease and end stage renal ailure. D: Macroscopic haematuria is usually due to papillary necrosis. E: Survival of patients with sickle cell nephropathy on dialysis programmes is worse than patients with diabetes of the same age. F: Red cells in the vasa recta of the renal medulla are protected from sickling by the hypertonic environment. G: Loss of urinary concentrating ability is common in patients with sickle cell disease. H: Glomerular filtration rate is typically 30% of normal by the age of 10 years. I: Administration of recobinant human erythropoietin reduces the number of sickling crises by approximately 50%. J: Renal medullary carcinoma is a relatively indolent tumour in patients with sickle cell disease. Comment :In sickle cell disease, sickling is a particular problem in the medullary vasa recta due to hypertonicity and hypoxia. Reducing medullary hypertonicity may reduce renal damage. Diminished concentrating ability is an early and universal finding in sickle cell disease. Survival in patients with sickle cell nephropathy is similar to non-diabetic dialysis patients. Interestingly early in the course of sickle cell disease GFR is actually increased, and only falls below normal by the age of about 30. Hypertension is significantly less common in sickle cell disease than in normal black individuals. Renal medullary carcinoma is a highly aggressive tumour which occurs in patients with sickle trait or sickle cell disease. D : G: 17. A 30-year-old man, a haemodilaysis patient with diabetic nephropathy, has the following blood results: calcium 2.9 mmol/l, phosphate 2.5 mmol/l. Alkaline phosphatase and albumen are in the normal range and PTH is at the lower end of the normal laboratory range. The patient's current medication includes alfacalcidol each day and calcium acetate as a phosphate binder before each meal. Select the most appropriate statement. A: Phosphate restriction in the diet should be reinforced, a change of phosphate binder may be necessary and alfacalcidol should be reduced or stopped. B: The dialysate calcium should be lowered to return the calcium to the normal range. C: The calcium acetate dose should be reduced to return the calcium to the normal range. D: The patient has autonomous hyperparathyroidism. E: These values are satisfactory for a haemodialysis patient. Comment :

100

MOHAMMED IS-HAG

The calcium x phosphate product is high putting the patient at risk of metastatic calcification. Lowering the phosphate (rather than the calcium) is the most important measure, as the elevated phosphate stimulates parathyroid proliferation and PTH secretion. This is best done by reinforcing dietary restriction and ensuring that the patient is recieving adequate dialysis. The vitamin D analogue will be increasing absorption of calcium and phosphate so this should be reduced or stopped; ideally the patient will remain on a small dose as this helps to stop parthyroid proliferation.If these changes do not return the calcium and phosphate to satisfactory values, the patient should be changed to a phosphate binder which does not contain calcium. Aluminium based compounds are best avoided because of the risk of toxicity so sevelamer would be an appropriate (but expensive) choice. A : 18. Which of the following would you regard as the clearest contraindication to choosing continuous ambulatory peritoneal dialysis as treatment modality for a patient with endstage renal failure? A: Presence of large (22 cm) bilateral polycystic kidneys B: Known impairment of left ventricular function C: Severe visual impairment D: Previous succesful repair of an inguinal herinia E: Presence of a defunctioning colostomy and mucus fistula. Comment : Impaired left ventricular function is a more significant concern for haemodialysis than continuous ambulatory peritoneal dialysis (CAPD). Visual impairment raises some challenges for CAPD, but these can be overcome in most instances. Moderate size polycystic kidneys do not usually present a problem for CAPD. E : 19. Which of the following options do you think would be the most appropriate in supporting a diagnosis of reflux nephropathy in a 40-year-old man with plasma creatinine 540 micromole/litre, 2.5g proteinuria per 24 hours and a childhood history of repeated urinary tract infections? A: Micturating cystogram B: Renal ultrasound C: Intravenous urography D: Computerized tomography with intravenous contrast E: Isotopic imaging with 99mTc-DTPA. Comment : In the majority of patients with reflux nephropathy, ureteric reflux does not persist beyond childhood so a micturating cystogram is not usually helpful. Ultrasound is a good method of detecting renal scars, although it is less sensitive than intravenous urography or DMSA scanning in detecting small scars in those with normal renal function. Intravenous urography is often relatively unhelpful in patients with this degree of renal impairment due to poor concentration of contrast. DMSA, which is taken up by tubular cells after filtration by the glomerulus, can be used to detect scars (as opposed to DTPA which is filtered by the glomerulus and not taken up by tubular cells). B : 20. A 19-year-old army recruit presents with acute renal failure. Which of the following features is not consistent with a diagnosis of rhabdomyolysis? A: elevated plasma creatinine kinase level B: anuria C: elevated plasma creatinine D: blood and red cell casts in the urine E: hyperkalaemia. Comment : Rhabdomyolysis results from muscle injury and the release of myoglobin from muscle cells. Myoglobin is toxic to the renal tubules and oliguria or even anuria can occur. Myoglobinuria usually occurs. Muscle cells also release creatinine kinase and potassium when they are injured. Red cell casts are not a feature of rhabdomyolysis and would suggest a glomerular disease. D : 21. A 46-year-old man presents with a 33-year history of lithium carbonate therapy for bipolar affective disorder. He is polyuric (11 litres) and has 2 grams / 24 hours of proteinuria. His GFR is 47-mls/min/1.73 m2. BP is raised at 150/100 mm Hg. He has a renal biopsy. What would be typical findings? A: IgA glomerulonephritis B: Crescentic glomerulonephritis C: Renal Vasculitis D: Focal glomelurosclerosis and interstitial nephritis E: Granulomata. Comment : Lithium toxicity is much more common than appreciated by most psychiatrists. It is rarely detected early enough for reversibility. It is typically a chronic interstitial nephritis but a secondary FSGS lesion due to hyperfiltration of remnant nephrons can be seen in 30% of cases. Options include running lithium at very low levels, or a switch to valproate or carbemazepine. D :

101

MOHAMMED IS-HAG

22.

PLATE 21 A 28-year-old woman received a kidney transplant from her brother. Six weeks post transplantation she was admitted with epigastric pain and vomiting. She was febrile. Simple blood tests demonstrated markedly abnormal LFTs with a hepatocellular pattern. An oesophago-gastroduodenoscopy (OGD) was performed and the gastric biopsy is shown below. Which of the following medications had been given at the time of transplantation to try to prevent this complication? A: Aspirin B: Ranitidine C: Nifedipine D: Septrin E: Acyclovir. Comment : The gastric biopsy shows a number of very abnormal large cells. These cells contain both nuclear and cytoplasmic inclusion bodies. They are characteristic of cytomegalovirus infection . The donor in this case was CMV positive and the recipient was CMV negative. This is a very high risk combination for CMV disease and antiviral prophylaxis was given in the form of acyclovir. This may not be the most effective prophylaxis and some units will use either oral gancicolvir or valacyclovir. The diagnosis in this case was further confirmed by polymerase chain reaction (PCR) analysis of blood to detect CMV DNA and antingenaemia testing to show the presence of replicating CMV in the patients WBCs. She responded well to intravenous ganciclovir.E: 23. A 36-year-old renal transplant receipient of 18 months is admitted with a history of a rising serum creatinine,pyrexia,weight loss,anaemia,abnormal liver enzymes and cervical lymphadenopathy. His immediate post transplant period was complicated by an episode of cellular rejection and one of vascular rejection treated with methylprednisolone and anti thymoglobulin (ATG) respectively. Which of the following statements are true regarding his condition? A: Cytomegalovirus (CMV) disease is the most likely underlying problem. B: His anaemia is best treated with erythropoeitin. C: A transplant biopsy is unlikely to be helpful in establishing a diagnosis. D: If post transplant lymphoma is diagnosed, graded reduction in immunosuppression is likely to be of benefit. E: The features described are most consistant with chronic rejection. Post transplant lymphoma, or more correctly post transplant lymphoproliferative disease (PTLD), is associated with the degree of previous immunosuppression. In the case of this patient, he was exposed to baseline immunosuppressive drugs and also potent antirejection therapy in the form of ATG .It is often driven by EpsteinBarr virus and can be treated by graded reduction in immunosuppression. In a significant number of cases PTLD is present in the transplanted organ. Treatment of the underlying PTLD will often improve blood counts. It would be unusual for CMV disease to present at this time, usually occurring much earlier in the post transplant period. Chronic rejection rarely causes systemic symptoms such as those describe here. D :

102

MOHAMMED IS-HAG

24. A 30-year-old man, a haemodilaysis patient with diabetic nephropathy, has the following blood results: calcium 2.9 mmol/l, phosphate 2.5 mmol/l. Alkaline phosphatase and albumen are in the normal range and PTH is at the lower end of the normal laboratory range. The patient's current medication includes alfacalcidol each day and calcium acetate as a phosphate binder before each meal. Select the most appropriate statement. A: Phosphate restriction in the diet should be reinforced, a change of phosphate binder may be necessary and alfacalcidol should be reduced or stopped. B: The dialysate calcium should be lowered to return the calcium to the normal range. C: The calcium acetate dose should be reduced to return the calcium to the normal range. D: The patient has autonomous hyperparathyroidism. E: These values are satisfactory for a haemodialysis patient. Comment : The calcium x phosphate product is high putting the patient at risk of metastatic calcification. Lowering the phosphate (rather than the calcium) is the most important measure, as the elevated phosphate stimulates parathyroid proliferation and PTH secretion. This is best done by reinforcing dietary restriction and ensuring that the patient is recieving adequate dialysis. The vitamin D analogue will be increasing absorption of calcium and phosphate so this should be reduced or stopped; ideally the patient will remain on a small dose as this helps to stop parthyroid proliferation. If these changes do not return the calcium and phosphate to satisfactory values, the patient should be changed to a phosphate binder which does not contain calcium. Aluminium based compounds are best avoided because of the risk of toxicity so sevelamer would be an appropriate (but expensive) choice. A : 25. A 28-year-old man has had several attacks of renal colic. He passes stones consisting mainly of calcium oxalate. His 24-hour urinary calcium excretion is twice the upper limit of normal. Which two alterations to his diet are most likely to be helpful in reducing the risk of stone recurrence? A: Reduced salt intake B: Increased fluid intake C: Avoidance of alcohol D: Avoidance of refined sugar E: Avoidance of foods high in oxalate F: Instigation of a calcium-free diet G: Avoidance of dairy products H: Increased salt intake I: Reduced fluid intake J: Avoidance of dietary meat. Comment : Whatever the cause of urinary stones, increasing fluid intake always reduces the concentration of stone-forming substances in the urine and is always the most important modification to diet. Patients should be instructed to drink enough to obtain a urinary volume of over 3 litres / day, and those who continue to be troubled by stones may need to drink even more. Hypercalciuria, usually idiopathic, is found in 65% of patients with urinary stones and is explained by intestinal hyperabsorption of calcium. However, reducing dietary calcium intake is NOT the answer, since this leads to increased intestinal absorption of oxalate and can thereby increase the risk of stone formation. Avoidance of nonstaple foods high in oxalate (e.g. rhubarb, spinach) is sensible. There is also a (lesser) role for reduction in intake of salt and refined sugar. B : E : 26. A 45-year-old man with polycystic kidney disease and plasma creatinine 300µmol/l is found to have potassium 5.1 mmol/l, calcium 2.25 mmol/l, albumin 40 g/l, phosphate 1.65 mmol/l, and PTH 20 pmol/l (NR 1.1-6.8). He has already seen a specialist dietician for advice concerning potassium and phosphate intake. He is currently taking an angiotensin-converting enzyme (ACE)inhibitor for hypertension and a proton pump inhibitor for reflux oesophagitis. Which of the following statements is true? A: He should be commenced on alfacalcidol 0.5 mcg od, to correct deficiency of active vitamin D and suppress parathyroid hormone (PTH). B: An appropriate treatment would be calcium acetate 1g bd to be taken with meals. C: Aluminium hydroxide should be the first choice if a phosphate binder is prescribed. D: Parathyroidectomy should be considered, especially if he is keen on having a renal transplant in the future. E: Sevelamer (Renagel) should definitely be used here rather than other phosphate binders. Comment : Hydroxylated vitamin D does suppress PTH expression, and alfacalcidol should be considered for prophylaxis against renal bone disease and progressive hyperparathyroidism. However, the patient's phosphate level is already elevated, and vitamin D supplementation will tend to increase this further. Even if the phosphate level were not elevated this would be an inappropriately high starting dose. Parathyroidectomy is not indicated here - the patient only has a mildly elevated level of PTH which is likely to improve with better phosphate control. There would also be a risk of adynamic bone disease.

103

MOHAMMED IS-HAG

Aluminium-containing phosphate binders carry the risk of aluminium accumulation and CNS effects. Sevelamer is only clearly indicated if calcium-containing binders cannot be used because the calcium level (or calcium-phosphate product) are undesirably high. Calcium acetate is a very reasonable choice here. It should be taken with (or just before) meals and may offer advantages over calcium carbonate, especially in patients with reduced gastric acidity. B : 27. A 37-year-old man, a type I diabetic for six years, presents with a rising creatinine over two years. There is mild background diabetic retinopathy and no neuropathy. Proteinuria was also noted two years ago, and is now 12 grams / day and he is nephrotic. He has no microscopic haematuria. BP is mildly elevated at 145/95 mm Hg. This is not typical of diabetic nephropathy because: A: Onset of renal problems too soon after diagnosis of type I DM B: Patients under 40 do not get renal diabetic complications C: Significant hypertension is invariable in diabetic nephropathy D: The absence of microscopic haematuria excludes a glomerular lesion E: Nephrotic syndrome is not due to diabetic nephropathy. Comment : Typically it is 10 - 20 years after the diagnosis of type I DM that patients first experience micro-albuminuria, then heavier proteinuria, and renal functional decline (as hyperfitration and raised GFR finishes). Thus this is too soon, and also the proteinuria did not precede the renal impairment. Hypertension is seen in about 90% of patients with diabetic nephropathy (remember elevated BP in this setting is anything > 130/80 mm Hg). Few patients with diabetic nephropathy have microscopic haematuria, but some do). About one third of diabetic nephropathy patients go through a nephrotic phase. This patient requires a renal biopsy (and had membranous nephropathy which responded to immunosuppression). A : 28. In a patient presenting with the nephrotic syndrome, which one of the following features is not a recognised association? A: Diabetes mellitus B: Myeloma C: Cigarette smoking D: HIV infection E: Systemic lupus erythematosus. Comment : Diabetic nephropathy can cause heavy proteinuria, sufficient to cause the nephrotic syndrome. Myeloma can cause AL amyloidosis which is associated with proteinuria that can be severe enough to cause the nephrotic syndrome. HIV infection has been associated with a form of focal segmental glomeruosclerosis which is a cause of the nephrotic syndrome. Systemic lupus erythematosus can cause various patterns of glomerulonephritis, including a membranous nephropathy which is a cause of the nephrotic syndrome. C : 29. A 45-year-old man with polycystic kidney disease and plasma creatinine 300µmol/l is found to have potassium 5.1 mmol/l, calcium 2.25 mmol/l, albumin 40 g/l, phosphate 1.65 mmol/l, and PTH 20 pmol/l (NR 1.1-6.8). He has already seen a specialist dietician for advice concerning potassium and phosphate intake. He is currently taking an angiotensin-converting enzyme (ACE)inhibitor for hypertension and a proton pump inhibitor for reflux oesophagitis. Which of the following statements is true? A: He should be commenced on alfacalcidol 0.5 mcg od, to correct deficiency of active vitamin D and suppress parathyroid hormone (PTH). B: An appropriate treatment would be calcium acetate 1g bd to be taken with meals. C: Aluminium hydroxide should be the first choice if a phosphate binder is prescribed. D: Parathyroidectomy should be considered, especially if he is keen on having a renal transplant in the future. E: Sevelamer (Renagel) should definitely be used here rather than other phosphate binders. Comment : Hydroxylated vitamin D does suppress PTH expression, and alfacalcidol should be considered for prophylaxis against renal bone disease and progressive hyperparathyroidism. However, the patient's phosphate level is already elevated, and vitamin D supplementation will tend to increase this further. Even if the phosphate level were not elevated this would be an inappropriately high starting dose. Parathyroidectomy is not indicated here - the patient only has a mildly elevated level of PTH which is likely to improve with better phosphate control. There would also be a risk of adynamic bone disease. Aluminium-containing phosphate binders carry the risk of aluminium accumulation and CNS effects. Sevelamer is only clearly indicated if calcium-containing binders cannot be used because the calcium level (or calcium-phosphate product) are undesirably high. Calcium acetate is a very reasonable choice here. It should be taken with (or just before) meals and may offer advantages over calcium carbonate, especially in patients with reduced gastric acidity. B :

104

MOHAMMED IS-HAG

30.

PLATE 3 A 73-year-old man presents with a three month history of feeling non-specifically unwell, has lost 6 kg in weight and has recently noticed the rash shown in the enclosed image. The creatinine is 340 µmol/l. Urinalysis shows blood and protein. Renal ultrasound shows kidneys of 10.8 and 11.2 cm with increased cortical echogenicity. Which of the following statements is correct? A: Renal biopsy should be performed and is likely to show a focal necrotising glomerulonephritis B: The renal impairment is likely to be chronic (i.e. irreversible) C: Skin biopsy is likely to yield a specific diagnosis D: Renal biopsy should be performed and is most likely to show membranous glomerulonephritis E: Laboratory blood tests are unlikely to yield a specific diagnosis. Comment : The rash shown is consistent with a leucocytoclastic vasculitis, and the history would be entirely consistent with ANCA (antineutrophil cytoplasmic antibodies) positive systemic vasculitis. A skin biopsy will not yield a specific diagnosis, whereas a test for ANCA is likely to be positive.In view of the preserved renal size, active urine sediment and rash, it is likely that there is an acute (i.e. reversible) component to the renal failure and a biopsy should be performed. This is most likely to show a pauciimmune, focal segmental glomerulonephritis, but it is important to exclude other conditions. A : 31. In a patient with diabetes mellitus and a serum creatinine of 210 micromoles per litre, which one of the following findings would lead you to suspect a diagnosis other than diabetic nephropathy? A: Normal fundoscopy B: HbA1C of 8.1% C: Normal-sized kidneys on ultrasound examination D: Proteinuria of 1.2g per 24 hours E: Knowledge that the creatinine had been 110 micromoles per litre 18 months earlier. Comment : Diabetic nephropathy is a microvascular complication and rarely occurs without other evidence of microvascular disease, such as retinopathy. It is often associated with suboptimal glycaemic control and proteinuria. Renal size is preserved but following the development of microalbuminuria the disease may progress with alarming rapidity. A : 32. In renal amyloidosis, which one of the following statements is correct? A: nephrotic syndrome is a rare presentation B: amyloid deposits can affect the glomeruli, tubules and blood vessels C: there are nodular glomerulosclerotic lesions that mimic diabetic nephropathy D: renal vein thrombosis is not seen E: regression of deposits, and resolution of proteinuria, cannot be achieved.

105

MOHAMMED IS-HAG

Comment : In renal amyloidosis there is amorphous deposition of amyloid proteins in the tissues. Clinical presentation is with proteinuria, nephrotic syndrome or chronic renal failure. Renal vein thrombosis may complicate, as it can any cause of the nephrotic syndrome. AL amyloid may respond to cytotoxic therapy, but many cases do not, and in patients who are elderly and frail a conservative approach is usually adopted. B : 33. A 17-year-old male presents with a short history of sudden onset of severely swollen ankles. He is otherwise well, and urinalysis shows +++ protein. A urine collection shows that he is excreting 6g protein per 24 hours, the plasma albumen is reduced, and the creatinine is in the normal range. What is the most likely underlying diagnosis? A: Thin membrane nephropathy B: Membranous nephropathy C: Minimal change nephrotic syndrome D: IgA nephropathy E: Mesangiocapillary glomerulonephritis. Comment : The patient has nephrotic syndrome and at this age the commonest underlying diagnosis on renal biopsy is minimal change nephrotic syndrome. This would also be consistent with the lack of haematuria, the sudden onset and the normal excretory renal function. Thin membrane nephropathy presents with asymptomatic haematuria. IgA nephropathy can present with nephrotic syndrome but this is relatively unusual and microscopic haematuria is usually present.Mesangiocapillary glomerulonephrits or membranous nephropathy can present with nephrotic syndrome but are less common. C : 34. A 25-year-old woman presents to A&E 1 hour after consuming 28 x 500mg paracetamol tablets. Which of the following statements is true? A: If the INR is normal on a sample taken four hours from the time of ingestion, liver damage is unlikely to occur. B: Alcohol ingestion at the time of consumption of paracetamol is an indication for N-acetylcysteine treatment if paracetamol level at 4 hours exceeds the ‘high-risk’ line. C: Activated charcoal may be beneficial if given immediately. D: Onset of tinnitus may be an early symptom of liver failure. E: Deterioration in conscious level within the first 24 hours usually suggests hepatic encephalopathy. Comment : Abnormal blood clotting following paracetamol overdose results from loss of production of clotting factors by hepatocytes, and is therefore a good early marker of synthetic liver function. The INR rises first because Factor VII has the shortest half-life and is therefore the first to disappear from the blood. However it is unusual to see any abnormality in blood clotting less than 18 hours from ingestion, so normal INR at 4 hours is unhelpful. Abnormal INR at the time of admission may indicate prior chronic liver disease, warfrin ingestion or suggest that ingestion of the drug occurred earlier than the patient reports. Because acute alcohol intoxication inhibits liver enzyme function, less paracetamol will be metabolised to the toxic metabolite, so that liver damage is, if anything, less likely. Chronic alcohol abuse causes liver enzyme induction and is an indication for treatment with N-acetyl cysteine if paracetamol level exceeds the ‘high risk’ line. Activated charcoal is only likely to be beneficial if given within 1 hour of ingestion of paracetamol. If the patient complains of tinnitus, this suggests concurrent salicylate consumption, which requires specific treatment according to the plasma level. Hepatic encephalopathy rarely occurs less than 48 hours from consumption; reduction in level of consciousness in the first 24-48 hours is usually a result of concurrent consumption of sedative drugs (particularly opiates in combination drugs such as co-dydramol of co-proxamol) or hypoglycaemia. Regular monitoring of blood glucose is recommended for patients with elevated paracetamol level requiring treatment with N-acetyl cysteine, or following any change in conscious level. C : 35. A 55-year-old man with long-standing type II diabetes and intermittent claudication has recently started on continuous ambulatory peritoneal dialysis (CAPD) for end stage renal failure (ESRF) which is presumed to be due to diabetic nephropathy. He is blood group A, his wife is blood group O, and they have two children. He is interested in the possibility of transplantation. Which of the following is true? A: Risk benefit analysis shows that diabetics with end stage renal failure are better treated with dialysis than transplantation. B: In the UK, wife to husband transplants are not permitted under current legislation. C: An exercise ECG should be performed to determine if he needs coronary angiography prior to transplantation. D: Recent evidence suggests that this patient would benefit from combined kidney and pancreas transplantation. E: If a transplant is performed the CAPD catheter would usually be left in situ at the time of the operation. Comment : Although diabetics with end stage renal disease have a relatively poor survival rate following transplantation, their survival on dialysis is dismal; risk benefit analysis shows that this group

106

MOHAMMED IS-HAG

may benefit most from transplantation compared to dialysis. Wife-to-husband transplants (and viceversa) are legal in the UK and the results are generally excellent. An exercise ECG is not sufficient evaluation here, especially as the intermittent claudication may limit the test. This patient would need coronary angiography, and iliac angiography, prior to transplantation. Pancreatic transplantation is desirable in type I diabetics. The PD catheter is usually left in situ - in the event that the graft does not work for a period the patient can then be dialysed easily. The catheter is usually removed 3 months after a successful transplant. E : 36. A 35 year-old women presented 2 hours after collapsing at home with severe headache. On examination she was drowsy and had neck stiffness. Her temperature was 37.5. She had a mild right hemiparesis. Which of the following is the most appropriate first diagnostic investigation? A: CT brain scan B: Lumbar puncture and examination of the cerebrospinal fluid C: Four-vessel angiography D: MR angiogram E: MR brain scan. Comment : The history is suggestive of a subarachnoid haemorrhage. Urgent CT brain scan will identify more than 95% of patients with suspected subarachnoid haemorrhage if performed within 1-2 days after headache onset (van Gijn 1982). Lumbar puncture is potentially dangerous and will add no extra information if brain CT shows definite evidence of extravasated blood. If CT is negative and there are no contraindications, LP should be performed.Four vessel angiography will be needed later once the patient is stabilised to identify the source of the bleeding and surgical or endovascular treatment. In the acute stages of SAH (because of the speed of investigation and availability) CT imaging is superior to MRI. MRI imaging becomes much more useful if presentation is delayed after the first few days (>4) when CT sensitivity for subarachnoid blood rapidly declines.MR angiography is without risks and reasonably sensitive (90%) – useful for screening people at risk of intracranial aneurysms but less suitable for patients with subarachnoid haemorrhage. A : 37. In atherosclerotic renal artery stenosis (ARAS), which of the following is a clear-cut reason for arterial intervention (i.e. angioplasty/stenting)? A: Presence of raised blood pressure B: Heavy proteinuria C: Flash pulmonary oedema D: > 75% stenosis but normal contralateral kidney E: Abdominal bruit. Comment : The number of compelling reasons for arterial intervention in this setting is modest perhaps only flash pulmonary oedema, and severe stenosis in a single functional kidney. In ARAS the effect on BP of successful angioplasty or stenting is modest and disappointing - though hidden within the unimpressive "group" response to intervention, there can be individual striking responses (here, it is the "Resistive Index" concept recently reported by Rademacher et al (N Engl J Med 346(6):410-417) which has proved potentially useful in determining which patients may respond well). Proteinuria can be trivial or nephrotic-range in ARAS depending on whether there is secondary FSGS. An abdominal bruit is not a useful sign except to indicate abdominal atherosclerosis (more accurately, turbulent blood flow). The presence of a stenosis alone (except to those with an uninhibited oculo-stenotic reflex, often remuneration-driven) is not enough to warrant intervention on present evidence - this is what the mainly UK-based ASTRAL study is investigating currently (contact me / John Scoble for details). C : 38. You are providing a seminar to patients approaching the need for renal replacement. A group of patients contemplating starting continuous ambulatory peritoneal dialysis (CAPD) ask you about medical contraindications to this modality. Which of the following is not a relative contraindication to CAPD treatment? A: Previous perforated diverticular disease and sigmoid colectomy B: Bilateral inguinal herniae C: Diabetes mellitus D: Severe chronic obstructive pulmonary disease E: Arthritis mutilans. Comment : Previous pelvic surgery reduces the likelihood of successful peritoneal dialysis, adhesions often making catheter placement difficult and reducing the peritoneal surface available for dialysis. Inguinal herniae rapidly fill with peritoneal dialysate causing patient discomfort and inefficient dialysis. Peritoneal dialysis is a good treatment for patients with diabetes mellitus although the glucose load in the dialysate will necessitate a change in insulin dosage, which can be administered intraperitoneally. Splinting of the diaphragm by a large volume of intraperitoneal fluid often exacerbates chronic obstructive pulmonary disease (COPD), and this should be considered a relative, but not absolute contraindication. Effective peritoneal dialysis requires a degree of manual dexterity and would not be easy for a patient with arthritis mutilans. C :

107

MOHAMMED IS-HAG

39.

PLATE22 A 30-year-old woman presents with arthralgia and tiredness. She is found to be hypertensive (160/100 mmHg). The creatinine is 300 micromol/l, there is 0.9 g proteinuria per 24 hours and the kidneys are normal size on ultrasound. Full blood count shows a slightly reduced platelet count, mild anaemia and evidence of microangiopathy. The patient's hand is shown (see image). Select one of the following answers: A: High-dose oral or intravenous steroids are the most effective treatment. B: Angiotensin-converting enzyme (ACE) inhibitors should be avoided in this setting. C: The most likely diagnosis is haemolytic–uraemic syndrome. D: Autoantibodies to glomerular basement membrane are likely to be present in the serum. E: Renal biopsy is likely to show arteriolar changes, fibrin thrombi and fibrinoid necrosis. Comment : The cutaneous appearances and renal presentation are typical of scleroderma with a renal crisis. In patients who develop a renal crisis this often occurs early in the course of the disease. There is usually relatively minor proteinuria, hypertension and evidence of microangiopathy. Typically the renal biopsy will show arteriolar changes and fibrinoid necrosis, and the best current treatment is with ACEinhibitors. E : 40. During a routine medical check, a 27-year-old man who is taking no medication is found to have a blood pressure of 180/97 and is found to have a low serum potassium. His urine contains only a trace of protein on dipstick analysis. Which of the following diagnoses is likely? A: Minimal change nephropathy B: Primary hyperaldosteronism C: Mesangiocapillary glomerulonephritis D: Primary hyperparathyroidism E: Pseudohypoaldosteronism. Comment : Aldosterone enhances distal nephron sodium reabsorption and potassium secretion in the collecting tubules. The result is that if aldosterone levels are raised, potassium depletion is commonly seen. High levels of aldosterone can also cause sodium retention and hypertension. A high blood pressure can itself cause damage to the glomeruli and a trace of proteinuria is a common finding in hypertensive patients. B : 41. Which of the following is a feature of X-linked hypophosphataemic vitamin D -resistant rickets? A: Elevated parathyroid hormone (PTH) B: Hypercalacaemia C: Bone pain D: Hyperphospataemia E: Low bicarbonate.

108

MOHAMMED IS-HAG

Comment : This is the commonest hereditary form of renal phospate wasting. Serum phospate is low and urine phosphate is inappropriately high. This in combination with abnormal osteoblast activity leads to abnormal mineralisation of growing bone and Rickets. The defect is in the PHEX gene coding for a Zn metallopeptidase, although the mechanism of pathogenesis is unclear. Treatment is with oral phosphate (difficult to tolerate) and high dose activated Vitamin D. Renal damage may ensue due to treatmentassociated hypercalcaemia. C : 42. In cholesterol embolisation syndrome: A: Fever is a recognised feature. B: Hypocomplementaemia is not seen. C: Inflammatory markers are not raised. D: Anti-coagulation is recommended. E: Statin-based lipid-lowering therapy is not helpful. Comment : In cholesterol embolisation syndrome: · Acute presentations can be associated with fever. · Low complement is seen as part of the pseudo-vasculitis. · C-reactive protein (CRP) and erythrocyte sedimentation rate (ESR) are raised and there may be an eosinophilia. · Anti-coagulation can cause the problem and is contra-indicated. · Statins help by stabilising atherosclerotic plaques. A : 43. Which of the following is NOT a recognised cause of renal disease in sarcoidosis? A: Membranous nephropathy B: Amyloidosis C: Granulomatous interstitial nephritis D: Nephrolithiasis E: Hypercalaemia. Comment : Renal disease in patients with sarcoidosis can be identified in up to 20% of cases at post mortem although clinically significant disease is less common. Up to 14% develop renal stone and hypercalcaemia may lead to tubular toxicity and dehydration. The classic lesion of granulomatous interstitial nephritis is often found on renal biopsy but again is not usually clinically significant. A variety of glomerular diseases including membranous nephropathy have been described although causal relationship is not certain. Amyloidosis is not a feature of sarcoid. B : 44. An 18-year-old woman had macroscopic haematuria at the time that she had a sore throat. Following this she is found to have persistent microscopic haematuria, 0.1g/24-hour proteinuria and a normal plasma creatinine. Blood pressure is 115/64. A renal biopsy shows mesangial deposition of IgA on immunofluorescence. The tubules and interstitium are normal. Which of the following statements is true?

A: An angiotensin-converting enzyme (ACE) inhibitor should be given since it is known to reduce the risk of progressive renal failure in this setting. B: There is approximately a 30% chance of end stage renal failure over the next 5 years. C: It is important that the patient's siblings are screened by testing their urine for blood. D: The prognosis is relatively good compared to other patients with IgA nephropathy. E: Circulating levels of C3 are likely to be reduced. Comment : Episodes of macroscopic haematuria, normal renal function, female sex, normal blood pressure, minimal proteinuria and normal tubulointerstitial morphology all place this patient in a good prognostic category. Although there is probably a genetic component to IgA nephropathy it is very unusual for relatives to be affected. D : 45. A 32-year-old man presents to his GP with a non-specific history of malaise. He has no history of diarrhoea. Initial investigations identify a significant anaemia with fragmentation on the blood film and low platelets. A clotting screen is normal. His biochemistry identifies significant renal dysfunction with a serum creatinine of 260umol/l. BP was recorded at 170/100. What is the most likely diagnosis? A: Thrombotic thrombocytopenic purpura (TTP) B: Haemolytic uraemic syndrome C: Malignant hypertension D: Disseminated intravascular coagulation E: Systemic vasculitis. Comment : This is a classic presentation of non-diarrhoeal haemolytic uraemic syndrome (D- HUS) or atypical HUS. This may be sporadic or familial. In sporadic cases association with HIV, malignancy, systemic lupus erythematous (SLE) and some drugs (e.g. cyclosporin) has been reported. In some familial cases mutations have been detected in Factor H which regulates alternative pathway activation of compliment.TTP produces a similar picture but usually also has evidence of neurological involvement. There is also evidence that the mechanism underlying the disease is different with reduced activity of a von Willebrand factor cleaving protease ADAMTS13. Treatment of sporadic HUS is with plasma exchange and fresh frozen plasma. The renal outlook is relatively poor and mortality around 10%. B :

109

MOHAMMED IS-HAG

46.

This patient presented acutely unwell with the following ECG (see image). What is the correct diagnosis? A: Hyperkalaemia B: Ventricular tachycardia C: Acute pericarditis D: Acute anterolateral myocardial infarction E: Digoxin toxicity. Comment : The presence of tall, peaked T waves, flattened P waves, prolonged PR interval and wide QRS complexes are pathognomonic of hyperkalaemia. Give IV calcium immediately (10mls, 10% calcium gluconate) and call the renal team. A : 47. The following are NOT used for the estimation of renal function: A: Serum creatinine B: Serum cystatin C C: Creatinine clearance D: Ethylenediaminetetraacetic acid (EDTA) clearance E: Urinary albumin/creatinine ratio. Comment : Serum creatinine is the most commonly used screening test for renal function. Cystatin C is currently being evaluated and may prove to be more accurate in mild renal impairment. Creatinine clearance is used as a surrogate test for measuring the GFR. The clearance of the radioisotope EDTA however is the most accurate marker of GFR in routine clinical practice. The albumin / creatinine ratio is a method of quantifying urinary protein losses and not an estimate of renal function. E : 48. Which of these statements concerning the kidney is correct? A: The main site of action of thiazide diuretics is the thick ascending limb of the loop of Henle. B: Renin is produced primarily in the collecting duct. C: Parathyroid hormone increases phosphate reabsorption by the kidney. D: The main site of action of frusemide is in the thick descending limb of the loop of Henle. E: Calcidiol is hydroxylated by 1-alpha hydroxylase in the tubular epithelial cells. Comment : Frusemide acts on the Na+K+2Cl- cotransporter in the thick ascending limb of Henle's loop; thiazides act mainly on the Na+Cl- cotransporter in the distal convoluted tubule. Parathyroid hormone decreases phosphate reabsorption by the kidney. Renin is produced by the juxta glomerular apparatus. E : 49. In systemic vasculitis: A: the kidney is rarely affected B: macroscopic haematuria is typical C: focal necrotising glomerulonephritis and crescent formation are typical D: large deposits of immunoglobulin are seen in glomeruli on immunofixation E: interstitial inflammatory change on a renal biopsy is not recognised.

110

MOHAMMED IS-HAG

Comment : Renal involvement is common, often with a very active urinary sediment (microscopic haematuria, red cell casts and proteinuria). Many cases are associated with a positive antineutrophil cytoplasmic antibody (ANCA), although this is neither diagnostic or essential. Immunostains are usually negative and the renal lesion is often termed pauci immune. C : 50. In the investigation of significant proteinuria (> 1 gram / 24 hours), the following are relevant factors: A: Albumin/creatinine ratio B: Urinary sodium concentration C: Urinary albumin to IgG ratio D: Tamm–Horsfall protein concentration E: Urinary complement concentration. Comment : Urinary albumin/creatinine ratio is another way to quantitate proteinuria. Urinary sodium has little bearing on proteinuria. Tamm–Horsfall protein is a normal constituent of urine. Urine complement levels are not of clinical use. The ratio of albumin leak to large MW protein leak is a marker of protein selectivity - in minimal change disease typically there is preferential albuminuria, whereas in many other renal protein leaking conditions in adults there is non-selective proteinuria. C : 51. A 60-year-old man has end-stage renal failure and is about to start renal replacement therapy. He has read a booklet about peritoneal dialysis, but seems to have got himself muddled. Which one of the following statements is true about peritoneal dialysis? A: it is inferior to haemodialysis B: it is associated with long-term changes in peritoneal membrane structure and function C: infection rates are typically more than 1 episode of peritonitis per 6 months of treatment D: it is contra-indicated in diabetic patients E: it means patients are unable to go to work because of dialysis exchanges. Comment : Peritoneal dialysis (PD) was the bedrock of dialysis in the UK in the 1970s, and has become so in other countries with inadequate health resources. In wealthier countries it has been slower to become established and accounts for a far smaller proportion of treated patients. Actuarial survival is the same, or better, for PD in the early stages of dialysis, but once residual renal function is lost, PD increasingly struggles to dialyse many patients well. Some motivated patients can work with co-operation from employers. Travel on PD is also much easier. Diabetic patients can fare well, though the glucose load from PD changes insulin requirements. Peritonitis rates of one infection per 18 - 24 patient months are now the norm with modern systems. Automated peritoneal dialysis (shorter duration high volume overnight exchanges while the patient is asleep) is more convenient for many patients, and can extend the useful life of PD in 'fast transporters', i.e. patients in whom the osmotic gradient collapses quickly. B : 52. A 23-year-old man presents with a rash on his legs. Stick testing of his urine reveals proteinuria + and haematuria +++. What is the most likely diagnosis? A: Henoch Schonlein purpura B: Mixed essential cryoglobulinaemia C: Minimal change nephropathy D: IgA nephropathy E: Membranous glomerulonephritis. Comment : Mixed essential cryoglobulinaemia will often present with palpable purpura on the legs and nephritis, but is an uncommon disease of older patients. If the man did not have a rash, then IgA nephropathy would the most probable cause of his urinary findings. A : 53. In a patient with hypertension, which one of the following features is consistent with a clinical diagnosis of Liddle's syndrome (a mutation affecting the sodium channels in the distal tubules)? A: raised catecholamine levels B: high aldosterone levels C: skin nodules D: high renin levels E: hypokalaemia. Comment : Liddle's syndrome is caused by a mutation in the sodium channel (ENaC) in the distal nephron. The mutation keeps the channel open, which has a similar effect to a raised aldosterone level. However, the aldosterone levels are not raised and may be lowered as a result of feedback from the sodium and water retention. There is increased sodium reabsorption and potassium loss. The condition is sometimes called pseudohyperaldosteronism. The hypertension is caused by sodium retention. This is a difficult question, but if you know that sodium handling is abnormal and there is hypertension, it is likely that this is because of sodium retention. In the distal nephron sodium reabsorption is mechanistically linked to potassium secretion, so increased sodium retention will lead to increased potassium secretion and hypokalaemia. E :

111

MOHAMMED IS-HAG

54. A 30-year-old man with no antecedent illness presents with severely raised BP (150/110 mm Hg), frothy urine, peripheral oedema and lethargy. There is +++ blood and ++++ protein on urinalysis. 24-hour protein loss is 6.6 grams. Plasma albumin is 28 g/L. Plasma C3 is 0.10 (low). Plasma creatinine is 145 umol/l. Which of the following renal lesions is most likely to be found on renal biopsy?

A: IgA nephropathy B: Lupus C: Focal sclerosing glomerulosclerosis D: Post infectious glomerulonephritis E: Mesangiocapillary glomerulonephritis. 55. Comment : This man has nephritic syndrome (hypertension, oedema, and proteinuria). Lupus is unusual in young men. FSGS is typically nephrotic syndrome, and commoner in older people. Post infectious Gn is possible but the infection should be clinically apparent, or in the history. MCGN presenting in a young man with nephrotic syndrome and hypertension and hypocomplementaemia would fit best. E : 56. A 31-year-old woman presents to her GP with marked leg oedema. Urine dipstick analysis shows 4+ Protein. Blood tests identify normal renal excretory function (creat. 68) and marked hypoalbuminaemia (albumin 19). A renal biopsy is reported as normal other than foot process fusion on the electron microscopy. Which of the following agents would be considered inappropriate as initial therapy? A: Cyclosporin B: Frusemide C: Simvastatin D: Warfarin E: Prednisolone. Comment : The diagnosis is minimal change glomerulonephritis, also known as minimal change nephropathy, minimal change disease, lipoid nephrosis and idiopathic nephrotic syndrome. Standard initial therapy is with corticosteroids, typically prednisolone at a dose of about 1mg/kg/day, which introduces remission in around 80 % of patients. Supportive therapies are also given, diuretics to clear oedema will clearly be appropriate in this case, and some nephrologists would prescribe warfarin to reduce risk of thromboembolism and a statin to reduce hypercholesterolaemia, although many would elect to see whether or not the patient went into a rapid remission that would render these agents unnecessary. Ciclosporin is useful as a steroid-sparing agent in patients who frequently relapse but would not be used as initial treatment. A : 57. A 53-year-old man presents having felt unwell for several weeks with general malaise and fatigue. He has had recurrent sinusitis for over a year and occasionally noticed a rash on his chest. Routine blood tests show that he has a serum creatinine of 180micromoles/l. Which of the following is the most likely diagnosis? A: Chronic renal failure B: Systemic vasculitis C: Minimal change nephropathy D: Urinary tract infection E: Focal segmental glomerulosclerosis. Comment : In any patient with a history of chronic progressive worsening of symptoms and renal impairment, it is important to consider systemic vasculitis as a possible cause. In this case, the rash and sinusitis are further pointers. Sinusitis can arise from involvement of the nasal tract and sinuses in the Wegener's pattern of the disease. Often a patient with systemic vasculitis will have a long history of indolent disease, but will then present late with severe aggressive disease. Useful immunological tests include assays for anti-neutrophil cytoplasmic antibodies (ANCA) and direct assays for antibodies against the ANCA antigens myeloperoxidase and proteinase 3. B : 58. In Hepatitis-C associated renal disease: A: patients can never safely receive a renal transplant once on dialysis B: membranous nephropathy is the commonest histological lesion C: cryoglobulinaemia and mesangiocapillary glomerulonephritis are typical D: treatment with interferon is ineffective E: immunosuppression is contra-indicated. Comment : Mesangiocapillary glomerulonephritis (GN) is commonest type of GN associated with Hepatitis C, though membranous GN can be seen.Interferon and other anti-viral agents can be very effective at reducing disease activity and cryocrit. Immunosuppression is usually given.The issue of patients on dialysis with Hepatitis C (commonest in Italy and Spain) who are being worked up for renal transplantation is important. Whether or not liver function tests are abnormal, these patients require estimation of viral load, viral genotyping, and a liver biopsy. If the disease is active, or the liver badly damaged, then pre-emptive anti-viral treatment is often employed before transplantation. Transplantation of a liver and a kidney can also be considered. C :

112

MOHAMMED IS-HAG

59. A 40-year-old man with a 25-year history of diabetes treated with insulin is found to have 2g proteinuria per 24 hours, haemoglobin 10.5 g/dl, albumen 30g/l, calcium 2.0 mmol/l, phosphate 1.8 mmol/l, creatine 250 µmol/l, cholesterol 7 mmol/l. He has significant peripheral oedema. Blood pressure is 135/85 mm Hg. HbA1C is 9.5%. In order to preserve native renal function, which of the following do you consider most important? A: Strict restriction of dietary phosphate B: Commence a loop diuretic C: Commence an HMG CoA reductase inhibitor D: Commence an angiotensin converting enzyme inhibitor E: Greatly improved diabetic control aiming for HbA1c < 7.2 %. Comment : There is clear evidence that ACE inhibition in diabetics delays progression of renal failure even if they are normotensive. A loop diuretic will improve the peripheral oedema. Cholesterol lowering is appropriate given this patient's vascular risk factors, but there is little evidence that this alters the progression of renal disease. D : 60. A 28-year-old man presents with the nephrotic syndrome. Renal biopsy reveals membranous nephropathy. Which one of the following statements is true regarding this condition? A: prognosis for renal survival is universally poor B: the commonest cause of nephrotic syndrome in children C: immunostaining on the renal biopsy is usually negative D: most patients present with the nephrotic syndrome E: renal vein thrombosis is a very rare complication. Comment : Membranous nephropathy is the commonest cause of the nephrotic syndrome in adults, whereas most children have minimal change disease. It has a very variable outcome with spontanous remission occuring in up to 1/3 of patients. Renal failure can develop in about 20% of patients. All the complications of the nephrotic syndrome are seen including renal vein thrombosis. The renal biopsy shows a thickened glomerular basement membrane on light microscopy, granular IgG and C3 on immunostaining and subepithelial deposits on electron microscopy. D : 61. A 22-year-old woman presents with recurrent symptoms of urinary frequency and dysuria associated with cloudy urine. These settle rapidly with courses of antibiotics. Which one of the following would NOT be a recognised strategy for trying to prevent recurrence? A: Long-term prophylaxis with trimethoprim 100 mg at night B: Regular drinking of cranberry juice C: Practice of double micturition D: Advice to void the bladder before and after sexual activity E: Use a shower rather than a bath. Comment : Cranberry juice has antiseptic properties and has been shown in a controlled trial to be effective at reducing risk of urinary tract infection. The practice of double micturition endeavours to make sure that the bladder is completely empty after voiding, making it more difficult for infection to take hold. The woman should be advised to empty her bladder, wait for a further 60 seconds on the toilet and then try to void again. Some find that pressing on their suprapubic region will help them to express more urine. E : 62. A 53-year-old man is found to have a blood pressure of 185/95 at a routine medical check. His general practitioner checks his electrolytes and renal function, which are normal, and then starts him on an angiotensin-converting enzyme inhibitor. A week later, the general practitioner repeats these blood tests and arranges to see the patient again. Which one of the following is NOT a possible direct consequence of starting an angiotensin-converting enzyme (ACE) inhibitor ? A: A rise in plasma potassium B: A rise in plasma creatinine C: A rise in renal sodium reabsorption D: A fall in glomerular filtration rate E: A fall in aldosterone production. Comment : Angiotensin II has a number of actions on the kidney. It is a vasoconstrictor, but its effect on the efferent arterioles is greater than that on the afferent arterioles, resulting in an increase in glomerular filtration rate. It also promotes sodium reabsorption in the proximal tubule by an action on sodium/hydrogen exchange at this site. Angiotensin II stimulates aldosterone release from the adrenal gland. Aldosterone promotes distal tubular sodium reabsorption and potassium secretion. ACE inhibitors block the effects of angiotensin II by inhibiting its formation. Therefore, glomerular filtration rate often falls slightly and aldosterone levels fall. The fall in aldosterone reduces sodium reabsorption in the distal tubule and the fall in angiotensin II reduces sodium reabsorption in the proximal tubule. Conversely, the fall in aldosterone reduces distal tubular potassium secretion, so potassium levels can rise. C :

113

MOHAMMED IS-HAG

63.

A 28-year-old woman was referred for the investigation of hypertension. Routine examination demonstrated that she had absent pulses in the left arm. She was a non-smoker. An arch aortogram was performed (see image). Which of the following statements is correct? A: She has coarctation of the aorta which is the likely cause of her hypertension. B: The selective picture shows a severe left subclavian stenosis most probably due to atherosclerosis C: The study is normal and she most likely has essential hypertension D: The selective picture shows a severe left subclavian stenosis most probably due to Takayasu's arteritis E: She has hypertension due to renovascular disease. Comment : Hypertension in the young always needs investigation. The physical findings in this woman were highly suggestive of a large vessel disease process. The selective angio picture does not show coarctation but does reveal critical stenosis of the left subclavian artery, which is most likely to be due to Takayasu's arteritis in this case. Atherosclerosis would be unusual in a young nonsmoker.Hypertension is probably driven by renal ischaemia due to either direct renal vessel involvement or indirectly by aortic narrowing (not seen on these pictures). Raised erythrocyte sedimentation rate (ESR) is typical of the active inflammatory phase of this illness; in Japan there have been reports of linkage with tuberculosis - steroids and other immunosuppressants can be effective. D : 64. In Fibromuscular Dysplasia: A: A single lesion is typical B: Onset is rare in patients younger than 50 C: Males are more commonly affected than females D: Cure of hypertension is not unusual after angioplasty-stenting E: No other arteries are affected.

114

MOHAMMED IS-HAG

Comment : Multiple lesions, typical in younger female patients. Cure is not guaranteed but can be seen frequently (as opposed to atherosclerotic disease). Iliacs, carotids and coronaries can be affected. D : 65. Which one of the following is not a recognized feature of renal disease due to myeloma? A: Renal amyloidosis B: Pre-renal acute renal failure C: Minimal change disease D: Cast nephropathy E: Nephrotic syndrome Comment : Myeloma can cause hypercalcaemia which acts on the tubules to cause sodium and water loss which can in turn result in pre-renal acute renal failure. AL amyloidosis can occur in the kidney and can cause nephrotic syndrome. Renal disease can also arise from the formation in the tubules of casts which contain immunoglobulin light chains. Alternatively, light chain deposition disease can occur with deposits of light chains visible on electron microscopy in glomerular and tubular structures. C : 66. Which one of the following statements about haemolytic uraemic syndrome is correct? A: It is usually associated with an abnormal clotting profile (PT and APTT). B: It can complicate treatment with cyclosporin. C: When associated with E Coli O157, it responds to antibiotic therapy. D: It responds to treatment with steroids. E: Immunosuppression is commonly used to treat this condition. Comment : Haemolytic uraemic syndrome (HUS) is characterized by microangiopathic haemolytic anaemia, normal clotting profile, and low platelets. The kidneys are often involved and presentation with acute renal failure is not uncommon. The most common form is seen following infection with E.Coli 0157. Immunosuppresive therapy plays no part in the treatment of this condition; however infusions of fresh frozen plasma (FFP) in combination with plasma exchange is routinely used. There is little evidence that antibiotics play a major role in the treatment of HUS associated with E.Coli 0157. Cyclosporin can cause HUS in renal transplant recipients, especially in those also receiving rapamycin which increases tissue concentrations of cyclosporin. B : 67.

The following ECG was obtained from a 46-year-old man. What is the diagnosis? A: Hypokalaemia. B: Hypercalcaemia. C: Hypothermia. D: Hypocalcaemia. E: Hyperkalaemia. Comment : This tracing shows the classic appearances of severe hyperkalaemia. There is peaking of the T waves, reduction in P wave voltage and broadening of the QRS complex. The rate is slowed and an ominous sine wave pattern is apparent which usually preceeds asystole.Urgent treatment is required. E :

115

MOHAMMED IS-HAG

68. Haemodialysis (HD) patients: A: have better short-term (patient) survival than peritoneal dialysis (PD) patients B: typically exhibit first signs of beta-2-microglobulin amyloid in their knees C: have better blood pressure control than peritoneal dialysis patients D: with diabetes have > 50% 10 years survival E: have much better blood pressure control if their dialysis treatment times are extended to eight hours each session Comment : It is largely a futile exercise to claim that PD or HD is better. They are different. In many series, the short-term survival on PD can be as good or better than on HD. Diabetics do badly on all types of dialysis, with greatly increased death rates due to cardiovascular disease (myocardial infarction, LVF, sudden death). Beta-2-microglobulin amyloid eventually affects any patient on dialysis long enough, for example 10 years. The wrists (carpal tunnel syndrome), shoulders and hips are the typical places to have symptoms and to see bone cysts. Very prolonged treatment times on HD, for example 8 hours thrice weekly, or daily HD, are associated with greatly improved blood pressure control because a negative salt balance can be achieved and maintained. E : 69.

PLATE 23 The image shows the fundus of a patient who had a grand mal seizure 2 hours previously resulting in admission to the accident and emergency department (see image). The blood pressure was high following the seizure (200/130 mmHg). The creatinine was found to be 670 µmol/l and the urea 28 mmol/l. The kidneys are of normal size and not obstructed on ultrasonography. Select the most appropriate statement. A: The likely cause of the seizure and renal failure is an overdose of aspirin B: The seizure was most probably due to insulin-induced hypoglycaemia C: The blood film is likely to show evidence of microangiopathic haemolysis D: The renal failure is likely to be acute, and due to rhabdomyolysis E: The seizure was probably due to uraemic encephalopathy. Comment : The fundus shows papilloedema and haemorrhages. Taken together with the blood pressure the most likely cause of the renal failure is malignant phase hypertension, which is usually associated with microangiopathy. This degree of biochemical disturbance would be unlikely to give rise uraemic encephalopathy and seizures without another factor. Two hours following a seizure is too early for rhabdomyolysis to induce this degree of acute renal failure. C :

116

MOHAMMED IS-HAG

70. You are talking to a 42-year-old man with end-stage renal failure and his wife regarding the possibility of renal transplantation. Which one of the following statements regarding renal transplantation is true? A: it is not ethical to allow donation except for donor-recipient pairs who are proven relations B: acute rejection rates of > 50% are typical C: the rate of lymphoma is increased compared to dialysis patients D: a rise in plasma creatinine in the first 2 weeks is invariably due to rejection E: delayed graft function beyond four weeks will always result in graft loss. Comment : Because of the increasing mismatch between cadaveric donors and needy recipients, many approaches have been devised towards expanding the donor pool. These include non-heart beating donation, and increasing the use of live-related and live-unrelated donors. In the situation of living donation, there is typically less good tissue-type matching than in cadaveric donation, but any disadvantage is heavily outweighed by the excellent quality of the donated kidneys (by design). With modern anti-rejection drugs (rapamycin, mycophenylate mofetil, tacrolimus), acute rejection rates are around 20%. Lymphoma is greatly increased, especially Epstein-Barr virus driven tumours. Early rises in plasma creatinine can be due to acute tubular necrosis (ATN), calcineurin toxicity, infection or rejection. C : 71. A 58-year-old man is reasonably fit and weighs 70 kg with known chronic renal failure due to reflux nephropathy. His plasma creatinine reading is 320 µmol/L; the value one year ago was 280 µmol/L. His potassium level is 5.6 mmol/L, urea 19.0 mmol/L, phosphate 1.8 mmol/L, calcium 2.25 mmol/L, haemoglobin 10.1 g/dL, ferritin 120 µmol/L. His appetite is good, but he feels tired. Blood pressure is 120/72 mmHg on an ACE inhibitor and he has 1.8g proteinuria per 24 hours. What treatment would you recommend? A: He needs to commence dialysis within the next few days. B: He should start oral alfacalcidol at 0.25 mcg per day in order to prevent renal bone disease. C: Treatment with recombinant human erythropoietin would probably be appropriate. D: This level of blood pressure is too low for a patient with chronic renal impairment and his ACE inhibitor should be reduced or stopped. E: His dietary protein should be restricted to in order to delay progression of his renal failure. Comment : There is no immediate indication for dialysis, and a good appetite is very reassuring in this context. He is likely to have significant hyperparathyroidism, due to hyperphosphataemia and reduced active vitamin D. However, commencing alfacalcidol will further increase his phosphate level, which is undesirable. A more appropriate course is to ensure that dietary phosphate (and potassium) intake is restricted, and to prescribe a phosphate binder (such as calcium acetate).Blood pressure control, preferably with an ACE (angiotensin converting enzyme) inhibitor or ARB (angiotensin receptor blocker) is the most important measure in delaying progression of renal disease. A reading of 120/72 is satisfactory and not too low. Restriction of dietary protein intake has not proved to be an effective or practical strategy in humans to prevent progression of renal disease.This patient would almost certainly benefit from rhEPO to correct his anaemia. The ferritin above 100 µmol/L makes iron deficiency unlikely. Other causes of anaemia (eg B12 deficiency, haemolysis, myeloma) should be considered, but at this level of renal function he will have relative EPO deficiency. C : 72. Which of the following is true with end-stage renal disease treated by haemodialysis? A : indigestion is best treated with aluminium containing antacids B: dialysis restores normal life expectancy C: parathyroid homorne (PTH) levels are often high D: sexual function and libido are usually normal E: sodium intake is not important. Comment : Aluminium can accumulate in end-stage renal disease, even with dialysis and it has been associated with dialysis-related dementia. For this reason, it is usual to avoid aluminium containing compounds when possible. Sexual function and libido are often impaired - women are usually infertile and man can have lowered testosterone levels with end stage renal disease. Sodium intake is important and is usually restricted. A high sodium intake can make patients thirsty, but if they drink to satisfy the thirst then they will become hypervolaemic. This is because they have no way of excreting this fluid between dialysis sessions. transplantation is necessary to maintain life in patients with end stage renal disease, but morbidity and mortality still remain higher than for the rest of the population. The prognosis is better with transplantation than with dialysis. Parathyroid hormone levels tend to rise in end stage renal disease for several reasons, including high phosphate levels which may be associated with low calcium levels and a deficiency of vitamin D, which is normally synthesised by the kidney. C :

117

MOHAMMED IS-HAG

73.

A patient on dialysis presents with a large swollen tender shoulder. Aspiration yields a milky slurry. Plain films show this appearance (see image). What is the most likely underlying cause of this problem? A: Poor control of calcium and phosphate B: Primary hyperparathyroidism C: Osteolytic metastasis D: Myeloma E: Septic arthritis. Comment : This is an example of tumoural calcinosis. Excess calcium and phosphate (which are at best only metastable in plasma) precipitate in either crystalline or amorphous forms in the skin, joints, blood vessels and other places. This ectopic calcification can cause local effects as here, including pain (though often these are painless) and deformity. Treatment is by engendering a negative calcium and phosphate balance, e.g. increased dialysis clearance, stopping positive calcium balance, and recourse to renal transplantation. A : 74. Which of the following conditions does not characteristically cause chronic interstitial nephritis when renal involvement is present? A: Chronic lead intoxication B: Primary Sjogren's syndrome C: Sarcoidosis D: Balkan nephropathy E: Scleroderma renal crisis. Comment : Renal involvement in scleroderma is a serious complication and the leading cause of death in this condition. Renal crisis is characterised by significant hypertension and rapidly deterioration renal function. The pathological findings are similar to those seen in accelerated hypertension often with an associated microangiopathy. All the other conditions are characterised by a chronic interstitial nephritis. Balkan nephropathy is a renal disease found in populations around the tributaries of the river Danube. The toxic agent is most likely a fungal toxin. E: 75. A 42-year-old woman presents with a plasma creatinine of 240micromoles/l and 3g of proteinuria per 24 hours. She weighs 200 kg. A renal biopsy is not possible because of her size. What is the most likely diagnosis? A: minimal change nephropathy B: focal segmental glomerulosclerosis C: membranous nephropathy D: Bartter's syndrome E: myeloma. Comment : There is a recognised association between severe obesity and focal segmental glomerulosclerosis. B :

118

MOHAMMED IS-HAG

76. Which of the following is not a well-recognised cause of renal disease due to multiple myeloma? A: Ig A nephropathy B: Cryoglobulinaemia C: Amyloidosis D: Hypercalaemia E: Light chain deposition disease. Comment : Renal disease is a common feature of multiple myeloma and many different pathologies have been identified. The most common reason is usually seen in the context of dehydration, sepsis and hypercalcaemia. High calcium can lead to enhanced light chain toxicity and tubular damage as well as inducing renal vasoconstriction. Excess light chains are responsible for several different pathologies including classic myeloma kidney (Cast nephropathy), renal amyloidosis (usually lambda light chains) and light chain deposition disease (usually kappa light chains). Cryoglobulinaemia is uncommon but may cause a membranoproliferative glomerulonephritis. IgA nephropathy is unrelated and is characterised by mesangial deposition of non-clonal IgA which is abnormally glycosylated. A : 77. Which of the following statements concerning patients with Anderson–Fabry disease is untrue? A: Angiokeratomas particularly affect the face B: It is inherited in an X linked fashion C: Females can be affected D: The defect is in lysosomal alpha-galactosidase A E: Painful peripheral neuropathy is a common feature. Comment : The incidence is estimated at 1:40 000. The angiokeratomas typically affect the ‘bathing trunk’ area, rather than the face. Although regarded as X linked recessive, it is now clear that female carriers can be severely affected. A : 78. In acute interstitial nephritis: A: proteinuria is typically in the nephrotic range B: eosinophils in the renal interstitium suggests a drug-allergy C: steroid therapy is unhelpful D: complete recovery of renal function is unusual E: secondary glomerular pathology is frequently seen. Comment : Acute interstitial nephritis usually presents with renal impairment. Systemic manifestations include fever, arthralgia and skin rash. Some cases are idiopathic, but the commonest cause is allergic reaction to drugs, typically penicillins or NSAIDs, but many other agents have been incriminated, including frusemide, cimetidine and allopurinol. B :

119

MOHAMMED IS-HAG

1. A 25-year-old woman attends A&E with a 48-hour history of unilateral headache of gradual onset associated with visual blurring and nausea. The headache is similar to her previous migraine, but examination reveals that her right pupil is larger than the left. Eye movements are normal and there is no diplopia. Which of the following statements is correct? A: If the left pupil fails to respond to light it is probably a Holmes–Adie pupil. B: Maintenance of the degree of inequality in different light intensities is suggestive of physiological anisocoria. C: She may have a right Horner's syndrome. D: A left Argyll-Robertson pupil is a likely explanation. E: The features are strongly suggestive of 3rd cranial nerve palsy caused by posterior communicating artery aneurysm. Comment : Minor discrepancies in pupillary diameters are common and usually represent physiological anisocoria, confirmed by the maintenance of the degree of inequality in different light intensities.A Holmes–Adie pupil is usually larger, and fails to react to light, with sluggish reaction to accommodation.Argyll–Robertson pupils are small, fail to react to light or accommodation, and may be unilateral as in this case; however since the cause of this condition is neurosyphilis, a condition now almost unheard of in the UK, this would be extremely unlikely in a woman of this age. When pupillary abnormalities are identified, always consider the possibility of a Horner's syndrome on the side of the smaller pupil - there will be ipsilateral partial ptosis and may be anhydrosis of the same side of the face. Third cranial nerve palsy due to compression by a posterior communicating artery aneurysm may cause dilation of the ipsilateral pupil; however it is uncommon for eye movements to be spared. B 2.

A 50-year-old heavy smoker is referred to the clinic with a six-month history of paraesthesiae in his fingertips, progressive weakening of grip strength in the upper limbs and general stiffness in his arms and legs. He is a manual labourer and often involved in heavy lifting. On examination, he has increased tone in the upper and lower limbs, power of 4/5 for finger flexion, finger extension and finger abduction and pathologically brisk reflexes with inverted C5/6 jerks and extensor plantar responses. An MRI of his cervical spine is shown (see image). What is the diagnosis? A: Cervical spondylosis B: Syringomyelia C: Epidural metastases from a primary neoplasm D: Transverse myelitis E: Acute disc herniation.

120

MOHAMMED IS-HAG

Comment : The MRI scan shows multilevel degenerative change in the cervical region. There is severe cord compression with myelopathic change due to osteophytic indentations at the C3/4 and C4/5 levels. Cervical spondylotic myelopathy is the most frequent cause of myelopathy in those over the age of 50. It usually develops insidiously, but may be associated with sudden acceleration, especially following a slight head and neck injury. Clinical findings include bilateral upper motor neurone weakness of the legs (paraparesis, paraplegia) or legs and arms (quadriparesis, quadriplegia), bilateral impairment of sensation with a 'level' separating a region of normal sensation from a region of impaired sensation (either spinothalamic-mediated or posterior column-mediated sensory modalities may be impaired), and bowel or bladder sphincter dysfunction. Sphincter dysfunction is usually preceded by motor or sensory findings. Less commonly, neck pain and Lhermitte's sign (electric shock-like sensation down the spine with neck flexion) may be present.Cervical spondylotic myelopathy is more common in congenitally narrow spinal canals where disc protrusion, osteophytes, ligamentum flavum hypertrophy, posterior longitudinal ligament ossification and vertebral body subluxations may all play a role in compressing the cord. Decompressive surgery is usually considered to be an effective treatment in severe cases or rapid progression, but treatment is less clear-cut in mild and moderate cases without rapid progression. Some patients improve with conservative treatment such as intermittent cervical immobilization with a soft collar, use of anti-inflammatory medications and intermittent bed rest in patients with pain. Many variables play a role in decompressive surgery, including the risk of operating on an irrelevant stenotic process (e.g. in patients with motor neurone disease) and surgery-related morbidity and mortality. In slowly evolving cases, the best timing for surgery is also difficult to determine. In a recently reported two-year prospective randomized trial of patients with slowly progressive mild and moderate cervical spondylotic myelopathy, surgical treatment did not show better results than conservative treatment over the follow-up period. In general, best surgical and non-surgical treatment results are seen in cases with mild neurological impairment that is present for less than six months, and when the age of the patient is below 70. A 3. A 24-year-old female with known type 1 diabetes mellitus is admitted via the accident and emergency department in a semi-concious state (Glasgow Coma Score [GCS] = 10). The admitting SHO documents her to be agitated and tachypnoeic with a respiratory rate of 36 breaths/minute. The staff nurse reports that the blood sugar checked with a bed-side stick test is ‘high’. In these circumstances: A: intravenous fluids must be given with caution until a chest radiograph has been undertaken to exclude pulmonary oedema B: a ventilation/perfusion (V/Q) scan should be arranged once the patient has been stabilized and adequately resuscitated C: broad-spectrum antibiotics should be commenced as soon as blood cultures have been taken D: intravenous fluids must not be supplemented with potassium chloride until the result of repeat electrolytes performed at 4 hours post-admission are available E: oral fluids should be with withheld. Comment : The picture here is one of moderate to severe diabetic ketoacidosis (DKA) – a condition that is still associated with significant morbidity and mortality. The tachypnoea almost certainly reflects the presence of a significant metabolic acidosis (with an attempt to compensate through ‘blowing off’ carbon dioxide - Kussmaul’s respiration), rather than primary lung pathology (although the latter may of course coexist and could have precipitated the acute episode). Patients with DKA are usually severely dehydrated (5-10L) and aggressive fluid resuscitation is a cornerstone of management – aim to give 5-6L within the first 24 hours, but bear in mind the clinical setting, for example a fit 24-year-old is likely to tolerate more aggressive fluid replacement than an elderly patient with a history of cardiac disease – consider central venous pressure (CVP) monitoring in the latter group. In either case, do not delay/withhold fluids whilst awaiting a chest radiograph (which may take several hours to obtain!). Unless there are specific clinical indications, a V/Q scan should not be routinely requested and broadspectrum antibiotics should not be routinely given – remember that the white cell count may be raised in DKA even in the absence of infection/sepsis. Although potassium levels are often normal or even slightly elevated at the point of admission, lifethreatening hypokalaemia may develop rapidly, especially in response to insulin (which ‘pushes’ potassium inside cells). Therefore, whilst it is reasonable (and usually advisable) not to add potassium to the first bag of fluid, thereafter supplementation should be instituted unless there are strong clinical indications to the contrary. In a semi-conscious patient the risk of aspiration is high (especially in the presence of delayed gastric emptying as is typically the case with DKA), and accordingly the patient should be made nil by mouth and a nasogastric tube sited. E

121

MOHAMMED IS-HAG

4.

PLATE 24

The wife of this 63-year-old man complained that he was sleepy and increasingly aggressive. Which of the following is LEAST likely to be helpful in his management? A: Non-invasive ventilation B: Salbutamol nebules C: Loop diuretics D: Long-term oxygen therapy (LTOT) E: 'Flu vaccination. Comment : This man with severe kyphoscoliosis has probably developed chronic respiratory failure. All patients with significant respiratory disease should be offered annual 'flu vaccination. He should be assessed for non-invasive ventilation and long-term oxygen therapy and is likely to have cor pulmonale with pitting oedema, which will be improved symptomatically with careful use of diuretics. He might have reversible airways obstruction (as this is a common condition,) but this would be an additional problem. B 5. A 48-year-old man is found to have a blood pressure of 176/112 when he attends his general practitioner for a ‘new patient check-up’. He takes occasional anxiolytics for anxiety, but his past medical history is otherwise unremarkable. Physical examination is normal, excepting for obesity (BMI 32). A ‘routine’ biochemical screen is normal, excepting for potassium 3.3 mmol/l. The two most likely causes of his hypertension are: A: Renal hypertension B: Hypothyroidism C: Renovascular hypertension D: Cushing’s syndrome E: Primary hyperaldosteronism (Conn’s syndrome) F: Acromegaly G: Essential hypertension H: Isolated clinic (‘white coat’) hypertension I: Phaeochromocytoma J: Coarctation of the aorta. Comment : All of the conditions listed, excepting hypothyroidism, might explain hypertension, but all other than essential hypertension and ‘white coat’ hypertension are rare (together accounting for less than 5% of cases). Although a secondary cause of hypertension is very unlikely it would be important to look for clues in history and examination that might suggest renovascular disease (ischaemic heart disease, transient ischmaemic attack (TIA) / stroke, peripheral vascular disease), renal disease (previous nephritis, results of urine testing for e.g. insurance / employment medicals). Episodes of palpitations, sweating or headache may suggest phaeochromocytoma, but a less exotic cause such as anxiety would be a much more likely explanation. The serum potassium concentration is just below the lower limit of normal, but primary aldosteronism (Conn’s syndrome) remains exceedingly unlikely. In the case of an obese man it is also important to note that the blood pressure reading may be falsely elevated as a result of inadequate blood pressure cuff size, and it would be important to ensure that readings were taken with appropriate equipment. GH

122

MOHAMMED IS-HAG

6. A 65-year-old male patient presents with a 3-month history of progressive difficulty in swallowing, and dysarthria. Examination reveals a weak, fasciculating, spastic tongue and a brisk gag reflex and jaw jerk. The likely diagnosis is: A: Basilar artery thrombosis B: Multiple sclerosis C: Miller–Fisher syndrome D: Amyotrophic lateral sclerosis E: Wernicke's encephalopathy Comment : Multiple sclerosis typically presents in the younger age group ( is eight to 48 hours.Treatment of salmonella gastroenteritis is symptomatic. If the patient is very ill, then ciprofloxacin is the antibiotic of choice, but antibiotics prolong the duration of stool carriage and should not be given routinely. C : 34. A 24-year-old man presents with a 6-month history of watery diarrhoea with blood, weight loss and arthralgia. The two most likely diagnoses would be: A: Bacterial overgrowth B: Crohn's disease C: Pancreatic carcinoma D: Antibiotic-induced diarrhoea E: Ulcerative colitis F: Caecal carcinoma G: Irritable bowel syndrome H: Giardiasis I: Tropical sprue J: Laxative abuse. Comment : While any of the conditions in the list can present as diarrhoea, the presence of blood in the stool and arthralgia is suggestive of inflammatory bowel disease. Bowel cancer can also present with diarrhoea and rectal bleeding but is much more rare in this age group. B : E : 35. Wilson's disease is: A: acommon inherited disorder of copper metabolism that responds to regular venesection B: arare disorder of copper metabolism that presents with cardiac and endocrine dysfunction C: associated with a Coomb’s negative haemolytic anaemia D: associated with a characteristic single amino acid substitution in the ATP binding cassette region of a the ATPB7 gene that is present in over 95% of caucasian patients caused by a relative deficiency of caeruloplasmin, which is the major serum copper binding protein. E: Comment : Wilson's disease is a rare autosomally recessively inherited disorder of copper metabolism associated with mutations in the ATPB7 gene,which appears to encode a membrane bound copper transporting ATPase.Many different mutations are found in various kindreds, and there is corresponding heterogeneity in the disease, which may present with progressive neurological dysfunction, particularly of the basal ganglia, insidious onset of hepatic cirrhosis,or with fulminant liver failure in younger patients.In cases of fulminant liver failure associated with Wilson's disease, Coomb's negative haemolytic anaemia is a characteristic clinical feature and may be the only clue to the aetiology of the disorder.Rare and minor cardiac abnormalities may also be present.Raised circulating levels of copper,increased excretion of copper in the urine,and reduced circulating caeruloplasmin levels are thought to be secondary features.Fulminant liver failure usually necessitates emergency liver transplantation, while chronic neurological damage may be limited by the use of copper chelating treatment with penicillamine or trientene. Oral zinc appears to limit the uptake of copper from the intestine, and may be beneficial.C: 36. Which of the following forms of acute viral hepatitis has a much higher mortality in pregnant than non-pregnant females? A: Hepatitis A B: Hepatitis B C: Hepatitis C D: Hepatitis E E: Hepatitis G. Comment : The mortality of hepatitis E in pregnant women, particularly in the third trimester, are of the order of 20-30%. It is transmitted by the faecal oral route like hepatitis A and epidemics have been associated with faecal contamination of water. There is no chronic phase of infection. Hepatitis G virus is currently not known to cause any human disease, not even hepatitis, despite its name. D : 37. A 56-year-old man is admitted with melaena, tachycardia and hypotension. Which of the following are true? A: Consider an urgent abdominal CT with contrast if he has previously had an abdominal aortic aneurysm repair. B: Intravenous omeprazole is indicated. C: If the ulcer base is clean, there is a 20% risk of re-bleeding. D: A normal haemaglobin level signifies a minor bleed. E: The overall mortality rate in the UK for upper gastrointestinal bleeding is about 5%. Comment : A suspected aorto-enteric fistula requires urgent investigation. If endoscopy reveals a clean ulcer base, the re-bleed rate is less than 5%. Stigmata of recent haemorrhage on endoscopy (fresh clot, visible vessel or active bleeding) are indications for intravenous omeprazole. These findings are also indications for endoscopic therapy. The overall mortality rate for upper gastrointestinal bleeding in the UK is 14%. A :

166

MOHAMMED IS-HAG

38. A 44-year-old man with a family history of diabetes is referred by his GP because of mildly elevated LFT's. He weighs 108Kg, and is 1.78m tall. His waist measures 168cm and his hips 140cm. Which of the following statements are FALSE? A: He is clinically obese. B: Obesity has a genetic component. C: He is at increased risk of developing diabetes. D: He most likely has an underlying endocrine disorder. E: He is at increased risk of developing cerebrovascular disease. F: He should be put on a strict 1000kcal/day diet. G: Hypothyroidism should be excluded. H: Calorie controlled diet should be combined with long-term low-intensity exercise. I: His abnormal LFTs may be due to fatty liver. J: Gastroplication should be reserved for extreme cases where other methods have failed. Comment : This man has a BMI of 34 with a WHR>1; by definition obese. Patients are at increased risk of a wide range of illnesses including diabetes and cardiovascular disease. The condition usually occurs due to a simple imbalance of energy input and expenditure and rarely has an underlying primary endocrine disorder. Management should aim for gradual weight loss with a 600kcal/day deficit and gentle long-term exercise program. Orlistat may be beneficial if added in to these treatment modalities. Surgery remains controversial and should be reserved for severe and resistant cases. D : F: 39. A 34-year-old lady, born in Hong Kong, is referred to clinic because at a hospital employment medical she was found to have hepatitis B. Her serology is as follows: HbsAg positive, HBe Ab positive. Liver function tests, full blood count and clotting profile were all normal. Her partner has the following serology: HBs Ab positive, HBc Ab positive. There is no family history of liver disease. Which of the following statements is correct? A: Pregnancy is contraindicated. B: She cannot perform exposure prone procedures without her viral load being measured. C: Her risk of developing hepatocellular carcinoma is high. D: She needs interferon treatment. E: She is at high risk of infecting her partner sexually with hepatitis B. Comment : She is likely to be a ‘healthy carrier’ of hepatitis B. Her risk of clearing hepatitis B spontaneously is 1% a year. This is not increased by interferon. Interferon is used to treat eAg positive individuals with high serum transaminases and in this setting causes seroconvertion to e Ab, a fall in viral load and normalisation of liver biochemistry in about 30% cases. As a young woman, with a normal liver and e Ab (normally corresponds to a low viral load of 100,000 mixed bacteria / ml in a duodenal aspirate. Small bowel enema will detect diverticula or strictures as a cause for overgrowth.Initial treatment is with 10 days of augmentin or tetracycline. Prolonged courses may be needed, and cyclical antibiotics for one week every four weeks may be used. Laparotomy / resection would only be contemplated if these manoeuvres failed, and if anatomical abnormality was confined to a relatively small segment of bowel. A : 93. A patient with progressive bulbar palsy is referred for placement of a percutaneous enterogastrostomy tube for feeding. Which of the following is correct? A: There is a significant risk of bleeding, infection, and pain following the procedure, and a substantial risk of procedure-related mortality. B: The procedure requires a general anaesthetic and is most efficiently performed by means of a small laparotomy. C: The procedure is ideally performed under radiological guidance, which obviates the need for endoscopy and sedation. D: The main advantage for this patient would be the reduced risk of aspiration of gastric contents. E: In view of the likelihood that the patient has a neurological condition such as motor-neurone disease, blood gas measurement should be obtained before the procedure is undertaken. Comment : PEG tube placement is often requested from the gastroenterology and endoscopy service because endoscopic placement of the tube obviates the need for general anaesthesia and surgery, and because the durability of endoscopically-placed tubes is greater than that of radiologically-placed tubes. However, there is a significant risk of complications, including life-threatening bleeding and infection. This should be explained to the patient and their carers as part of the informed consent process. Although bypassing the need to swallow may reduce aspiration caused by disordered swallowing, a PEG tube does not prevent regurgitation of gastric contents, or their aspiration into the larynx, trachea, and bronchial tree. It is quite correct to be wary of respiratory difficulties occurring in patients with neurological disorders who undergo endoscopy. However, normal blood gas measurements could provide false reassurance – the critical measurement is the patient’s ventilatory reserve, best determined by measuring their vital capacity. A : 94. A 30-year-old man presents with progressive central abdominal pain and vomiting associated with significant weight loss (five stone in 3 months).He gives a history of binge drinking and depression, and smokes twenty cigarettes per day. Because eating provokes abdominal pain and vomiting, he has eaten virtually nothing for a month. CT scanning of his abdomen showed a normal pancreas but dilated loops of small bowel with a possible terminal ileal stricture. His albumin level was 20 and C-reactive protein level was 50.Which statement is NOT true? A: His symptoms are consistent with a Crohn’s stricture. B: Chronic pancreatitis is very unlikely in view of the normal CT. C: Infliximab should be prescribed as soon as possible. D: Regular monitoring of his electrolytes is indicated in view of a risk of refeeding syndrome. E: He should be informed of a risk of a stoma if surgery is indicated. Comment : This gentleman had two strictures at operation (right hemicolectomy) which were due to Crohn’s disease.The best treatment for such a patient is surgical and not medical. There is no clear evidence to suggest that post-operative medical therapy reduces the risk of recurrent Crohn’s disease necessitating surgery, although typically Pentasa and/or immunomodulators such as Azathioprine are prescribed. However, abstinence from smoking should be encouraged as the rate of recurrence is reduced by approximately 50% in non-smokers.In view of his recent near-total starvation and significant weight loss, refeeding syndrome is a possibility; careful attention to his electrolyte levels, gradual reintroduction of feeding, and replenishment of B vitamins is required. All patients undergoing colonic surgery need to be aware of the possibility of stoma formation, even if, as in this case, it is unlikely. C :

182

MOHAMMED IS-HAG

95. A 46-year-old man presents with jaundice. Dipstick testing of his urine reveals the presence of bilirubin but no urobilinogen. This means that: A: jaundice is likely to be pre-hepatic B: bile must be flowing freely into the gut C: jaundice cannot be obstructive D: renal function is normal E: there must be complete obstruction to bile flow. Comment : When haemoglobin is broken down the porphyrin ring is converted into biliverdin and thence to bilirubin.Unconjugated bilirubin is relatively insoluble and is transported in the blood as a complex with albumin. It is not excreted in the urine.Hepatocytes take up unconjugated bilirubin and conjugate it to form the soluble diglucuronide, which is excreted into the bile.Further metabolism by gut bacteria forms the soluble colourless compound urobilinogen. Some of this enters the blood stream and is excreted in the urine. Urobilinogen remaining in the gut is converted to the brown pigment, urobilin, and is excreted. If a patient with jaundice has no urobilinogen in their urine, then they must have complete obstruction to bile flow. E : 96. A 55-year-old man is still anaemic, has some loose stools and has failed to regain weight several months after starting a gluten-free diet for coeliac disease. Which of the following is the commonest cause of failure to respond to treatment? A: Pancreatic insufficiency B: Dietary non-compliance C: Small bowel adenocarcinoma D: Enteropathy associated B cell lymphoma of the small intestine E: Ulcerative jejunitis. Comment : Poor dietary compliance accounts for most cases where there is a failure to improve on treatment. Ulcerative jejunitis, small bowel adenocarcinoma and an enteropathy-associated T cell lymphoma are recognised but rare complications and there is an association with pancreatic sufficiency. B : 97. A 40-year-old female nursing home resident recently in hospital with Salmonella gastroenteritis is readmitted with further diarrhoea. In her management plan, which statement is LEAST appropriate? A: Rehydrate orally or intravenously depending on clinical and biochemical assessment. B: Nurse her in isolation. C: X-ray her abdomen and measure inflammatory markers such as ESR/CRP. D: Send stool on multiple occasions for culture and sensitivity, including C. difficile toxin. E: Request a colonoscopy as an inpatient. Comment : Not all episodes of infectious diarrhoea need treating with antibiotics, and often, resistance to certain antibiotics (e.g. ciprofloxacin) leads to sub-optimal treatment. Any patient who has been in hospital recently is at risk of C. difficile infection - especially if they may have had antibiotics. This needs to be actively tested for and treated appropriately (metronidazole as first-line treatment, then oral vancomycin). Hence, it is imperative in managing patients with diarrhoea to ensure adequate and suitable stool samples are sent early in the illness. Colonic imaging is usually not indicated in infective colitis (other than a simple X-ray to exclude toxic megacolon) and is only necessary if symptoms persist and stool cultures are negative. E : 98. Which of the following is true of anorectal abscess: A: It is more common in males B: It is most commonly caused by Crohn’s disease C: It is typically caused by infection of minute cracks in the peri-anal skin D: It is more common in those who practice receptive anal intercourse E: It usually responds to antibiotic treatment. Comment : Anorectal abscesses generally develop from infections of the anal glands, which lie in the intersphinteric space between the internal and external anal fissure, and drain into the anal canal via a duct that opens onto the dentate line. The condition is not associated with any sexual practices, although other peri-anal conditions, such as ulcers and warts are, but for unknown reasons, it is more common in men. Crohn’s has a prevalence of about seven per 100 000, and about a third of cases may involve the anorectal region with abscesses and fistulae. However anorectal abscess are more common. Most abscesses require incision and drainage, with or without antibiotic cover. There is a risk of persistent fistula formation after the abscess has drained. A : 99. Regarding Crohn’s disease: A: 5 aminosalicylic acid (5-ASA) compounds reduce the rate of relapse by about 30%. B: Finger clubbing is a recognised feature. C: Unlike ulcerative colitis, there is only a small genetic risk of developing the condition. D: Peri-anal inflammation is almost universally present. E: Patients should be encouraged to stop smoking, although this may increase the risk of relapse.

183

MOHAMMED IS-HAG

Comment : Crohn’s disease can affect any part of the intestinal tract, and three forms predominate: terminal ileal disease, colitis, and peri-anal Crohn’s. Peri-anal disease is the most infrequent (about 25%). Crohn’s is frequently associated with a positive family history (in up to 15% of patients), and the genetic component is probably greater than for ulcerative colitis. Smoking increases the risk of relapse, and 5ASA compounds only have a modest impact on acute or chronic disease. B : 100. A 34-year-old man is referred because of abnormal liver function tests (LFTs). Which of the following are true? A: An ALT:AST ratio of over 2.0 is suggestive of alcohol as a cause. B: Raised IgA is not uncommon in alcoholic liver disease. C: An AST of over 500 IU/l is typical of acute alcoholic hepatitis. D: Hepatis steatosis would rarely be the cause. E: Non-alcoholic steatohepatitis (NASH) invariably has a benign course. F: Obesity and type 2 diabetes are factors commonly associated with NASH. G: Mallory’s hyaline does not occur in NASH. H: Steroids are indicated if liver histology shows active inflammation in NASH. I: A raised ferritin almost always indicates genetic haemachromatosis (GH). J: Over 90% cases of GH in the United Kingdom are C282Y homozygotes. K: A low iron diet is mandatory in GH. L: GH has a low risk of complicating hepatoma compared to other causes of cirhosis. M: Negative anti-hepatitis C antibody assay excludes this as a cause. N: Primary biliary cirrhosis is not the cause in a male patient. O: Primary sclerosing cholangitis should be suspected if the patient has coeliac disease. Comment : An AST:ALT ratio of over 2.0 is often found in alcoholic hepatitis, with an AST rarely above 400IU/l. Hepatic steatosis (or fatty liver) is a common cause of mild LFT abnormality. When active inflammation is present (non-alcoholic steatohepatitis), as many as 40% may progress histologically and some may develop cirrhosis, particularly those with evidence of balloon degeneration, Mallory’s hyaline or fibrosis. Some improvement in LFT can be seen following efforts to lose weight and exercise. Ferritin is an acute phase protein, so is often raised in alcoholic hepatitis. All suspected genetic haemochromatosis (GH) patients should be genotyped: 93-95% are C282Y homozygotes. Venesection of one unit of blood removes 250mg iron and maximum absorption is 1mg per day, so dietary intervention is limited to avoiding excess iron. Hepatoma most commonly complicates cirrhosis due to GH; hepatitis B and C. Polymerase chain reaction may detect hepatitis C RNA in the presence of negative hepatitis C serology in acute infection or in the presence of immunodeficiency/immunosuppression. Primary sclerosing cholangitis is associated with ulcerative colitis. B : F:J: 101. A 47-year-old woman is referred with iron deficient anaemia. She has no gastrointestinal symptoms. Which of the following are true? A: Coeliac disease is very unlikely in the absence of symptoms. B: Raised serum anti-gliadin antibody levels are 95% sensitive for the diagnosis of coeliac disease. C: The prevalence of coeliac disease in the UK approaches 1 in 200. D: Howell Jolly bodies in the blood film suggest haemolysis. E: Antibody profiles take at least 1 year on a strict gluten-free diet to become negative. F: Osteoporosis is a recognized complication of coeliac disease. G: Coeliac disease patients with ulcerative jejunitis tend to be at lower risk of malignancy. H: Barium follow-through invariably shows increased flocculation of barium and coarser mucosal folds. A single biopsy is adequate for the diagnosis of coeliac disease provided it is obtained distal to the I: duodenal bulb.

J: Pancreatic insufficiency is the commonest cause of failure to improve on a gluten-free diet. Comment : Anaemia is a common presentation of coeliac disease. Raised anti-endomysial antibodies have a 95% sensitivity for the diagnosis of coeliac disease, which affects up to 1 in 200 of the UK population. Howell Jolly bodies are a feature of hyposplenism which is associated with coeliac disease. Barium studies may be normal and as the histological changes are patchy, four biopsies are advised. Endomysial antibody levels usually fall within 6 weeks of dietary treatment and non-compliance is the commonest reason for an absence of clinical or serological response. Osteoporosis is a common complicating factor and improves following institution of the diet. Ulcerative jejunitis is associated with an increased risk of enteropathy associated T cell lymphoma. C:F: 102. Which of the following statements concerning liver transplantation is correct? A: Donor and recipient should have three matches on human leukocyte antigen (HLA) tissue-typing B: Is most commonly performed for cases of acute liver failure C: Overall one-year survival rates are approximately 60% D: HIV infection is not necessarily a contra-indication to transplantation E: Xenotransplantation is widely used in Japan. Comment :Donor and recipients are matched on the basis of blood group and body size - unlike renal transplantation, HLA matching is not required. Approximately 15% of liver transplants are performed for

184

MOHAMMED IS-HAG

acute liver failure, the remainder are performed in patients with chronic liver disease.Overall one-year survival rates currently run between 80 and 95%. One-year survival rates are lower in those patients transplanted with acute hepatic failure, but these only form the minority of transplants.Improvement in prognosis for HIV-infected patients as a result of new anti-retroviral therapies mean that under exceptional circumstances HCV co-infected patients may be considered for transplantation. Xenotransplantation (the use of genetically modified animal donors) remains a theoretical advance in transplantation and at the present time, no human trials have taken place. Religeous objection to organ donation in Japan means that the majority of liver transplant grafts are obtained from living donors. D : 103. A 35-year-old woman presents with abdominal pain and vomiting. She has been unwell for about six weeks with anorexia, nausea and weight loss. Examination reveals a tender palpable mass in the right iliac fossa. The two most likely diagnoses are: A: Psoas abscess B: Tuberculosis C: Abdominal secondaries D: Crohn’s disease E: Campylobacter F: Caecal carcinoma G: Hypernephroma H: Appendix mass I: Ovarian carcinoma J: Lymphoma. Comment : The presentation with gastrointestinal symptoms occurs on a background suggesting a systemic component to the illness, hence the most likely diagnoses are an inflammatory mass associated with Crohn’s disease or an appendix abscess. In an older patient caecal carcinoma would be a more likely explanation.Aside from assessment of cardiovascular status (is resuscitation required?) and for the presence of peritonism (which would suggest a perforated viscus and the need for laparotomy), examine carefully for signs that would support a diagnosis of Crohn’s disease. These include clubbing, aphthous ulceration, perianal skin tags / ulceration / fistulae and (less commonly) seronegative arthritis, sacroiliitis, iritis and skin rashes (erythema nodosum, pyoderma gangrenosum). D : H: 104. A 37-year-old building labourer presents with heartburn and acid regurgitation. Which of the following statements are true? Oesophagogastroduodenoscopy (OGD) is required in order to diagnose gastro-oesophageal reflux disease A: (GORD).

B: C: D: E: F: G: H: I:

First-line management should be a proton-pump inhibitor. He should be encouraged to drink milk at bedtime. It is essential to eradicate Helicobacter pylori for optimal symptom control. The patient should be encouraged to stop smoking. A surgical corset may improve posture and hence symptoms. Oesophageal pH monitoring may be useful in managing drug-resistant cases. All patients with Barrett’s oesophagus should be entered into a surveillance programme. There is an increased incidence of oesophageal cancer in patients with a long history of gastro-oesophageal reflux disease.

J: Surgical fundoplication optimises symptom control and is the treatment of choice in the majority of cases. Comment :GORD is a clinical diagnosis based on history. Only a minority of patients with GORD will have endoscopic oesophagitis, and a normal OGD does not exclude the diagnosis. First-line management should be lifestyle advice, followed by simple antacids. A proportion of patients will not require a proton-pump inhibitor.Although milk has a short-term alkali effect, its fat content and presence in the stomach will stimulate gastrin and acid production. Patients should be advised to eat or drink nothing for three or four hours before retiring. Belts and corsets increase intra-abdominal pressure and will exacerbate symptoms, so they should be discouraged. Not only does nicotine relax the lower oesophageal sphincter, but also smokers produce less saliva, one of the clearing mechanisms for transient lower oesophageal sphincter relaxations.H. pylori has no role in GORD, and indeed some studies suggest eradication may worsen symptoms. Oesophageal pH monitoring is not sufficiently available for routine diagnosis, but it may be useful in confirming the diagnosis in apparently resistant cases. The debate over Barrett’s surveillance continues, though none would argue for it in short-segment disease or the elderly.While in patients with oesophageal cancer there is an increased history of reflux symptoms, the reverse is not true. Any association has yet to be proven. While surgical (or endoscopic) fundoplication may be beneficial in a small number of drug-resistant cases, there is significant morbidity (gas bloat, flatulence etc) and mortality. Symptoms also frequently recur over time requiring a return to medical therapy. E :G:

185

MOHAMMED IS-HAG

105. A 43-year-old woman has been diagnosed as suffering from bronchiectasis on a highresolution computed tomography (HRCT) scan of the lung. Which one of the following statement is NOT true? A: Her immunoglobulin levels should be checked. B: She is at risk of developing a pneumothorax. C: Massive haemoptysis is the commonest cause of death in her age group. D: She is at risk of developing brain abscess . E: Recurrent chest infections are unlikely at this age. Comment : Congenital and acquired immunodeficiency are well known causes of bronchiectasis, hence serum immunoglobulins should be checked, particularly the IgG subclass (IgG1,2,3 & 4). Patients with immunoglobulin deficiency should be treated with regular immunoglobulin infusion.Pneumothorax, recurrent chest infections and brain abscess are well recognised complications.Haemoptysis is a common symptom but rarely causes death. C : 106. A 54-year-old Caucasian gentleman is referred with mildly abnormal liver function tests (ALT 74 U/l [5-45], AST 63 U/l [5-40]). He has been complaining of tiredness and aches and pains in his knees and his GP therefore tested his full blood count (FBC), ferritin and sugar which show: Hb 15.5g/dl [12.5-15.5], ferritin 3452 micrograms/litre [12-200], fasting glucose 13.2 mmol/l [160 ms is associated with risk of arrhythmias. Always check for other poisons in cases of polypharmacy overdose. Also check arterial blood gases for signs of hypoventilation and acidosis. C : 11. A 25-year-old man with a history of bipolar affective disorder and opiate abuse is brought into A&E by ambulance. His GCS is 9/15. He smells strongly of alcohol and has pinpoint pupils. Which of the following statements about his management is NOT correct? A: It is essential to check his BM stix and give dextrose if this is low. B: He should be given naloxone 400mcg iv. C: Treatment with 50g of activated charcoal is not indicated. D: Gastric lavage is not indicated. E: There is no benefit in taking blood for paracetamol and salicylate levels. Comment : Basic management should always include maintenance of airway, fluid resuscitation where appropriate and correction of hypoglycaemia. He is known to have a history of opiate abuse and has a reduced GCS and pinpoint pupils so it would be reasonable to give naloxone iv. This is the correct dose and can be repeated as often as necessary. Activated charcoal does not adsorb lithium, opiates or alcohol. Gastric lavage has not been shown to be of benefit, indeed there is some evidence that it increases absorption by forcing the drugs through the pylorus. E is not true. Overdoses are usually of multiple medications and this should be done. E : 12. A 45-year-old patient with a past history of alcohol-related chronic liver disease presents to A&E following a 400ml fresh haematemesis. On examination he is jaundiced with palmar erythema and marked ascites. Pulse is 120 beats per minute and blood pressure 100/70. In addition to fluid resuscitation, which of the following treatments is most likely to be beneficial in his initial management, while awaiting upper GI endoscopy? A: Ranitidine 50mg intravenously B: Omeprazole 40mg intravenous bolus C: Tranexamic acid 1g intravenously D: Terlipressin 2mg intravenous bolus E: Propranolol 40mg orally. Comment : It is likely that this patient is bleeding from oesophageal varices, resulting from alcoholinduced portal hypertension. Terlipressin reduces the likelihood of continued bleeding by reducing portal pressure, and may be helpful either prior to endoscopy, or as an adjunct to endoscopic therapy. Intravenous omeprazole has been shown to reduce the likelihood of peptic ulcer rebleeding after endoscopic therapy, but there is no evidence to support its use as an empirical therapy prior to endoscopy or in the management of variceal haemorrhage. Tranexamic acid is an antifibrinolytic and

303

MOHAMMED IS-HAG

has been shown to provide a slight reduction in mortality following peptic ulcer haemorrhage in one meta-analysis, while intravenous ranitidine has no impact on outcome of upper GI bleeding from any source. Propranolol is helpful in the primary and secondary prevention of variceal haemorrhage but has no role in the acute setting. D : 13. A 26-year-old woman presents with a week's history of progressive numbness and weakness in her lower limbs. Which of the following suggests a diagnosis of Guillain-Barré syndrome (GBS)? A: Optic atrophy on fundoscopy B: A sensory level C: Ankle weakness with saddle area sensory loss D: Autonomic dysfunction E: Proximal weakness > distal. Comment : Optic atrophy suggests a diagnosis of demyelination. Clear-cut sensory levels classically occur with cord compression. Ankle weakness with saddle area sensory loss would point towards a cauda equina lesion (always examine the saddle area!). In Guillain Barré syndrome there are often cardiovascular abnormalities reflecting autonomic dysfunction. Indeed the autonomic dysfunction can result in sudden death and can manifest as extreme hyper or hypotension, tachycardia and bradycardia. In GBS the weakness is greater distally. D : 14. An elderly man is brought into A&E by ambulance because he had been found wandering down his street early in the morning. He gives a fluent history of his past life, but is unable to explain what he had been doing. On examination he smells of alcohol. He has nystagmus and bilateral lateral gaze palsies. Which of the following statements is incorrect? A: The lesions are in the mamillary bodies and thalamus. B: His red cell transketolase is low. C: Examination of his pupils is normal. D: All of his deficits will resolve with 3 days of parenteral thiamine. E: A CT scan of his head is likely to be normal. Comment : This man has Wernicke–Korsakoff syndrome. It is caused by thiamine deficiency and is common in alcoholic patients. However, it should be considered in all patients with malnutrition. The classical presentation is of confusion, ataxia, ophthalmoplegia and nystagmus. The neurological signs do tend to improve with 3 days of parenteral thiamine, but there are often residual memory problems. CT of the head is likely to be normal. An MRI may show evidence of neuronal loss and demyelination in the midbrain structures. D : 15. A 47-year-old man has re-bled on the ward following emergency endoscopic treatment of his oesophageal varices. In relation to Sengstaken–Blakemore tube placement, which of the following statements is incorrect? Inflation of the oesophageal balloon is likely to be required if bleeding is originating above the cardia. A: B: Formal airway protection with the insertion of a cuffed endotracheal tube with appropriate anaesthesia is recommended if the patient has grade II encephalopathy. If the oesophageal balloon is inflated, pressure should be released for up to 5 minutes every 30 minutes. C: D: Aspiration ports should be left to drain freely and aspirated regularly. E: Gastric aspiration port may be used for oral medication. Comment : Since oesophageal varices usually originate in the stomach, tamponade of the varices at the gastrooesophageal junction by inflation of the gastric balloon is usually effective in controlling haemorrhage. Inflation of the oesophageal balloon is rarely required and risks necrosis of the oesophageal wall, necessitating regular reduction of the pressure. Regular aspiration of the appropriate ports prevents build-up of secretions; the gastric aspirate port may be used as a nasogastric tube for delivery of essential oral medication, such as lactulose for the treatment of encephalopathy. Aspiration pneumonia is a major cause of death and significant morbidity in these patients; the risk is particularly high in confused or encephalopathic patients who should be intubated for airway protection prior to insertion of the tube. A : 16. A 35 year-old women presented 2 hours after collapsing at home with severe headache. On examination she was drowsy and had neck stiffness. Her temperature was 37.5. She had a mild right hemiparesis. Which of the following is the most appropriate first diagnostic investigation? A: CT brain scan B: Lumbar puncture and examination of the cerebrospinal fluid C: Four-vessel angiography D: MR angiogram E: MR brain scan. Comment : The history is suggestive of a subarachnoid haemorrhage. Urgent CT brain scan will identify more than 95% of patients with suspected subarachnoid haemorrhage if performed within 1-2 days after headache onset (van Gijn 1982). Lumbar puncture is potentially dangerous and will add no extra information if brain CT shows definite evidence of extravasated blood. If CT is negative and there

304

MOHAMMED IS-HAG

are no contraindications, LP should be performed. Four vessel angiography will be needed later once the patient is stabilised to identify the source of the bleeding and surgical or endovascular treatment. In the acute stages of SAH (because of the speed of investigation and availability) CT imaging is superior to MRI. MRI imaging becomes much more useful if presentation is delayed after the first few days (>4) when CT sensitivity for subarachnoid blood rapidly declines.MR angiography is without risks and reasonably sensitive (90%) – useful for screening people at risk of intracranial aneurysms but less suitable for patients with subarachnoid haemorrhage. A : 17. A 70-year-old man presented with increasing shortness of breath. He had suffered a myocardial infarction complicated by arrhythmias two years earlier. Investigations revealed: Ward oximetry: O2 saturation 84% on air; Chest radiograph: Bilateral patchy infiltration of lung fields, cardiothoracic ratio of 20/31 cm. No evidence of pulmonary oedema. Which one of the following drugs he was prescribed might explain these findings? A: Aspirin B: Amiodarone C: Lisinopril D: Furosemide E: Atorvastatin. Comment : Amiodarone, an antiarrhythmic drug, can cause pulmonary toxicity – an acute alveolitis that may or may not be followed by fibrosis. It is estimated that approx 6% of patients taking 400mg or more per day for 2 months or more will develop overt pulmonary toxicity. The mechanism may include both immunologically mediated and direct toxic effects. Radiographic appearances are varied: most frequently there is a diffuse nodular or alveolar pattern sometimes with upper lobe predominance. Drug cessation will normally be required. B : 18. A 38-year-old man presented with a short history of increasing lethargy and 3 days of irregular palpitations. Investigations revealed: serum sodium 142 mmol/L (137-144) serum potassium 7.9 mmol/L (3.5-5.0) serum bicarbonate 10.0 mmol/L (20-28) serum creatinine 550 µmol/L (60-110) What is the best immediate therapy? A: Nebulised salbutamol B: Intravenous calcium gluconate C: Intravenous sodium bicarbonate D: Intravenous insulin and dextrose E: Oral calcium resonium Comment : This man has dangerous hyperkalaemia resulting from acute renal failure. Although there is no mention of ECG changes, the history of palpitations should raise the possibility of arrhythmias. As such, 10mL 10% calcium gluconate i.v. will immediately reduce myocardial excitability. This should prevent any dangerous cardiac arrhythmias. It does not reduce plasma potassium levels. All the other agents could be used to lower potassium whilst arrangements for urgent dialysis are made. B : 19. A 45-year-old man with chronic alcoholic liver disease was admitted earlier in the day following a large haematemesis. He was treated with intravenous terlipressin and urgent endoscopy was arranged after initial fluid resuscitation and correction of a mild coagulopathy with fresh frozen plasma. Endoscopy revealed bleeding oesophageal varices, which were injected with sclerosant with apparently good effect. He has been stable on the ward for the past 6 hours, but you are called to see him in the early hours of the following morning because he has had a further 500ml haematemesis, and his blood pressure has dropped from 130/90 to 90/50. Which of the following measures would be most appropriate in addition to fluid resuscitation? A: Urgent repeat endoscopy and sclerotherapy B: Change terlipressin to octreotide C: Administer 10mg vitamin K intravenously D: Insert sengstaken tube, inflate gastric balloon and apply traction E: Urgent surgical intervention. Comment : Balloon tamponade is the most effective treatment for control of variceal bleeding if endoscopic therapy has failed. Even though the varices were bleeding from within the oesophagus, inflation of the gastric balloon with application of traction usually stops the bleeding by compressing the vessels as they cross the gastrooesophageal junction. Rarely, inflation of the oesophageal balloon may be required. It is unlikely that repeat endoscopy would prevent on-going bleeding if he has already received successful sclerotherapy, unless there was doubt about the source of bleeding previously. Octreotide is less effective than terlipressin in controlling portal pressure, and therefore switching therapy is unlikely to be helpful. Although the effects of the fresh frozen plasma administered earlier will probably have worn off, vitamin K will probably not help as the coagulopathy is probably due to synthetic dysfunction rather than vitamin K deficiency. Surgical transection of the oesophagus is rarely indicated, and has largely been superseded by radiological shunt insertion (e.g. TIPSS). D :

305

MOHAMMED IS-HAG

20. You are called to see a 78-year-old man who is deteriorating rapidly following surgery for gallstones. He suffers from hypotension, pyrexia, tachypnea and falling urine output. Which of the following statements about severe sepsis do you agree with? A: Severe sepsis accounts for 10% of all critical care admissions in England, Wales and Northern Ireland. B: No drugs are recommended by NICE for the treatment of severe sepsis. C: Severe sepsis kills almost half the patients affected. D: Drotrecogin alpha (activated ) is a recombinant IL-6 inhibitor. E: If the patient survives severe sepsis, sequelae are rare. Comment : Severe sepsis accounts for over a quarter of admissions to critical care. Despite advances in critical care, almost half of those patients die and those who survive often suffer permanent organ damage. Drotrecogin alpha (activated) has been recommended by NICE for treatment of adult patients with severe sepsis resulting in the failure of two or more organ systems. It is a recombinant activated protein C, but its main mechanism of action clinically appears to be anti-inflammatory. C : 21. A 25-year-old woman presents to A&E 1 hour after consuming 28 x 500mg paracetamol tablets. Which of the following statements is true? A: If the INR is normal on a sample taken four hours from the time of ingestion, liver damage is unlikely to occur. B: Alcohol ingestion at the time of consumption of paracetamol is an indication for N-acetylcysteine treatment if paracetamol level at 4 hours exceeds the ‘high-risk’ line. C: Activated charcoal may be beneficial if given immediately. D: Onset of tinnitus may be an early symptom of liver failure. E: Deterioration in conscious level within the first 24 hours usually suggests hepatic encephalopathy. Comment : Abnormal blood clotting following paracetamol overdose results from loss of production of clotting factors by hepatocytes, and is therefore a good early marker of synthetic liver function. The INR rises first because Factor VII has the shortest half-life and is therefore the first to disappear from the blood. However it is unusual to see any abnormality in blood clotting less than 18 hours from ingestion, so normal INR at 4 hours is unhelpful. Abnormal INR at the time of admission may indicate prior chronic liver disease, warfrin ingestion or suggest that ingestion of the drug occurred earlier than the patient reports. Because acute alcohol intoxication inhibits liver enzyme function, less paracetamol will be metabolised to the toxic metabolite, so that liver damage is, if anything, less likely. Chronic alcohol abuse causes liver enzyme induction and is an indication for treatment with N-acetyl cysteine if paracetamol level exceeds the ‘high risk’ line. Activated charcoal is only likely to be beneficial if given within 1 hour of ingestion of paracetamol.If the patient complains of tinnitus, this suggests concurrent salicylate consumption, which requires specific treatment according to the plasma level. Hepatic encephalopathy rarely occurs less than 48 hours from consumption; reduction in level of consciousness in the first 24-48 hours is usually a result of concurrent consumption of sedative drugs (particularly opiates in combination drugs such as co-dydramol of co-proxamol) or hypoglycaemia. Regular monitoring of blood glucose is recommended for patients with elevated paracetamol level requiring treatment with N-acetyl cysteine, or following any change in conscious level. C : 22. Which of the following is NOT a strong indication for urgent CT scanning prior to lumbar puncture in a patient with meningism? A: Abnormal level of consciousness B: AIDS C: Abrupt onset of headache without fever D: Age 150 micromols. Similarly, it should be withdraw temporarily in the following instances: Acute tissue hypoxia – MI, sepsis, and after ionic contrast media radiological studies. C : 27. You see a patient in the outpatient clinic with multiple myeloma whose recent blood tests have shown a serum potassium of 5.7mmol. His recent drug history includes the use of melphalan, interferon, and pamidronate, and he is clinically well, with no ECG changes. Which one of the following oral agents would you prescribe for treatment of hyperkalaemia in this patient? A: Calcium carbonate B: Calcium polystyrene sulphonate (Resonium) C: Sodium polystyrene sulphonate (Resonium) D: Bendrofluazide E: Salbutamol.

348

MOHAMMED IS-HAG

Comment : Ion-exchange resins may be used to remove excess potassium in mild to moderate hyperkalaemia. They can either be calcium or sodium based, with the site of ion exchange taking place mainly in the large intestine, thus allowing either oral or rectal administration. Calcium based resins may provoke hypercalcaemia in patients with multiple myeloma, or metastatic cancer and are best avoided in such instances (especially as this patient has recently received pamidronate, which suggests that hypercalcaemia has been a problem). Salbutamol can be given by nebuliser for more severe cases of hyperkalaemia, especially in patients who are fluid overloaded. C : 28. A 73-year-old man presents to A&E with drowsiness and confusion. He is noted to be tachycardic and tachypnoeic. His pulse oximeter reading is 90% on room air and he is not cyanosed. Apparently his wife had been admitted with similar symptoms earlier in the week. Which one of the following is most likely? A: Paracetamol overdose B: Salicylate overdose C: Carbon monoxide poisoning D: Cerebrovascular accident E: Pneumonia. Comment : This man has carbon monoxide (CO) poisoning. Pulse oximeters cannot distinguish between COHb and HbO2, therefore it is essential to take arterial blood gases and – to make the specific diagnosis – measure the level of CO. It is important to think about prevention: CO alarms are cheap and readily available. C : 29. A 60-year-old woman is admitted feeling generally unwell. Her serum potassium is found to be elevated at 7.0 mmol/l. Which of the following drugs is the LEAST likely to have contributed to her hyperkalaemia? A: Lisinopril B: Bendrofluazide C: Losartan D: Spironolactone E: Slow K. Comment : Bendrofluazide tends to cause hypokalaemia. The other drugs listed may cause hyperkalaemia, although this is not usually clinically significant when used alone at therapeutic doses. The development of significant drug-induced hyperkalaemia is more likely when more than one of these agents is used in combination, or if the patient has co-existing renal impairment. B : 30. A cachectic 87-year-old man is admitted with acute urinary retention and is found to have a creatinine of 520 micro mol/l. He has been on a number of medications. Which of the following is least likely to be nephrotoxic in this situation? A: Ibuprofen B: Ramipril C: Allopurinol D: Lansoprazole E: Sulfasalazine. Comment : As he has muscle wasting, a creatinine this high probably indicates severe renal impairment. This should improve with catheterisation and a good fluid input, but it is essential to stop all potentially nephrotoxic drugs. Do not rely on your memory -always check with the BNF as continuing a nephrotoxic drug at this stage may lead to irreversible renal failure. Of this list, only lansoprazole should be continued until his renal function has improved. D : 31. A 20-year-old woman, previously fit, well and with no significant past medical history, presents 10 weeks into pregnancy with a painful swollen calf. Ultrasound examination confirms that she has a deep venous thrombosis. How should this be managed up to the time of delivery? A: Tubigrip stocking; avoid anticoagulation. B: Initiate and then continue treatment with warfarin until delivery. C: Initiate and then continue treatment with heparin until delivery. D: Initiate treatment with heparin; load with and convert to warfarin, continued until delivery. E: Initiate and then continue treatment with heparin and warfarin until delivery. Comment : During a critical window of 6-9 weeks of pregnancy warfarin causes (warfarin embryopathy) manifest, most obviously as hypoplasia of the nose and limbs. After this period warfarin is associated with neurological damage – mental retardation, microcephaly, optic atrophy and blindness – that may be haemorrhagic in origin and hence reduced by tight anticoagulant control. Warfarin is also associated with increased risk of abruptio placentae in later pregnancy and increased bleeding in mother and fetus if used at term. Most physicians would manage this woman with low molecular weight heparin throughout her pregnancy, although this is not without problems, in particular of maternal osteoporosis. Those that would use warfarin would not continue its use to term but switch to heparin. C :

349

MOHAMMED IS-HAG

32. In a healthy volunteer study, the diuretic response to intravenous doses of loop diuretics A, B and C were compared. A 1mg dose of diuretic A produced a similar diuresis to 40 mg of diuretic B and 50mcg of diuretic C. It was found that maximal doses of A produced a similar diuresis to maximal doses of B, but the response obtained with maximal doses of C was considerably lower than that of A or B. Which of the following statements is correct in relation to the action of A, B and C: A: Drugs A and B are of similar potency. B: Drug C is more potent than drug A but of lower efficacy. C: Drug C is of lower potency than drugs A and B but of greater efficacy. D: Drug A is of greater efficacy than drugs B and C. E: No conclusion can be drawn regarding the relative efficacy and potency of the three drugs. Comment : The potency of a drug relates to the amount of drug needed to produce a given effect. An equivalent diuresis to that seen with drugs A and B is produced with a considerably smaller dose of C. It is therefore more potent. Efficacy relates to the maximal response that can be produced by the drug when taken to high levels. Despite increasing the dose, drug C is unable to produce an equivalent maximal diuresis to that obtained with drugs A or B, it is thus of lower efficacy. The potency of a drug is not often of importance but can become significant if the drug has low solubility and has to be packaged or delivered in such a way that space is limited. Examples might include metered dose aerosols or depot injections. B : 33. A 74-year-old woman presents with breathlessness. She is a small woman (55 kg) with a chest infection. She is not very unwell, but is in atrial fibrillation at a rate of 170/min. Her electrolytes are normal (K 4.2 mmol/l). As well as treating her pneumonia, you decide to digitalize by prescribing: A: Digoxin 0.25 mg orally once daily B: Digoxin 1.0 mg orally over 24 hours in divided doses C: Digoxin 1.0 mg intravenously over 20 min D: Digoxin 0.125 mg orally once daily E: Digoxin 0.25 mg orally three times daily for one week, then twice daily for one week, then once daily thereafter. Comment : The options for treatment of atrial fibrillation are: 1. DC cardioversion if the patient is compromised haemodynamically or has ischaemic cardiac pain 2. Digoxin - 1.0-1.5 mg orally in divided doses over 24 hours, but can be given intavenously in emergency (0.25-0.5 mg over 10-20 min, repeated after four to eight hours to total intravenous loading dose of 0.5-1.0 mg) 3. 'Medical cardioversion' with amiodarone or flecainide In this clinical context it is likely that the atrial fibrillation (if new) will revert to sinus rhythm as the woman recovers from her pneumonia and most physicians would digitalize in preference to the other options described. B: 34. A 38-year-old man presents with acute renal failure, with serum creatinine 988 micromol/l. A house physician performs arterial blood gas analysis (breathing air) and finds pH 7.12, pO2 12.8 kPa, pCO2 3.2 kPa, BE -12 mmol/l. He asks you what a base excess of –12 means. You reply: A: It means that the serum bicarbonate concentration is 12 mmol/l. B: It means that the serum bicarbonate concentration is 12 mmol/l below normal. C: It means that the pH is 0.12 units below the lower limit of the normal range for the machine being used. D: An algorithm is used to predict what pH would arise in normal blood in the presence of the pCO2 actually measured, the base excess being the amount of acid that would have to be added or removed to obtain the pH actually measured. E: An algorithm is used to predict what pH would arise in normal blood in the presence of the pCO2 actually measured, the base excess being the amount of base that would have to be added or removed to obtain the pH actually measured. Comment : The base excess is a figure calculated by many blood gas machines to aid interpretation of data. The principles of the calculation are as follows: predict the pH that would arise in normal blood in the presence of the pCO2 actually measured; then calculate the amount of acid or base that would have to be added to the blood to change the calculated pH into the pH as actually measured. This value is the base deficit or excess, in mmol/l, which quantifies the metabolic component of acid-base disturbance. Renal failure causes metabolic acidosis with compensatory respiratory alkalosis. In this man the predicted pH (on the basis of measured pCO2 3.2 kPa) would be alkalotic, and acid would have to be added to the blood to change this to the acidotic value actually measured (pH 7.12). This is expressed as a NEGATIVE base excess. E :

350

MOHAMMED IS-HAG

35. A man is brought to the accident and emergency department by ambulance. He is unconscious (GCS 5) with pin-point pupils and a slow respiratory rate. Immediate specific treatment should be: A: Naloxone (0.4 mg) intravenously, repeated if no effect B: N-acetyl cysteine (150 mg/kg over 15 min) intravenously, then 50 mg/kg over 4 hours, then 100 mg/kg over 16 hours C: Dextrose (50 ml of 50% solution) intravenously, repeated if no effect D: Naloxone (4 mg) intravenously, repeated if no effect E: Insert stomach tube (after securing airway) and give activated charcoal. Comment : The working diagnosis must be opioid overdose, the treatment for which is intravenous naloxone (0.4 mg), repeated up to a total dose of 2 mg depending on clinical response. The half-life of naloxone is shorter than that of opioids, hence if this man wakes up it can be anticipated that he will 're-narcose'. A naloxone infusion may be necessary. A : 36. A 56-year-old man is admitted to hospital with crushing central chest pain. He is known to have had a myocardial infarction (MI) 3 months ago. Despite treatment with simvastatin in addition to other usual cardiac medication, his ECG shows that he has had a second MI. Which of the following statements is LEAST accurate? A: The overall effect of a drug depends on the net effect on polymorphisms within its metabolic pathways and genetic variation in target cells. B: There is an interaction between apo E genotype and lipid response to statin therapy. C: When a genetic variation affects more than 1% of the population it is termed a polymorphism. D: Lack of concordance with treatment is a possible explanation. E: Data from RCTs show that all MI patients should receive a statin. Comment : A, B and C are correct statements about pharmacogenetics, and D is also true. E looks sensible, but remember that RCTs show what happens in a group of patients in comparison with a group of controls. In any trial showing the survival advantage on a drug, some individuals will have a greater than average response, some less than average and some will be harmed. At present we cannot easily distinguish between these groups, so all patients are given a statin. E : 37. A 17-year-old male fails to breathe spontaneously after an operation. Talking to his family, his sister has previously had similar problems. Which of the following drugs could have caused this problem? A: Thiopentone B: Atracurium C: Suxamethonium D: Cisatracurium E: Halothane. Comment : Suxamethonium is a depolarising neuromuscular blocking agent that is metabolised by plasma pseudocholinesterases. Approximately one in 2500 individuals have deficiency of this enzyme, resulting in prolonged neuromuscular blockade if given suxamethonium. Management of these patients is by prolonged ventilation until the action of the drug wears off. Relatives of affected patients should be screened. C : 38. A patient is taking morphine sulphate continuous (MST) 240 mg b.d. for carcinoma of the prostate with painful bone metastases. What dose of oramorph for breakthrough pain would you prescribe? A: 40 mg B: 10 mg C: 100 mg D: 80 mg E: 60 mg Comment : The breakthrough dose of short acting morphine should be 1/6th of the total 24-hour dose. There should be no time limit on the prescription. D : 39. A 45-year-old man is taking long-term theophylline for asthma. One evening, he is admitted to the Accident and Emergency department with convulsions. You suspect theophylline toxicity. Which one of the following statements is true? A: His convulsions should not be treated until a theophylline level is available. B: Theophylline toxicity may have been precipitated by the concomitant prescription of phenytoin. C: Theophylline toxicity may have been precipitated by the concomitant prescription of erythromycin. D: Theophylline toxicity only occurs in the elderly. E: Theophylline is an example of a drug with a wide therapeutic range (therapeutic index). Comment : Convulsions should be treated immediately in the usual way, without waiting for confirmation of the theophylline level.Theophylline is metabolised by the CYP450 enzymes in the liver. Erythromycin inhibits CYP450 enzymes and increases the half-life of theophylline and hence plasma

351

MOHAMMED IS-HAG

theophylline concentrations, which may lead to toxicity. By contrast, phenytoin induces CYP450 enzymes, which will decrease the half-life of theophylline and may lead to inadequate therapeutic levels.Theophylline toxicity is more likely in the elderly due to age-related reduction in the rate of its metabolism, but it can occur at any age.Theophylline is an example of a drug with aNARROW therapeutic range. It is recommended that plasma theophylline levels be maintained between 10 and 20 mg/l. C : 40. In a patient with normal renal function, a dose of 40mg of furosemide (frusemide) will initiate a significant diuresis. By contrast, in a patient with advanced chronic renal failure, a dose of 250mg furosemide may be needed to have the same effect. Why is this? A: Absorption of furosemide from the gut is reduced. B: Renal blood flow is reduced. C: Tubular secretion of furosemide is reduced. D: The NaK2Cl co-transporter in the renal tubule is downregulated. E: Renal metabolism of furosemide is increased. Comment : Furosemide is secreted into the renal tubule by the same mechanism with which weak organic acids are excreted and works from within the tubular lumen by inhibiting the action of the NaK2Cl co-transporter in the medullary thick ascending limb. Organic acids accumulate in renal failure and compete for tubular secretion with furosemide. This competition can be overcome by using a larger dose of the drug. C : 41. Drug metabolism is often affected by renal function. Serum creatinine is often used as a measure of renal function, but creatinine concentration is affected by factors other than the creatinine clearance, the most commonly used clinical method of determining glomerular filtration rate (GFR). The Cockcroft and Gault formula makes allowance for some of these factors to provide an estimate of creatinine clearance from the serum creatinine concentration. Which factors, apart from serum creatinine concentration itself, are taken into account when calculating creatinine clearance (and therefore GFR) by the Cockcroft and Gault formula? A: Sex, age, body weight B: Sex, body weight, height C: Age, body weight, height D: Sex, age, body weight, height E: Sex, age, body mass index Comment : The formula for determining a male’s creatinine clearance is (1.1 x [150-age] x body weight in Kg) / (72 x serum creatinine concentration in micromol/l). The formula for determining a female’s creatinine clearance is (0.9 x [150-age] x body weight in Kg) / (72 x serum creatinine concentration in micromol/l). A : 42. A 35-year-old man presents with pneumonia requiring antibiotic treatment, but is noted to suffer from acute intermittent porphyia. Which of the following antibiotics would be the best treatment choice? A: Cephradine B: Amoxicillin C: Erythromycin D: Cefotaxime E: Flucloxacillin. Comment : Prescribing in patients with acute porphyrias can be difficult, since drug classes or individual drugs can precipitate acute attacks. The antibiotic classes of cephalosporins and sulphonamides precipitate acute attacks. Additionally, small changes in chemical structures of drugs can change the porphogenicity of drugs within a class (amoxicillin is safe, but flucloxacillin is not). Further information is available from the Welsh Medicines Information Centre in Cardiff. B : 43. A patient with chronic Hepatitis C is referred to your clinic for anitiviral treatment. However, interferon-alfa and ribavirin may not be suitable for those patients who: A: have moderate-severe disease B: were previously intravenous drug users C: are heavy alcohol drinkers D: are haemophiliacs and have not had a liver biopsy E: have relapsed following initial response to interferon-alfa monotherapy. Comment : Combination therapy with interferon-alfa and ribavirin is generally recommended for those with moderate-sever disease (histological diagnosis of significant scarring and/or significant necrotic inflammation). While NICE guidance suggests that problems with drug interactions, safety, and compliance may arise in existing intravenous drug users, those who have given up the habit should not be excluded from therapy. However, treatment is not generally recommended in those patients who consume large quantities of alcohol, given the increased risk of liver damage. In cases where a liver biopsy carries a high risk (e.g. haemophilia), treatment can be initiated without histological confirmation.Both treatment-naïve patients and those who have relapsed following initial response to interferon-alfa should be considered for 6 months of combination therapy. C :

352

MOHAMMED IS-HAG

44. A patient presents with acute dystonia and oculogyric crisis after being treated with metoclopramide. Which statement is true with regards to this adverse drug reaction? A: It occurs only after long-term use of metoclopramide. B: It is most common in middle-aged men. C: It can persist for several days after withdrawal. D: It does not occur with prochlorperazine. E: It is best treated with procyclidine. Comment : Dystonic reactions are well-recognized with dopamine receptor antagonists. They occur shortly after starting therapy, particularly in girls and young women as well as the elderly. The problem usually subsides within 24 hours following cessation of treatment and can be treated with procyclidine 5-10 mg i.m. (antimuscarinic). E : 45. A 24-year-old Type I diabetic is currently on a basal-bolus regime, comprising twice a day basal Isophane insulin, complemented by short-acting insulin at meal times. He has recently heard about insulin glargine, and wondered if it would be suitable for him. Which statement concerning insulin glargine is true? A: It is formulated by adding zinc suspension to insulin B: It is rapid-acting and should be injected just before meals C: It is particularly useful for patients troubled by hypoglycaemic episodes D: It needs to be mixed thoroughly before injecting E: It has little effect on fasting blood glucose. Comment : Insulin glargine is a long-acting insulin analogue, produced by modifying the chemical structure of insulin. This gives it a smooth, prolonged absorption profile with no peaks. As such, it is a long-acting agent, suitable for providing a basal level of insulin which attempts to mimic the normal physiological state. Its smooth profile reduces the risk of hypoglycaemia, and when given at night, provides good control of the fasting blood glucose. Unlike crystalline suspensions, insulin glargine does not need to be mixed thoroughly prior to injection, thus making it easier to use. C : 46. A 28-year-old man with asthma presents with an acute attack. He is very breathless and cannot complete sentences. Which of the following is the best immediate management? A: Nebulised salbutamol (5 mg) driven with air B: Organise chest radiograph to exclude pneumothorax C: Nebulised salbutamol (5 mg) driven with high flow oxygen via reservoir bag D: Nebulised salbutamol (50 mg) driven with 35% oxygen E: Nebulised salbutamol (5 mg) driven with 35% oxygen. Comment : Beta2-agonists are pulmonary vasodilators as well as bronchodilators. Their administration can rapidly worsen the V/Q mismatch which is the cause of hypoxia in asthma. They can therefore cause reduction in arterial oxygen tension unless supplemental oxygen is given, and this man should be given the highest inspired oxygen concentration that can be obtained.It would also be reasonable to mix ipatropium bromide (500 microg) with the salbutamol given in option C. C : 47. A 40-year-old woman has developed haemolytic anaemia secondary to drug therapy. Which of the following drugs is NOT a well-recognised cause of haemolytic anaemia? A: Phenoxymethylpenicillin B: Mefenamic acid C: Methyldopa D: Ranitidine E: Rifampicin. Comment : Several drugs cause haemolytic anaemia, by a variety of mechanisms. Penicillin binds covalently to the red cell membranes; rifampicin causes immune complex association with red cell membranes leading to complement activation; methyldopa and mefenamic acid may induce the formation of autoantibodies against components of red cells. Ranitidine does not commonly cause haemolytic anaemia. D : 48. An 87-year-old woman is admitted after being found wandering in her nightie. She says she is looking for her cat. On examination, she smells strongly of alcohol, but is otherwise well. Her medications include paroxetine for depression, lorazepam for anxiety (she is not sure how many she is taking), cimetidine for longstanding peptic ulcer disease and thyroxine 50mcg daily. Which medication can be stopped immediately? A: Paroxetine B: Lorazepam C: Alcohol D: Cimetidine E: Thyroxine. Comment : This patient has an acute confusional state, and all causes of this should be excluded, especially infection.

353

MOHAMMED IS-HAG

It is important to remember that many psychiatric drugs should not be stopped precipitously and these include selective serotonin reuptake inhibitors (SSRIs) such as paroxetine and benzodiazepines (such as lorazepam). If the dose of these medications needs to be changed, this must be done very slowly, otherwise they can produce an acute withdrawal state with worsening confusion and agitation. The same applies to alcohol. Her family can bring in her usual tipple, so that the nurses can monitor intake. Otherwise, prescribe chlordiazepoxide in reducing doses.It is important to check her thyroid-stimulating hormone (TSH) level to ensure she is on the correct dose of thyroxine, but this is unlikely to be the cause of the confusion. Cimetidine has been shown to cause confusion in older people.Does she still need ulcer healing treatment? If so, consider a proton pump inhibitor (PPI). D : 49. A 48-year-old woman with a renal transplant is established on ciclosporin, azathioprine and prednisolone to prevent transplant rejection, and enalapril and bendrofluazide for hypertension. After a 14-day course of ketoconazole for oesophageal candidiasis her creatinine is found to have increased from 100 mmol/L to 400 mmol/L.Her deterioration in renal function is most likely attributable to: A: hypertension poorly controlled on enalapril and bendrofluazide B: nephrotoxic effects of ketoconazole C: ciclosporin toxicity due to inhibition of ciclosporin metabolism by ketoconazole D: transplant rejection due to induction of ciclosporin metabolism by ketoconazole E: effect of enalapril on background of stenosis of artery supplying renal transplant. Comment : Ketoconazole inhibits liver enzymes that metabolise cyclosporin, increasing plasma cyclosporin concentration. Cyclosporin is nephrotoxic; ketoconazole may be hepatotoxic. A deterioration may also be seen in renal function if the patient has severe hypertension, which could be caused by cyclosporin and/or prednisolone therapy, or is started on enalapril on a background of renal artery stenosis. However in this case the close relationship between the timing of the course of ketoconazole and the increase in creatinine makes option C the most likely. C : 50. A 38-year-old woman with a previous history of essential hypertension presents in the sixth week of pregnancy requesting advice about drug therapy for hypertension in pregnancy. Which one of the following statements is true? A: Blood pressure usually rises during the 2nd trimester of pregnancy, hence additional drug therapy may be required. B: Pre-eclampsia is no more common in women with pre-exisiting essential hypertension than previously normotensive women. C: Methyldopa is contra-indicated during pregnancy. D: Beta blockers may restrict foetal growth if given during pregnancy. E: Anti-hypertensive therapy should always be avoided in early pregnancy. Comment : Blood pressure usually falls during the second trimester of pregnancy before rising again in the third trimester. Women with pre-existing hypertension have a slightly higher risk of developing pre-eclampsia than those who are normotensive.Methyldopa, labetalol and (increasingly) nifedipine are usually the antihypertensive treatments of choice in pregnancy. Methyldopa is often poorly tolerated due to side-effects. ACE inhibitors and angiotensin receptor antagonists are contra-indicated as they may cause renal agenesis and foetal loss. Beta blockers may restrict foetal growth but are sometimes used. In some cases anti-hypertensive therapy is necessary in early pregnancy, with the risks and benefits of any drug therapy carefully considered and discussed with the patient. In some cases antihypertensive therapy is necessary in early pregnancy, with the risks and benefits of any drug therapy carefully considered and discussed with the patient. D : 51. A 5-year-old girl presents having ingested some of her mother's ferrous sulphate tablets. Which of the following is true? A: Whole bowel irrigation is appropriate in all cases. B: Gastrointestinal toxicity occurs 24 hours post ingestion. C: Serial iron concentrations should be measured after desferrioxamine has been given. D: Activated charcoal should be given if presenting less than 1 hour after ingestion. E: Desferrioxamine can be given i.m. Comment : Iron tablets are radio-opaque on abdominal radiographs until they have dissolved. In patients where tablets are seen in the stomach and the small intestine then either gastric lavage or whole bowel irrigation should be considered respectively. Iron is not adsorbed by activated charcoal. Following ingestion toxicity can be divided into phases: · Phase 1 - Gastrointestinal toxicity within 30 minutes to hours · Phase 2 - Latent asymptomatic phase · Phase 3 - Lethargy, neurological toxicity, renal failure, pulmonary oedema, hepatic failure and DIC can occur after 12 to 48 hours · Phase 4 - Gastrointestinal strictures occur 2 to 5 weeks after ingestion.Indications for desferrioxamine include a serum iron concentration of greater than 90micromol/L or significant hypotension, metabolic acidosis and neurological features. Desferrioxamine should be given i.v. but in exceptional circumstances can be given i.m. Following administration, standard biochemical measures of iron concentration are not useful as these will measure both free and chelated iron. E :

354

MOHAMMED IS-HAG

52. A 39-year-old lady who who has a past history of treated hypertension is in her 3rd trimester of pregnancy and requires on-going anti-hypertensive treatment. Which antihypertensive would you definitely not prescribe? A: Hydralazine B: Labetalol C: Lisinopril D: Methyl-dopa E: Nifedipine. Comment : Evidence underpinning the choice of anti-hypertensive therapy in pregnancy is inadequate to make firm recommendations. There are no reports of serious effects with methyl-dopa following long and extensive use. Calcium antagonists, labetalol and hydralazine are commonly used, particularly for resistant hypertension in the third trimester. However, angiotensin-converting enzyme (ACE)-inhibitors should be avoided because they may cause oligohydramnios, renal failure and intra-uterine death. C : 53. A 21-year-old university student complains of difficulty sleeping. She is in the middle of sitting her final exams and would like some medication for a few days to help her sleep. However, she is concerned about potential 'hang-over' effects and would prefer a drug which doesn't cause daytime drowsiness. Which agent would you prescribe? A: Diazepam B: Midazolam C: Promethazine D: Loprazolam E: Clomethiazole. Comment : Diazepam has a long half-life, principally because of its active metabolites. Midazolam is short-acting but is only used intravenously. Promethazine is an antihistamine with a 12-hour half-life and may cause daytime sedation. Clomethiazole is less safe in overdose, has dependence potential and is only licensed for sedation in the elderly. Loprazolam is short-acting (half-life 6–12 hours) and would be a reasonable choice. D : 54. A 38-year-old asthmatic woman presents with an acute attack. Her arterial blood gases breathing air are as follows: pH 7.36, pO2 9.8 kPa, pCO2 5.2 kPa. These mean: A: the attack is not severe B: she should be given supplemental oxygen, but is unlikely to need a high FiO2 to achieve normoxia C: cardiorespiratory arrest could be imminent D: her respiratory effort may be failing because she is getting tired E: she could have had a pneumothorax. Comment : A normal or elevated pCO2 in an asthmatic indicates failing respiratory effort, and although this woman's oxygen saturation is not severely depressed she is in danger of decompensation and - aside from high flow oxygen, nebulised salbutamol and ipatropium, and steroids - it would be prudent to inform the ICU of her existence. The gases are not bad enough, however, to suggest that cardiorespiratory arrest is imminent.Pneumothorax must be excluded in any asthmatic, but the presence or absence of pneumothorax can never be inferred from arterial blood gas analysis. D : 55. A 79-year-old lady presents to A&E with confusion, headache and tinnitus. Her GP has recently started her on an analgesic and you are worried she may have taken too much. Which of the following would most likely explain her symptoms? A: Paracetamol B: Aspirin C: Voltarol D: Co-codamol E: Codeine phosphate. Comment : Aspirin in excess causes symptoms of nausea, vomiting, headache, confusion and tinnitus or hearing difficulties. Whilst the co-codamol and codeine phosphate could cause confusion, they would not cause the tinnitus. All analgesics taken for a prolonged period of time can lead to an analgesic-induced headache. B : 56. A 73-year-old gentleman with dementia attends clinic with his wife. She has heard about memantine and wonders if it would be suitable for her husband. Which of the following are true of memantine? A: It has no interaction with amantadine. B: It is licensed for patients with all types of dementia. C: It inhibits renal excretion of ranitidine. D: It enhances the effects of barbiturates. E: It is an acteylcholinesterase inhibitor. Comment : Memantine is the first licensed NMDA receptor antagonist for the management of moderate to severe Alzheimer's disease. There is some published evidence that memantine has small

355

MOHAMMED IS-HAG

benefits in reducing deterioration in patients with Alzheimer's disease, but little evidence for use in other types of dementia. Several drug interactions are known: · NMDA antagonists “N-Methyl-Daspartate” (e.g. ketamine, amantadine) can precipitate psychosis · Dopamine agonists - effects enhanced · Barbiturates and neuroleptics - effects reduced · Drugs excreted by cationic transporters in the kidney (e.g. ranitidine, quinine, nicotine) - excretion reduced leading to higher plasma concentrations C : 57. A 16-year-old girl has been referred from her GP. She states that she has taken thirty paracetamol tablets (500mg) over the last 12 hours. She is currently seeing the psychiatrists for outpatient management of anorexia. Which of the following is the most appropriate immediate management? A: Perform gastric lavage and check paracetamol levels four hours after the last tablet was taken B: Give activated charcoal and check paracetamol levels four hours after the last tablet was taken C: Give activated charcoal and check paracetamol levels immediately D: Give activated charcoal and start n-acetyl cysteine E: Wait for the results of paracetamol levels before instituting treatment. Comment : Paracetamol levels are very difficult to interpret if the patient has taken the tablets over a period of time. This girl is at high risk because of the alleged number she has taken and the coexistant anorexia. Gastric lavage is only ever indicated if the patient has taken a large number in the hour or so before presentation. Charcoal will help to prevent absorption. It would be safest to start n-actyl cysteine and check her clotting over the next 24 hours. D : 58. A middle-aged man is brought by ambulance to the Medical Admissions Unit. He was fitting when picked up and is still having a grand mal convulsion. The most appropriate treatment is: A: Lorazepam 2 mg intravenously B: Fosphenytoin 15 mg/kg body weight phenytoin equivalent, intravenously at a rate of 100-150 mg phenytoin equivalent /min C: Phenytoin 15 mg/kg body weight, intravenously at a rate of 50mg/min D: Diazepam 10 mg intravenously E: Phenobarbitone 10 mg/kg body weight, intravenously at a rate of 100 mg/min. Comment : All of these are recognized treatments for status epilepticus. First-line treatment should be with intravenous benzodiazepine, with lorazepam preferred to diazepam because of its longer duration of action. Fosphenytoin is the preferred second-line treatment (phenytoin if this is not available). Phenobarbitone is one of several agents that can be used as third-line treatment, but seek specialist advice if first and second-line treatments are ineffective. A : 59. A 27-year-old woman develops difficulty breathing and her lips and tongue swell about five minutes after starting to eat a curry. She is brought to the accident and emergency department by ambulance. She is cyanosed and wheezing. Aside from high flow oxygen via a reservoir bag, which of the following treatments would be your top priority? A: Hydrocortisone 200 mg intravenously B: Chlorpheniramine 10 mg intravenously C: Epinephrine (adrenaline) - 0.5 ml of 1/1000 solution intravenously D: Epinephrine (adrenaline) - 0.5 ml of 1/1000 solution intramuscularly E: Salbutamol 5 mg nebulized. The history clearly suggests anaphylaxis and treatment with intramuscular epinephrine (0.5 ml of 1/1000) is required.In extremis, epinephrine can be given intravenously, but at reduced dosage: make a 1/10,000 solution (by diluting 1 ml of 1/1000 to 10 ml with 0.9% saline) and give this at 1 ml/min (0.1 mg/min) until a response has been obtained (or a total of 0.5 mg - 5 ml - has been given). D : 60. A 23-year-old male has a diagnosis of methaemoglobinaemia. Which of the following statements is false? A: Pulse oximetry is a not a good guide to his oxygen saturation. B: Ascorbic acid reduces the methaemoglobinaemia. C: Oral activated charcoal should be given for dapsone poisoning. D: Methylene blue is required only if there are signs of toxicity. E: Methylene blue is a cause of methaemoglobinaemia. Comment : Methaemoglobinaemia occurs when haemoglobin is oxidised and is unable to carry oxygen. Drug classes which cause this condition include: · antibiotics (dapsone, sulphonamides and trimethoprim) · nitrites and nitrates · local anaesthetics (lignocaine and prilocaine). Pulse oximeters measure both oxyHb and metHb, therefore giving false reassurance in patients with high levels of metHb, whose measured oxygen saturation does not accurately represent ability to carry oxygen to the tissues. Treatment of patients with less than 30% metHb is removal from the exposure, oxygenation and ascorbic acid (directly reduces the metHb). Methylene blue is used in those with severe poisoning or MetHb levels greater than 30%; excessive doses of methylene blue can themselves cause ethaemoglobinaemia. Dapsone is adsorbed by activated charcoal. D :

356

MOHAMMED IS-HAG

61. You see a 48-year-old Afro-Caribbean man in the outpatient clinic with uncomplicated essential hypertension. His blood pressure today is 154/102mmHg despite optimization of nonpharmacological therapy. Which one of the following drugs would you use as the first-line agent in this patient? A: Atenolol 50mg od B: Nifedipine 10mg tds C: Amlodipine 5mg od D: Ramipril 2.5mg od E: Enalapril 5mg bd. Comment : Non-pharmacological therapy should always be optimized prior to commencement of medication, whenever possible. Hypertension is particularly common in Afro-Caribbeans and associated with particularly higher risk of complications. Therefore effective long-term treatment, with a low threshold for multiple therapy where necessary, is particularly important. Studies indicate that drugs such as ACE (angiotensin-converting enzyme) inhibitors and Beta-receptor antagonists are less effective in Afro-Caribbeans. The reason appears to be related to the finding that the renin-angiotensinaldosterone (RAA) system is commonly suppressed in the majority of Afro-Caribbeans. As such, drugs that suppress the RAA system are less likely to be effective. Calcium-channel blockers (CCBs) and diuretics appear to be more effective in this subgroup. However, diuretics may not be suitable in this case as they are commonly associated with impotence. Short-acting CCBs do not provide prolonged BP control, can cause reflex tachycardia and may be associated with higher mortality. Therefore, longacting CCB should be the first-line drug of choice. Ideally, a once-daily agent with that provides a smooth 24-hour BP control (e.g. Nifedipine LA 30mg od or Amlodipine 5 mg od) to improve compliance would be preferable. C : 62. A 72-year-old man is admitted to the coronary care unit with an acute myocardial infarction. He suffers a cardiac arrest. Basic life support is being given as you arrive. The ECG monitor reveals ventricular fibrillation (VF). The first defibrillation attempt should be made at: A: 200 J B: 400 J C: 360 J D: 20 J E: 100 J. Comment : The European Resuscitation Council guidelines for adult Advanced Life Support suggest that - after Basic Life Support and a precordial thump (if appropriate) - VF/VT should be treated with up to three defibrillation shocks, the first two at 200 J and the third at 360 J. A : 63. A 48-year-old man of African origin presents with blood pressure 200/110mmHg. Urinalysis is negative and fundoscopy shows AV nipping. Which of the following treatment options is most appropriate? A: Oral atenolol 50mg od if hypertension confirmed over 1–2 weeks B: Oral enalapril 10mg bd if hypertension confirmed over 1–2 weeks C: Urgent admission for control of accelerated hypertension D: Oral nifedipine capsules 20mg tds if hypertension confirmed over 1–2 weeks E: Oral nifedipine LA 30mg od if hypertension confirmed over 1–2 weeks Comment : The diagnosis of accelerated hypertension requires the finding of fundal haemorrhages and exudates, with or without papilloedema, as manifestations of fibrinoid necrosis. This patient has very high blood pressure but no evidence of accelerated hypertension. BTS guidelines suggest that the finding of blood pressure 200–219/110–119mmHg should be confirmed with repeat measurements after 1–2 weeks, then treated if still elevated. Black patients have low renin hypertension, therefore ACE inhibitors and beta-blockers as single agents do not lower blood pressure in this group. Calcium channel blockers and diuretics are effective agents. Nifedipine should be used as a long-acting preparation, not a short acting one. E : 64. An 18-year-old woman is brought to the accident and emergency department by a friend, who says that she has been ill for 24 hours with 'flu-like symptoms and headache. She is unwell, drowsy and has a purpuric rash on her arms. Your immediate action is to: A: order a CT scan of her brain B: perform a lumbar puncture C: take blood cultures and await result D: give acyclovir 10 mg/kg intravenously E: give cefotaxime 2 g intravenously. Comment : The working diagnosis must be meningococcal meningitis and the woman must be given appropriate an appropriate antibiotic (e.g. cefotaxime 2 g) intravenously without delay.In the very old or immunocompromised it would be appropriate to add ampicillin 2 g six-hourly to cover Listeria, and acyclovir (10 mg/kg eight-hourly, with dose reduction in renal failure) should also be given if herpes simplex encephalitis is a possibility. E :

357

MOHAMMED IS-HAG

65. A 50-year-old man with metastatic colorectal cancer complains of regurgitation of food and a feeling that it sticks retrosternally. Chest radiograph and upper gastrointestinal endoscopy are normal. Which antiemetic is most likely to be of benefit? A: Cyclizine B: Haloperidol C: Prochlorperazine D: Metoclopramide E: Ondanestron. Comment : It sounds as though there may be a neuro muscular cause for his symptoms. Another possibility is external compression, but the endoscopy did not show this. Metoclopramide is both an antidopaminergic and gastrokinetic agent. It may improve oesophageal motility. D : 66. A 29-year-old man with a history of epilepsy has been well controlled on carbamazepine and clonazepam for the last 5 years. He now wishes to consider withdrawing from or reducing his medication. Which of the following statements are correct? A: There is about a 60% chance of experiencing a relapse in the first year during withdrawing from anti-epilepsy treatment. B: Both anti-epileptics can be safely withdrawn simultaneously. C: The dose of carbamazepine can be reduced safely by 10% every 2-4 weeks. D: He can be advised that he can continue driving during withdrawal from anti-epilepsy treatment as long as she remains free from seizures. E: It is likely that he will subsequently require higher doses to regain control with the current therapy, if discontinuation fails. Comment : Studies suggest that about 25% of patients who have decided to stop their anti-epilepsy treatment relapse within a year of starting to taper down their medication. The likelihood of seizure is greatest during withdrawal and in the subsequent 6 months. The DVLA recommends that patients should not drive during this period. Doses of drugs such as carbamazepine, lamotrigine, phenytoin, sodium valproate and vigabatrin should be reduced by about 10% every 2-4 weeks. Barbiturates, benzodiazepines and ethosuximide should be tapered more slowly by reducing dosage by about 10% every 4-8 weeks. Only one drug should be withdrawn at one time, with a period of 1 month between completing withdrawal of one drug and beginning withdrawal of the next. There is no evidence to support the belief that patients become resistant to their original therapy following discontinuation. C : 67. A 43-year-old woman with pulmonary hypertension attends clinic and asks to be prescribed bosentan. Which of the following is true? A: It is licensed for use in all patients with pulmonary hypertension. B: It inhibits the effects of endothelin-2 (ET-2). C: It binds to both endothelin A (ET-A) and endothelin B (ET-B) receptors. D: It is excreted primarily unchanged in the urine. E: It is not effective in patients with scleroderma. Comment : Bosentan is an antagonist of endothelin-1 binding to ET-A and ET-B receptors. It is licensed for the treatment of 'pulmonary arterial hypertension (PAH) to improve exercise capacity and symptoms in patients with grade III functional status'. It has been shown to be effective in patients with primary PAH and in those with PAH secondary to scleroderma. It is excreted in bile following metabolism by the cytochrome P450 enzymes and this is a potential source of interaction with drugs metabolised by the same isoenzyme (e.g. glibenclamide, ritonavir, ketoconazole, ciclosporin and itraconazole). C : 68. A 72-year-old Caucasian woman in your outpatient clinic has uncomplicated essential hypertension. Her blood pressure is 162/102mmHg despite optimization of non-pharmacological therapy. Which one of the following treatments would you choose as the first-line agent for her? A: Atenolol 50 od B: Bendrofluazide 2.5mg od C: Bendrofluazide 5mg od D: Enalapril 5mg od E: Ramipril 2.5mg od. Comment : Hypertension is particularly common in those aged above 60 not least because of the steady rise in systolic blood pressure with age. These patients are at a high absolute risk of cardiovascular complications. Furthermore, anti-hypertensive treatment may also reduce incidence of heart failure and possibly dementia. Non-pharmacological therapy for hypertension should always be optimised prior to commencement of medication, whenever possible. Low-dose diuretics are accepted as the first-line treatment for hypertension in the elderly and appear to confer greater benefit than Betaadrenergic receptor antagonists in this subgroup. 1. Treatment of isolated systolic hypertension in the elderly with the long-acting calcium channel blocker, nitrendipine, has been shown to reduce stroke and cardiovascular outcome. Therefore, calcium channel blockers may be suitable when diurectics are not tolerated, ineffective or contra-indicated.2 B :

358

MOHAMMED IS-HAG

69. A 65-year-old gentleman attending the cardiology clinic complains of swelling and tenderness of his breasts. You diagnose probable gynaecomastia. Which of the following drugs is most likely to be the cause? A: Simvastatin B: Amiodarone C: Digoxin D: Aspirin E: Ramipril. Comment : Digoxin is the most likely cause of his gynaecomastia. This side effect is more common with longer-term use and may be unilateral or bilateral. Important differential diagnoses to consider include male breast cancer, liver disease, testicular tumours and hyperthyroidism. Other drugs that can cause gynaecomastia include oestrogens, spironolactone, cimetidine, verapamil and nifedipine. The gynaecomastia usually improves on stopping the drug or reducing the dose. C : 70. A 53-year-old woman presents with a digoxin overdose. Which of the following statements is false? A: Tachyarrhythmias do not occur. B: Peak effects can be delayed after ingestion by 6-12 hours. C: Digoxin can precipitate severe hyperkalaemia (>6.5mmol/l). D: DC cardioversion can precipitate intractable ventricular fibrillation (VF) or asystole. E: Activated charcoal reduces absorption. Comment : Digoxin is slowly absorbed, hence peak effects can be delayed by up to 12 hours after overdose. Oral activated charcoal may reduce absorption, although giving multiple doses is controversial. Nausea and vomiting occur early after poisoning; other features include confusion, headache and visual disturbances. Any brady or tachyarrhthymia can occur. Treatment is with correction of hyperkalaemia and atropine for bradyarrhythmias. Digoxin-specific antibodies are useful in i) those with hyperkalaemia resistant to treatment, ii) bradyarrhythmias with hypotension non-responsive to atropine and iii) tachyarrhythmias with hypotension. Anti-arrhythmic drugs and DC cardioversion should be avoided as they can precipitate intractable asystole or VF. A : 71. You have decided to start a syringe driver on a dying patient whose symptoms were previously well controlled on oxycodone SR (OxyContin) 80mg bd. What dose of diamorphine should you chose for your 24-hour syringe driver? A: 30mg B: 60mg C: 90mg D: 110mg E: 130mg. Comment : 160mg oxycodone over 24 hours is equivalent to 320mg morphine which is equivalent to approx 106mg diamorphine D : 72. A 28-year-old male presents following an overdose and anticholinergic syndrome is suspected. Which one of the following is true? A: Tricyclic antidepressants are not a cause B: Bradycardia is common C: Physostigmine is the treatment of choice D: Mydriasis occurs E: Urinary incontinence is common. Comment : Anticholinergic syndrome occurs following overdose with drugs that have prominent anticholinergic activity including tricyclic antidepressants, antihistamines and atropine. Features include dry, warm, flushed skin, urinary retention, tachycardia, mydriasis (dilated pupils) and agitation. Although physostigmine, a reversible inhibitor of acteylcholinesterase, is effective in treating symptoms, there is a significant risk of cardiac toxicity (bradycardia, AV conduction defects and asystole). Treatment therefore consists of withdrawal of the precipitating drug and supportive care. D : 73. A frail 58-year-old lady with advanced breast cancer is admitted with abdominal pain and constipation secondary to opioids. Which of the following is the laxative of choice? A: Sodium Picosulphate B: Fybogel C: Senna D: Codanthramer E: Lactulose. Comment : Bulk forming drugs such as fybogel have little to offer in opioid-induced constipation. Senna can cause abdominal cramps. Codanthramer can cause skin burns in faecal incontinence. This patient is frail and may not be able to clean herself well and is therefore at risk of this most unpleasant complication. Lactulose can cause bowel distension and increased abdominal cramps. A :

359

MOHAMMED IS-HAG

74. In relation to the use of inliximab in inducing remission in inflammatory bowel disease, which of the following statements is correct? A: Infliximab is a monoclonal antibody that inhibits interleukin. B: Infliximab is licensed for severe or fistulating ulcerative colitis. C: Infliximab increases the risk of infection. D: Acute infusion-related reactions are very rare (80%). Comment : Infliximab is a monoclonal antibody that inhibits tumour necrosis factor. It is licensed for severe or fistulating Crohn's disease refractory to corticosteroid or immunosuppressant therapy, but not for ulcerative colitis. Acute infusion-related reactions are common (1-10%) but serious reactions such anaphylactic reactions are uncommon (0.1-1%). Antibodies to infliximab develop in about 24% of patients taking other immunosupressants and in about 37% of those not on such therapy. Infliximab is associated with an increased likelihood of infection and especially tuberculosis (0.1-1% of patients). Therefore, patients should be evaluated for active and inactive tuberculosis prior to treatment; and monitored closely for infections. C : 75. A 24-year-old man with chronic renal failure, for which he receives haemodialysis three times per week, decides to go on holiday. One week later, having missed two dialysis sessions, he presents feeling unwell and breathless. Examination reveals pulmonary oedema. His ECG shows no P waves, broad QRS complexes and peaked T waves. What is the first treatment you should give him? A: 10 U actrapid with 50 ml of 50% glucose intravenously B: 10 ml of 10% calcium gluconate intravenously C: 10 mg salbutamol by nebuliser D: 100 mg lignocaine intravenously E: Transfer to the dialysis unit for haemodialysis. Comment : All of the interventions listed, with the exception of intravenous lignocaine, are recognised treatments for hyperkalaemia, but this man is at clear risk of cardiac arrest and only calcium gluconate acts instantly to 'stabilise' the heart (although it does not alter the serum potassium level).ECG monitoring will show that the ECG assumes a more normal morphology after calcium gluconate is given. Appropriate treatment then would be to give insulin / dextrose or salbutamol whilst arranging urgent transfer to the dialysis unit. B : 76. It has been decided that a patient should be changed from MST 100mg bd to the equivalent transdermal fentanyl dose. Choose the correct dose from those shown below A: 125ug/hr B: 25ug/hr C: 100ug/hr D: 50ug/hr E: 75ug/hr. Comment : Fentanyl Conversion Table Oral 24 hour morphine dose Fentanyl ug/hr Less than 135 mg 25 50 75 100 125 150 175 200 225 250 275 1035-1124 300 D:

360

MOHAMMED IS-HAG

1. Four new drugs are tested for radical cure of vivax malaria. All the new drugs are more expensive and have more severe side effects than primaquine, so you will only change if there is at least a 25% increase in cure rate. Which drug do you recommend having seen these results? (For each one primaquine is the standard treatment against which the new drug is compared.) A: Drug A. Relative Risk of relapse 1.63 (95% confidence interval 1.2-2.3) B: Drug B. Relative Risk of relapse 0.67 (95% confidence interval 0.35-1.2) C: Drug C. Relative Risk of relapse 0.87, p
View more...

Comments

Copyright ©2017 KUPDF Inc.
SUPPORT KUPDF